You are on page 1of 103

Universidad La Salle.

Facultad Mexicana de Medicina.


Curso de extensión universitaria para la preparación del Examen Nacional para
Aspirantes a Residencias Médicas.
Examen Módulo Medicina Interna.
Modalidad a distancia.

1.- Paciente femenino de 28 años de edad, que acude a consulta por presentar dolor en el
recto, sin encontrarse una causa orgánica después de múltiples exploraciones médicas.
Además refiere que desde hace 4 años ha presentado vómitos, dolor abdominal
generalizado, palpitaciones, mareos, disfagia, visión borrosa, dolor en los miembros
inferiores, dismenorrea y dispareunia. Se observa triste y ansiosa. ¿Cuál es el diagnóstico
más probable?

a) Trastorno de ansiedad crónico


b) Depresión crónica con somatización
c) Trastorno de somatización
d) Trastorno de conversión

Referencia:

López- Ibor J J, Valdés M M. Manual diagnóstico y estadístico de los trastornos


mentales (DSM- IV). Masson 2005. 545 – 574.

El Trastorno de somatización se caracteriza por la presencia de muchos síntomas


somáticos que no pueden explicarse por los hallazgos físicos o de laboratorio. Comienza
antes de los 30 años, puede perdurar durante años, es crónico y va asociado a malestar
psicológico, a un deterioro del funcionamiento social y laboral y a la búsqueda excesiva de
ayuda médica. Para hacer el diagnóstico deben presentarse dolor en cuatro zonas del
cuerpo, dos síntomas gastrointestinales, un síntoma sexual y un síntoma pseudoneurológico.
La ansiedad y la depresión son las patologías psiquiátricas más prevalentes.

2.- Una mujer de 25 años de edad presenta fiebre y artritis inflamatoria que afecta a las
articulaciones metacarpofalángicas e interfalángicas proximales ¿Cuál de las siguientes
manifestaciones es muy sugestiva de lupus eritematoso diseminado y no de artritis
reumatoide?

a) Sedimento urinario activo (eritrocitos, leucocitos, cilindros celulares, sin bacterias).


b) Artritis inflamatoria de articulaciones metacarpofalángicas e interfalángicas.
c) Derrame pleural de la radiografía de tórax.
d) Pruebas de funcionamiento hepático anormales.
Referencia:
Allen R. M. MMS Medicina Interna. 5ª. Edición. National Medical Series. Mc. Graw Hill.
2006. (capítulo 10IIG;VIIG l;cuadro 10-10).

Un sedimento urinario activo sugiere glomerulonefritis, dato común en pacientes con lupus
eritematoso diseminado, pero no en quienes tienen artritis reumatoide. Pueden encontrarse
artritis de articulaciones metacarpofalángicas e interfalángicas proximales derrames
pleurales y anemia en cualquiera de las dos enfermedades. Las anomalías de la función
hepática son atípicas en el lupus eritematoso sistémico, pero comúnmente son resultado de
disfunción hepática relacionada con fármacos (p. ej., por antiinflamatorios no esteroideos
en cualquier enfermedad.

3.- Paciente de 52 años de edad con diagnóstico de neumonía adquirida en la comunidad,


quien recibió tratamiento antibiótico a base de Ceftriaxona. No tuvo respuesta adecuada, y
en el estudio diagnóstico para determinar la causa, se encontró un derrame pleural del
60%, el cual se puncionó. En el estudio citoquímico y bacteriológico de este líquido, se
reportan cocos grampositivos en la tinción de Gram, y un pH de 7. Con esto, usted
considera como indispensable:

a) Iniciar cobertura con vancomicina.


b) Colocación de sonda endopleural.
c) Intubación orotraqueal y ventilación con volúmenes altos.
d) Ingreso a Unidad de Terapia Intensiva.

Los hallazgos del citoquímico y tinción son característicos de empiema. El drenaje del
mismo es la maniobra terapéutica base para el tratamiento y resolución del mismo.

Kasper DL, Braunwald E, Fauci AS, Hauser SL, Longo DL, Jameson JL. Harrison´s Principles
of Internal Medicine. McGraw Hill. 16 Ed. 1536 p.

4.- Ante un paciente con diagnóstico de trastorno obsesivo-compulsivo, usted decide


indicar el siguiente fármaco por ser el principal tratamiento de elección:

a) Antipsicóticos.
b) Antidepresivos inhibidores de la recaptación de noradrenalina.
c) Antidepresivos inhibidores de la recaptación de serotonina.
d) Benzodiacepinas.
El abordaje farmacológico del TOC se basa en los inhibidores selectivos de la recaptura de
serotonina (ISRS), medicamentos que han resultado ser efectivos y seguros. Como grupo,
los ISRS son igual de eficaces que la clorimipramina, pero producen menos efectos
secundarios y, por lo tanto, mejor tolerancia y mejor apego al tratamiento. La efectividad
antiobsesiva parece ser independiente de su actividad antidepresiva.
Entre los factores predictores de respuesta al tratameinto, se ha propuesto que los altos
puntajes en las compulsiones predicen una mala respuesta al tratamiento. Para los casos
refractarios y resistentes o cuando hay síntomas de comorbilidad, se han utilizado, con
relativo éxito, combinaciones con diferentes IRS, con benzodiacepinas, o bien, con
potenciadores como el litio, o antipsicóticos, como la risperidona y el haloperidol. El uso del
carbonato de litio es controvertido, aunque parece ser útil como potenciador a largo plazo
entre 15 y 30% de los pacientes. La combinación de ISRS con antipsicóticos comenzó a
utilizarse en los pacientes con síntomas psicóticos, aunque ahora se combinan también en
los pacientes resistentes.
Tratamiento farmacológico del TOC
Cristina Lóyzaga*
Humberto Nicolini*

*División de Investigaciones Clínicas. Instituto Nacional de Psiquiatría.


Ramón de la Fuente. Calzada México-Xochimilco, 101, San Lorenzo
Huipulco, 14370, México D.F.
Primera versión: 14 de septiembre de 2000.
Aceptado: 26 de septiembre de 2000.

5.- Un paciente con carcinoma pulmonar presenta náuseas, vómitos y letargo y se encuentra
que tiene una concentración sérica de calcio de 13.4 mg/100 ml. ¿Cuál de los siguientes
agentes debe ser el primer paso para el tratamiento?

a) Etidronato intravenoso
b) Mitramicina intravenosa
c) Glucocorticoides intravenosos
d) Solución salina y furosemida intravenosos

Allen R. M. MMS Medicina Interna. 5ª. Edición. National Medical Series. Mc. Graw Hill.
2006. (capítulo 9 IIIA 6 a, b, d, 7 c (1), (3)). La hipercalciemia causada por otras
enfermedades aparte del hiperparatiroidismo puede tratarse con solución salina y
furosemida intravenosos. El reemplazo de líquidos con solución salina intravenosa, seguido
por diuresis forzada con solución salina y furosemida intravenosas, es una forma rápida y
segura de disminuir el calcio sérico y debe intentarse primero. Pueden añadirse
pamidronato, mitramicina o calcitonina si se necesita disminuir aún más la concentración de
calcio. Los glucocorticoides son eficaces para tratar la hipercalciemia causada por exceso
de vitamina D, sarcoidosis y algunas neoplasias malignas hemáticas, pero no disminuyen el
calcio sérico en la mayor parte de los casos de hipercalciemia relacionada con tumores
sólidos.

6.- Una mujer de 20 años de edad presenta trombosis venosa profunda de nueva aparición
en la extremidad inferior izquierda. Tiene antecedentes de trombocitopenia leve y dos
abortos; fue tratada para sífilis hace dos años debido a una prueba de reagina rápida del
plasma positiva, aunque el anticuerpo antitreponémico fue negativo. ¿Cuál de los siguientes
trastornos es más probable que represente el conjunto de su historia clínica?

a) Síndrome de anticuerpos antifosfolípido


b) Lupus eritematoso diseminado (SLE)
c) Síndrome de anticuerpo Ro
d) Arteritis de Takayasu

Allen R. M. MMS Medicina Interna. 5ª. Edición. National Medical Series. Mc. Graw Hill.
2006. (capítulo 10VIIFl,2c, G3b). Esta paciente tiene varias manifestaciones sutiles de
síndrome de anticuerpo antifosfolípido, que en conjunto hacen de éste un diagnóstico
probable. La prueba de reagina plasmática rápida (RPR) positiva, con resultados negativos
de la prueba de treponemas, probablemente refleja anticuerpos que producen reacción
cruzada a la cardiolipina o a componentes fosfolípidos de antígenos treponémicos. La
trombocitopenia es frecuente en esta situación, debido a las interacciones de plaquetas y
células endoteliales y a la coagulación inducida por los anticuerpos. Los abortos pueden
deberse a coagulación en vasos placentarios pequeños. Las trombosis venosas profundas o
incluso la coagulación en arterias mayores pueden ser causadas por hipercoagulabilidad
consecuente. No hay otras pruebas de lupus eritematoso diseminado, aunque pueden
encontrarse anticuerpos antifosfolípido en alrededor de 33% de los pacientes.

Tampoco se describen datos de síndrome de anticuerpo Ro o enfermedad indiferenciada


del tejido conjuntivo. La arteritis de Takayasu se presenta en mujeres asiáticas jóvenes,
pero no hay datos de isquemia de grandes vasos arteriales característica de esta
enfermedad.

7.- Masculino de 56 con derrame pleural. Se realiza una toracocentesis y se analiza el


fluido revelando que tiene una diferencia de proteínas con el suero de 0.9 y diferencia de
DHL con suero de 1. Este fluido es un:

a) Trasudado
b) Exudado
c) Derrame
d) Infiltrado
Toracentesis y análisis del líquido pleural. La toracentesis diagnóstica requiere
menos de 30 ml de líquido. En los derrames enquistados es útil la ecografía para localizar
con precisión el líquido, y con ello hacer la toracentesis más fácil y sin riesgos (Ver guía
para drenaje y succión pleural).

Los derrames pleurales se clasifican en trasudados y exudados. Un trasudado es un filtrado


de plasma que resulta del aumento de la presión hidrostálica o de la alteración de la
permeabilidad capilar. Los trasudados se asocian con insuficiencia cardiaca congestiva,
sindrome nefrótico, cirrosis y condiciones de sobrecarga de volumen.

El exudado es un líquido rico en proteínas resultante de una inflamación local o por una falla
en la eliminación de proteínas por los linfáticos o ambos mecanismos. Los exudados se
producen en infecciones colagenopatías y neoplasias.

Existen criterios para la diferenciación entre trasudados y exudados (los exudados deben
cumplir al menos uno de los siguientes criterios):

1. Relación de proteína pleural/sérica > 0.5


2. Relación de LDH pleural/sérica > 0.6
3. LDH pleural > 200 Ul
4. Estos criterios tiene significancia diagnóstica con una sensibilidad del 98% y
especificidad de 77%.

Otros criterios útiles para el diagnóstico son:

a. Colesterol total en líquido pleural mayor de 60 mg/dl


b. Relación de colesterol pleural/sérico mayor 0.4

Otro análisis del líquido pleural de importancia es la concentración de glucosa. Un resultado


bajo (< 60 mg/dl) en el líquido pleural es sugestivo de empiema, neoplasia, TBC, LES o
pleuresía reumática.

El pH normal es de 7.60 encontrándose menor a 7.30 en las mismas entidades patológicas


descritas para la glucosa, y en la ruptura esofágica; en los derrames por neoplasia un pH
bajo se relaciona con menor sobrevida y menor respuesta a la pleurodesis química.

La medición de triglicéridos es útil ante la sospecha de quilotórax (> 110 mg/dl).

La medición de adenosina deaminasa permite la diferenciación de TBC pleural y neoplasia


cuando es mayor de 45 UI.
El recuento y la diferenciación celular ayudan al diagnóstico etiológico del derrame. Los
trasudados en general tienen menos de 1.000 leucocitos/ml; recuentos mayores a
10.000/ml se ven en derrames paraneumónicos, mayores de 50.000/ml en empiema. Los
derrames crónicos (TBC, neoplasia) tienen menos de 5.000/ml. La linfocitosis es indicativa
de TBC, neoplasia, linfoma, sarcoidosis, pleuresía reumática. Se encuentra predominio
neutrofílico en neumonía, embolía y pancreatitis.

Ante la sospecha de neoplasia, se debe solicitar una citología del líquido pleural, la cual
tiene una sensibilidad importante.

LECTURAS RECOMENDADAS

American College of Physicians. Diagnostic thoracentesis and pleural biopsy in pleural


effusions Ann Inter Med 103:799, 1985.

Camacho Durán F, Restrepo Molina J. Enfermedades de la pleura. En: Fundamentos de


Medicina. Neumología. Tercera Edición.

Corporación para Investigaciones Biológicas. Medellín, 1986.

Des Jardins T. Enfermedades pleurales En: Enfermedades Respiratorias. Terry Des


Jardins Editor.

Editorial El Manual Moderno SA México DF, 1993.

Light RW, Mac Gregor M. The diagnostic separation of trasudates and exudates. Ann
Intern Med 77: 507, 1972.

Pacheco PM. Estudio del derrame pleural En: Enfermedades del Tórax. Fidel Camacho,
Jaime Páez, Carlos Awad Editores. Ediciones Médicas Zambón. Santafé de Bogotá, 1992.

Patiño JF, Arroyo de S. Guía práctica de toracentesis y de toracostomía cerrada (inserción


de tubo de tórax).Trib Médica 89:161, 1994.

Patiño JF, Arroyo de S. Guía para drenaje y succión pleural. Manejo del drenaje pleural.

8.- Se trata de paciente de 30 años que refiere datos de ansiedad, enojo e inseguridad
refiriendo que existe una constante sensación de que en su trabajo no realiza
correctamente las funciones que se encomiendan. Esto hace que necesite permanentemente
revisarlas una y otra vez, lo que le supone pérdida de tiempo y eficacia. Esta sensación es
tan dominante en su psiquismo que le conduce a la idea de fracaso y a la pérdida de
autoestima. El diagnóstico inicial corresponde a trastorno:
a) Psicótico paranoide
b) Por ansiedad fóbica
c) De personalidad evitativo-fóbica
d) Obsesivo-compulsivo

CRITERIOS DEL DSM-IV PARA EL DIAGNÓSTICO DEL TRASTORNO OBSESIVO-


COMPULSIVO. Fuente AMERICAN PSYCHIATRIC ASSOCIATION.

Criterios para el diagnóstico de F42.8 Trastorno obsesivo-compulsivo (300.3)

A. Se cumple para las obsesiones y las compulsiones:

Las obsesiones se definen por 1, 2, 3 y 4:

1. Pensamientos, impulsos o imágenes recurrentes y persistentes que se experimentan


en algún momento del trastorno como intrusos e inapropiados, y causan ansiedad o
malestar significativos.

2. Los pensamientos, impulsos o imágenes no se reducen a simples preocupaciones


excesivas sobre problemas de la vida real.

3. La persona intenta ignorar o suprimir estos pensamientos, impulsos o imágenes, o


bien intenta neutralizarlos mediante otros pensamientos o actos.

4. La persona reconoce que estos pensamientos, impulsos o imágenes obsesivos son el


producto de su mente (y no vienen impuestos como en la inserción del pensamiento)

Las compulsiones se definen por 1 y 2:

1. Comportamientos (p. ej., lavado de manos, puesta en orden de objetos,


comprobaciones) o actos mentales (p. ej., rezar, contar o repetir palabras en
silencio) de carácter repetitivo, que el individuo se ve obligado a realizar en
respuesta a una obsesión o con arreglo a ciertas reglas que debe seguir
estrictamente.

2. El objetivo de estos comportamientos u operaciones mentales es la prevención o


reducción del malestar o la prevención de algún acontecimiento o situación negativos;
sin embargo, estos comportamientos u operaciones mentales o bien no están conectados
de forma realista con aquello que pretenden neutralizar o prevenir o bien resultan
claramente excesivos.
B. En algún momento del curso del trastorno la persona ha reconocido que estas obsesiones
o compulsiones resultan excesivas o irracionales.

Nota: Este punto no es aplicable en los niños.

C. Las obsesiones o compulsiones provocan un malestar clínico significativo, representan


una pérdida de tiempo (suponen más de 1 hora al día) o interfieren marcadamente con la
rutina diaria del individuo, sus relaciones laborales (o académicas) o su vida social.

D. Si hay otro trastorno, el contenido de las obsesiones o compulsiones no se limita a él (p.


ej., preocupaciones por la comida en un trastorno alimentario, arranque de cabellos en la
tricotilomanía, inquietud por la propia apariencia en el trastorno dismórfico corporal,
preocupación por las drogas en un trastorno por consumo de sustancias, preocupación por
estar padeciendo una grave enfermedad en la hipocondría, preocupación por las
necesidades o fantasías sexuales en una parafilia o sentimientos repetitivos de culpabilidad
en el trastorno depresivo mayor).

E. El trastorno no se debe a los efectos fisiológicos directos de una sustancia (p. ej.,
drogas, fármacos) o de una enfermedad médica.

Especificar si:

Con poca conciencia de enfermedad: Si, durante la mayor parte del tiempo del episodio
actual, el individuo no reconoce que las obsesiones o compulsiones son excesivas o
irracionales.

9.- Se trata de paciente femenino de 43 años de edad con diagnóstico de polimiosistis al


realizar la exploración física, la disminución de la fuerza muscular se caracteriza por:

a) Ser de predominio proximal


b) Ser de predominio distal
c) Acompañarse de dolor intenso
d) Afectar únicamente a la musculatura distal

La polimiositis (PD) y dematomiositis (DM), son enfermedades inflamatorias en las cuales el


compromiso principal es la debilidad muscular, generalmente proximal y simétrica con
atrofia subsecuente, generalmente indolora.
Su etiología y patogenia siguen siendo desconocidas, siendo considerada dentro de las
enfermedades difusas del tejido conectivo.

Numerosos estudios sugieren que una exposición previa a diversos virus como Influenzae A
y B, Coxsackie virus tipo B y Picornavirus pueden desencadenar PM-DM; además se cree
que los fenómenos inmunológicos juegan un papel preponderante, por la presencia de
autoanticuerpos, depósito de inmunoglobulinas y complemento en las paredes vasculares.

A nivel anatomo-patológico las principales alteraciones se encuentran en los músculos


esqueléticos y en los vasos sanguíneos. Siendo la vasculitis un signo común en la
dermatomiositis infantil y su severidad indicará una peor evolución. Clínicamente los
síntomas iniciales son la debilidad muscular proximal, el rash típico, frecuente el edema en
cara y a veces en miembros, fiebre moderada, posteriormente dificultad para la deglución y
debilidad en la voz; asimismo manifestaciones viscerales que no se presentan siempre, pero
son de gran importancia en cuanto al pronóstico, asimismo transtornos ventilatorios
(pulmonares): por afectación primaria (neumonía intersticial) o secundaria a la disfunción
faríngea (aspiración) o a la debilidad de los músculos respiratorios (insuficiencia
ventilatoria), o como complicación del tratamiento (infecciones oportunistas).

Wortmann RL. Idiopathic inflammatory myopathies. A. Clinical features. In: Primer on the
rheumatic diseases. New York: Springer-Arthritis Foundation; 2008. p. 363-7.

10.- Un trabajador limpia vidrios con antecedente de fractura vertebral de C6 y fractura


del extremo distal del radio derecho tras caída de un tercer piso se presenta a consulta,
refiere dolor, limitación de la movilidad y hormigueo en los dedos 2º y 3º de la mano
derecha, de predominio nocturno, que mejora durante el día. ¿Cuál de los siguientes
diagnósticos es el más probable?:

a) Síndrome del túnel del carpo.


b) Radiculalgia C5-C6.
c) Distrofia simpático refleja.
d) Artrosis postraumática del carpo.

El Síndrome del Túnel Carpiano (STC) es una patología que afecta a la mano, provocada
por una presión sobre el nervio mediano a nivel de la muñeca. Esto provoca síntomas como
adormecimiento y hormigueos en la mano (especialmente en los dedos pulgar, índice,
corazón y mitad del anular). Puede existir dolor, que puede estar limitado a la mano y
muñeca, pero que en algunas ocasiones se irradia hacia el antebrazo. El STC con frecuencia
despierta al paciente por la noche, y los síntomas pueden aparecer con actividades como
conducir un vehículo, escribir, u otros ejercicios que suponen una utilización significativa de
la mano. En el síndrome del túnel del carpo avanzado, puede producirse una pérdida de
fuerza y una disminución de la masa muscular en la base del pulgar.
CAUSA

El túnel del carpo es un canal formado por los huesos de la muñeca y un ligamento (el
ligamento transverso del carpo) situado en la cara palmar de la muñeca. Por este túnel
transcurren todos los tendones que flexionan la muñeca y los dedos, y el nervio (nervio
mediano) que recoge la sensibilidad del pulgar, índice, corazón y parte del anular y moviliza
los músculos de la base del pulgar. Algunas personas nacen con túneles estrechos y por
tanto están predispuestas a problemas de presión sobre el nervio. La utilización vigorosa de
la mano, que conduce a una tendinitis de los tendones que flexionan el pulgar y los demás
dedos, también puede conducir a un síndrome del túnel del carpo a través del
engrosamiento de las vainas tendinosas. Las vainas engrosadas "rellenan" el túnel
presionando sobre el nervio. Las personas con artritis reumatoide, hipotiroidismo, diabetes,
amiloidosis, insuficiencia renal y algunos otros problemas médicos están más predispuestas
a padecer este síndrome.

1. Arthroshi I, Gummenson C, Johonsson R, Ornstein E, Ranstam J, Rossen I.


Prevalence of carpal tunnel syndrome in a general population. JAMA. 1999;282:153-
8 Periódico El Público, 17 de octubre del 2007,pag 26.
2. Durkan, JA. The carpal compression test: an instrumental device for diagnostic
carpal tunnel syndrome. Lancet. 1990;335:393-5.
3. Marshall S, Tardif G. Injection local of steroids in the carpal syndrome. Cochrane
Data Base of Systematic Reviews. 2005. Issue 5.
4. Verdugo RJ, Salinas RS, Castillo J, Cea JG. Tratamiento quirúrgico versus
tratamiento no quirúrgico para el síndrome del túnel carpiano. Cochrane Data Base.
2005.

11.- Masculino de 50 años con dolor precordial relacionado al esfuerzo, de corta duración,
de 4 meses de evolución y con ECG en reposo normal, el siguiente estudio de elección es:

a) Prueba de esfuerzo con protocolo de Bruce


b) Ecocardiograma de reposo
c) Prueba de Talio- Dipiridamol
d) Ecocardiograma con estrés farmacológico
El propósito fundamental de la prueba es el de demostrar la existencia de isquemia
miocárdica en los subgrupos de población con mayor prevalencia de cardiopatía isquémica o
bien en los subgrupos en donde la prueba se efectúa a manera de evaluación del
tratamiento en pacientes ya conocidos con cardiopatía isquémica. Existe otro subgrupo de
población en quienes este estudio ayuda a determinar la clase funcional en la que se
encuentran e incluye a pacientes con valvulopatías o con insuficiencia cardiaca de cualquier
etiología.

Protocolos de esfuerzo.

El protocolo más empleado es el de Bruce sobre treadmill, aunque existen otros protocolos
y su elección dependerá de las condiciones del individuo.

Los protocolos discontinuos son los que alternan periodos de esfuerzo que se intercalan con
periodos de reposo de duración similar, se emplean en escasas circunstancias. Los
protocolos continuos son los que no interrumpen el esfuerzo una vez iniciado hasta
finalizada la prueba, permiten mejor adaptación física y psicológica y es posible adaptar la
intensidad de forma individualizada para que la prueba tenga una duración de 6 a 12
minutos.

Los protocolos máximos son los que se suspenden debido a la sintomatología del paciente, a
los signos registrados durante la prueba o se alcanzan valores máximos de FC y VO2. Los
protocolos submáximos son los que se suspenden cuando el sujeto alcanza un nivel
determinado de carga, habitualmente el 85% de la FC máxima teórica (que se encuentra
entorno a los 170 lpm).

En la práctica diaria, el nivel de carga (VO2) se expresa en forma de trabajo externo (MET
ó equivalentes metabólicos) que corresponden a 3,5ml/kg/min de VO2, lo que permite
comparar protocolos entre sí (cada protocolo dispone de fórmulas para realizar el cálculo
de los METS), el error que cometen en el cálculo de los METS es mayor en protocolos
discontinuos.

Emplear la FC como único criterio para determinar el esfuerzo máximo es erróneo, por lo
que deberían tenerse en cuenta otros criterios, como es la percepción subjetiva por parte
del paciente mediante la escala de Borg (tabla I). Esta dificultad en la predicción del
esfuerzo máximo es lo que limita la realización de pruebas submáximas a la determinación
de la condición física de sujetos aparentemente sanos.

Tabla I. Escala de Percepción del esfuerzo de Borg.


Escala de 15 grados Escala de 10 grados
Valor Percepción Valor Percepción
6 No se siente nada 0 Nada
7 0,5 Muy muy leve
Muy muy leve
8 1 Muy leve
9
Muy leve 2 Leve
10
11 Considerablemente leve 3 Moderada
12 4 Algo fuerte
Moderadamente dura
13 5 Fuerte o intensa
14 6
Dura Muy fuerte
15 7
16 8
Muy dura
17 9 Muy muy fuerte
18 (submáxima)
Muy muy dura 10
19
20 Esfuerzo máximo
* A la izquierda la escala original de esfuerzo percibido en 15 grados (de 6 a 20) y a
la derecha la más nueva de 10 categorías.

Tabla II. Indicaciones clásicas de la ergometría.


I. Fines diagnósticos
A. Pacientes sintomáticos.
1. Dolor torácico:
a) Típico
b) Atípico
2. Clínica de equivalentes isquémicos.
A. Pacientes asintomáticos.
1. Con alteraciones en el ECG sugestivas de isquemia.
2. Con alta probabilidad de padecer Cardiopatía Isquémica (paciente con
múltiples factores de riesgo)
3. Cuando convenga descartar con cierta seguridad CI.
4. Con sospecha de CI silente.
5. Sedentarios que inician programa de actividad física.
6. Para estudio funcional de ciertas arritmias.
II. Con fines valorativos y pronósticos.
1. Seguimiento de paciente con CI conocida.
2. Tras IAM.
3. En exámenes prelaborales o laborales.
4. De la eficacia del tratamiento:
Médico.
Cateterismo y angioplastia.
Quirúrgico.
5. Respuesta de la Tensión Arterial.
6. En valvulopatías o miocardiopatías.
7. Estudio de arritmias y trastornos de la conducción aurículo-ventricular.
8. En cardiopatías congénitas.

Bibliografía

1. Gibbons RJ (Edit.). ACC/AHA 2002 Guideline Update of Exercise Testing. 2002


American College of Cardiology Foundation and American Heart Association
ACC/AHA; 2002 [Acceso 1-4-06]. Disponible en:
2. Guidelines for cardiac exercise testing. ESC Working Group on Exercise
Physiology, Physiopathology and Electrocardiography Eur Heart J 1993; 14: 969-988.
3. Fernando Arós Aros F, Boraita A, Alegria E, Alonso AM, Bardaji A, Lamiel R el al.
Guías de práctica clínica de la Sociedad Española de Cardiología en pruebas de
esfuerzo. Rev Esp Cardiol 2000; 53 (8): 1063-94.
4. Chaitman B. Las pruebas de esfuerzo. En: Braunwald E, editor. Tratado de
Cardiología. Medicina Cardiovascular. 4ª ed. Madrid Mc-Graw-Hill-Interamericana de
España; 1993. p. 177-197.
5. Schlant RC, Friesinger GC 2nd, Leonard JJ. Clinical competence in exercise testing:
a statement for physicians from the ACP/ACC/AHA Task Force on Clinical Privileges
in Cardiology. J Am Coll Cardiol 1990; 16: 1061-5.

6. Reyes Lopez de los M, Iñiguez Romo A, Goicolea de Oro A, Funes Lopez B, Castro
Beiras A. El consentimiento informado en cardiología. Rev Esp Cardiol 1998; 51: 782-
796.
7. Fletcher GF, Flipse T, Malouf J, Kligfield P. Current status of ECG stress testing.
Curr Probl Cardiol. 1998 Jul; 23(7): 353-423.
8. Alegría Ezquerra E, Alijarde Guimerá M, Cordo Mollar JC, Chorro Gascó FJ,
Pajarón López A. Utilidad de la prueba de esfuerzo y de otros métodos basados en el
electrocardiograma en la cardiopatía isquémica crónica. Rev Esp Cardiol 1997; 50: 6-14
9. Wasserman K, Hansen JE, Sue DY, Whipp BJ, Casaburi R. Principles of exercise
testing and interpretation . 2ª ed. Philadelphia: Lea & Febiger; 1994. p. 95-111.
10. American college of Sports Medicine. Guideliness for exercise testing and
prescription. 5ª ed. Baltimore: Williams & Wilkins; 1995.
11. Borg GA. Psychophysical bases of perceived exertion. Med Sci Sports Exerc 1982;
14: 377-381
12. Froelicher VF, Umann TM. Exercise testing: clinical applications. En: Pollock ML,
Schmidt DH, editors. Heart disease and rehabilitation . 3ª ed. Champaign, IL: Human
Kinetics, 1995; p.57-79.
13. Myers J, Froelicher VF. Exercise testing. Procedures and implementation. Cardiol
Clin. 1993; 11(2): 199-213.
14. Weiner DA, McCabe C, Hueter DC, Ryan TJ, Hood WB Jr. The predictive value of
anginal chest pain as an indicator of coronary disease during exercise testing. Am
Heart J 1978; 96: 458-462.

12.- Una mujer de 40 años de edad en buena salud general experimenta dolor retroesternal
súbito con fiebre y falta de aire. Es fumadora y no toma medicamentos excepto
anticonceptivos orales. En la exploración física se encuentran taquipnea y temperatura de
38°C. Los datos de auscultación, percusión y radiográficos del tórax son normales. ¿Cuál de
los siguientes diagnósticos es más probable?

a) Traqueobronquitis
b) Neumonía atípica
c) Embolia pulmonar
d) Neumonía bacteriana

Allen R. M. MMS Medicina Interna. 5ª. Edición. National Medical Series. Mc. Graw Hill.
2006. (capítulo 2 VIII E 1, 2 a; capítulo 8 V C 3, 4). El diagnóstico más probable es embolia
pulmonar. El inicio agudo descarta neumonía atípica y hace poco probable el cáncer
pulmonar. Sin pruebas de tos productiva es poco probable que haya traqueobronquitis,
trastorno también subagudo. La neumonía bacteriana es muy improbable junto con la
radiografía de tórax. El tabaquismo y el uso de anticonceptivos orales predisponen a
trombosis venosa profunda y embolias pulmonares.

13.- Durante su consulta en la unidad de Medicina Familiar recibe a un paciente de 60 años


de edad que asegura ser hipertenso de más de cinco años de evolución, sin agudizaciones
además de ser portador de una fibrilación auricular, motivo por los cuales recibe
propranolol en dosis de 100 MG cada mañana. Al revisar su electrocardiograma usted
espera encontrar:

a) Ausencia de onda P, complejo QRS normal e intervalos R-R diferentes.


b) Ausencia de onda P, complejo QRS ensanchado, diferencia constante de los intervalos R-
R.
c) Presencia de onda P, complejo QRS ensanchado y eje rotado a la derecha.
d) Ritmo nodal con disociación A-V

Guadalajara J. Cardiología. Sexta Edición 151 – 152 La fibrilación auricular es la


arritmia cardiaca más frecuente. La despolarización caótica y desordenada de las aurículas
por múltiples ondas en simultánea, trae como consecuencia que se pierda la función
mecánica de la contracción auricular. Estos dipolos de activación múltiple y desordenados
alcanzan al nodo A-V y penetran en él, algunos pasan hacia el Haz de His mientras que otros
no lo alcanzan, debido a la penetración parcial del nodo por conducción decreciente
completa, esto se conoce como, conducción oculta; así, la rápida penetración de los
estímulos auriculares favorece la aparición de conducción oculta, la cual afecta en forma
impredecible el periodo refractario del nodo. El trazo electrocardiográfico típico es la
ausencia de onda P, complejo QRS normal e intervalos R-R diferentes.

14.- Femenino de 59 años, refiere vivir sola, como antecedentes refiere HTA y artritis
reumatoide, con buen control farmacológico. Presenta alucinaciones auditivas y
cenestésicas, con ideas delirantes de perjuicio con los vecinos y de contenido místico-
religioso de 4 meses de evolución. ¿Cuál sería el diagnóstico más probable?

a) Síndrome confusional agudo.


b) Depresión delirante.
c) Esquizofrenia de inicio tardío.
d) Psicosis psicógena.

LOS ÚLTIMOS AÑOS se ha incrementado el interés por el estudio de los estados


psicóticos de inicio tardío y su relación con el deterioro cognoscitivo y los procesos
demenciales ya instalados.
Las dificultades en la organización de un cuadro clínico específico de psicosis tardía que
responda a un estado nosológico, con características clínicas propias y a una etiología clara,
que le de cuerpo como una entidad, ha generado confusión entre los diversos investigadores
clínicos y epidemiológicos.

Esquizofrenia tardía: se caracteriza por la presencia de delirios, con alucinaciones y con


desorganización de la personalidad, alteración afectiva y conductas bizarras. Hay un menor
grado o ausencia de deterioro cognoscitivo, con una edad de inicio entre los 45 y 60 años.
No hay antecedentes de trastornos psicóticos, afectivos o demenciales.
. Esquizofrenia de inicio muy tardío en el anciano: son cuadros clínicos de tipo psicótico en
ancianos-ancianos (mayores de 60 años) que presentan sintomatología delirante poco
estructurada, con alucinaciones, con ausencia de deterioro cognoscitivo generalizado y
progresivo, sin compromiso afectivo significativo.
Existen otros estados psicóticos no esquizofrénicos de aparición en edades avanzadas,
como: el trastorno delirante de ideas persistentes, la paranoia, el trastorno delirante de
perjuicio, los delirios hipocondríacos de enfermedad y los delirios erotomaníacos.
Cuadro clínico
Diversos autores han intentado caracterizar un cuadro clínico de las psicosis de inicio
tardío en donde predominan los delirios, las alucinaciones, interpretaciones paranoides
(errores de percepción) y otros síntomas Scheneiderianos. En ancianos con psicosis tardías
el cuadro clínico puede estar conformado por:

Fenómenos delirantes, especialmente paranoides de tipo persecutorio o de referencia.


Pueden presentar un tipo especial de delirios denominados de tabique, los cuales se
caracterizan por la creencia de que detrás de las paredes de su habitación operan personas
con el propósito de hacerles daño o conspirar contra sus intereses e interferir contra su
vida.
. Fenómeno del eco del pensamiento: manifiestan que sus pensamientos son leídos o
robados.
. Presencia de alucinaciones especialmente auditivas, con contenidos de referencia,
persecutorios o eróticos. También pueden presentar alucinaciones visuales, táctiles y
olfativas.

. Ausencia de trastornos cognoscitivos generalizados y progresivos.


. Los trastornos afectivos no son significativos, de tal modo que se sospeche de cuadros de
trastorno esquizoafectivo; pero estos ancianos psicóticos pueden presentar cuadros
depresivos moderados, sin que lleguen a constituirse en una depresión mayor de tipo
psicótico.
. Edad de inicio por encima de 60 años.
. Co-morbilidad frecuente con personalidad premórbida esquizoide o paranoide.
. Alteraciones del comportamiento se presentan más en el período de estado de la crisis
psicótica, pero en general hay menos desorganización y conductas bizarras que en aquellos
con cuadros crónicos de esquizofrenia.
. Falta de insight o introspección.
. Signos neurológicos blandos como reflejo glabelar, temblor, movimientos anormales,
discinesia tardía, hipoacusia, rigidez.

ALARCÓN R.
Alteraciones Psiquiátricas en la Demencias, en Arango LJC, Fernández GS y Ardila A, Las
Demencias: Aspectos Clínicos, Neuropsicológicos y Tratamiento, Ed. Manual Moderno,
México,México, 2003.
ALMEIDA OP, HOWARD R, LEVY R, DAVID AS. Psychotic states arising in late life (late
paraphrenia). The role of risk factors. Br J Psychiatry, 1995; 166: 215-228, AMORES GF.
Funciones Cognitivas Superiores, Taller Master de Psicogeriatría, Universidad Autónoma de
Barcelona, Barcelona, Febrero, 2004, CASTLE D, MURRAY RM.
The epidemiology of late onset schizophrenia. Schizophr Bull, 1993; 19: 691-700
CHRISTENSON R, BLAZER DG.
Epidemiology of persecutory ideation in an elderly population in the community. Am J
Psychiatry, 984; 141: 1088-1091.
15.- Mujer de 42 años. Acude a consulta por cefalea y acúfenos. Tiene antecedente de DM
en padre. EF: peso 78 kg, estatura 1.62, perímetro abdominal 108 cm, TA: 140/100.
Laboratorio: glucosa de ayuno 116, prueba de tolerancia a la glucosa, resultado a las 2 horas
de 189.
Los diagnósticos que se establecen en la paciente son:

a) Obesidad y diabetes mellitus


b) Síndrome metabólico con intolerancia a la glucosa
c) Obesidad e intolerancia a la glucosa
d) Síndrome metabólico y diabetes mellitus

GLUCOSA:
<100 mg/dl = normal
≥100 y < 126 = glucosa de ayuno
alterada (GAA)
≥126 mg/dl = diabetes mellitas

Glucosa a las 2 horas postcarga:


≤ 140 mg/dl = normal
140 a 199 mg/dl = intolerancia a la glucosa
≥ 200 mg/dl = diabetes mellitus

El síndrome metabólico es la agrupación de diferentes factores de riesgo asociados con el


síndrome de resistencia a la insulina. En la literatura existen variaciones importantes en la
prevalencia de este síndrome, dependiendo de los criterios o definición que se empleen
para su diagnóstico. En la actualidad hay más de cinco definiciones del síndrome
metabólico; la descrita en 2001 y su actualización en 2005 por el Programa Nacional de
Educación para el Colesterol, Panel de Tratamiento del Adulto III (NCEP-ATPIII), es la
más empleada en la práctica clínica. Esta definición establece que se requiere la
presencia de tres o más de los siguientes cinco criterios: glucosa en ayuno ≥ 100
mg/dl, triglicéridos ≥ 150 mg/dl, colesterol HDL bajo (< 40 mg/dl en el hombre o <
50 mg/dl en la mujer), tensión arterial ≥ 130/85 mm Hg o en tratamiento para la
hipertensión y obesidad abdominal detectada mediante medición del perímetro
abdominal (> 102 cm en hombres y > 88 cm en mujeres).1 La obesidad ha alcanzado
proporciones epidémicas en los países occidentales y, por lo tanto, es un importante
problema de salud. De hecho, México ocupa el segundo lugar a nivel mundial en
sobrepeso y obesidad. La resistencia a la insulina que se promueve por la obesidad
abdominal o fenotipo de obesidad androide se considera la responsable de algunos
factores de riesgo que se agrupan en el síndrome metabólico.2
Estudios recientes indican que la prevalencia del síndrome metabólico en la población
general en México es de aproximadamente 26.6 %, similar a los resultados informados por
la Tercera Encuesta de Evaluación Nacional en Salud y Nutrición en población de Estados
Unidos, que fue de 26.7 %.3 Sin embargo, la prevalencia aumenta conforme avanza la edad,
llegando a ser hasta de 44 % en individuos mayores de 50 años.4
El síndrome metabólico no sólo incluye algunos factores de riesgo cardiovascular
tradicionales, sino otros componentes que representan aspectos involucrados en el
desarrollo y progresión de enfermedades cardiovasculares, como la disfunción endotelial,
marcadores de inflamación y alteraciones en la coagulación. La sinergia de la combinación
de estos factores se considera causante del aumento en el riesgo de morbilidad y
mortalidad asociada con enfermedades cardiovasculares en este grupo de pacientes. Los
pacientes que completan criterios para síndrome metabólico tienen tres veces más riesgo
de padecer enfermedad arterial coronaria y cerebrovascular.5 Más aún, en pacientes con
enfermedad vascular sintomática se encontró que el síndrome metabólico se asociaba con la
extensión del daño vascular.6,7
El síndrome metabólico es uno de los principales problemas de salud pública del siglo XXI.
El diagnóstico es sencillo, principalmente con base en datos clínicos y bioquímicos
sistemáticos, los cuales están al alcance de cualquier sistema de salud.

Síndrome metabólico, impacto clínico y angiográfico en pacientes con síndrome coronario


agudo
Volumen 78, No. 2, Marzo-Abril 2010
Alejandra Madrid-Miller,* Antonio Alcaraz-Ruiz,* Gabriela Borrayo-Sánchez,**
Eduardo Almeida-Gutiérrez,* Rosa María Vargas-Guzmán,* Ricardo Jáuregui-Aguilar***

16. - Male, 25, is brought to ER after a solvent factory accident 30 minutes ago, he shows
burn injuries on both legs, genitals, anterior and posterior surface of lower trunk. Select
the estimation of body surface area in burns according to Rule of nines.

a) 55%
b) 60%
c) 25%
d) 46%
Esta ilustración muestra la Regla de los Nueve para áreas corporales.

Divide el cuerpo en secciones que representan el nueve por ciento del total del área de la
superficie del cuerpo (TBSA). Puede ser utilizado conjuntamente con los pacientes adultos
quemados para determinar el TBSA quemado. Las secciones incluyen la cabeza y cuello,
brazos, torso (pecho, abdomen anterior, región dorsal y región lumbar), perineo y piernas.

A mayor extensión, mayor gravedad. Las quemaduras en cara, genitales, manos o pies por si
mismas ya son graves. En cualquier caso, sea cual sea el área quemada, si supera el 15%
también se considerará grave.

Se estima que el reparto en porcentaje sería el siguiente:

Cabeza y cuello: 9%
Tronco anterior: 18% (Tórax 9 y abdomen 9)
Tronco posterior: 18% (parte alta espalda 9 y baja 9)
miembros inferiores (cada uno): 18%
miembros superiores (cada uno): 9%
periné: 1%

Libro: Urgencias Medicas De Flint de Cain Editorial: MC GRAW-HILL EDUCATION ...


17.- Mujer de 26 años, con tumor anexial de 6 cm líquido, dolor abdominal, fiebre,
leucorrea, con historia de cervicovaginitis de repetición y dispareunia crónica, última
menstruación hace una semana. El diagnóstico más probable será:

a) Cistadenoma
b) Embarazo ectópico
c) Quiste de ovario
d) Enfermedad pélvica inflamatoria

La EIP puede cursar con los siguientes síntomas:

• Dolor abdominal bajo (incluyendo dolor anexial, dispareunia). Es el síntoma más


frecuente (95%)
• Aumento del flujo vaginal, flujo de características anormales (74%)
• Sangrado anormal (intermestrual, poscoital) (45%)
• Síntomas urinarios (35%)
• Vómitos (14%)
• Es posible la ausencia de síntomas

Y en ella podemos encontrar estos signos:

• Dolor a la movilización del cuello, dolor anexial en la exploración vaginal bimanual


(99%)
• En el examen con espéculo observamos cervicitis y descarga endocervical purulenta
(74%)
• Fiebre (> 38º C) (menos del 47%).
• Masa pélvica: sugiere abceso tuboovárico (ATO)
• Peritonitis

CRITERIOS CLINICOS PARA EL DIAGNOSTICO DE SALPINGITIS

a. Dolor abdominal con o sin rebote.


b. Sensibilidad a la movilización del cérvix.
c. Sensibilidad anexial.

Los tres criterios anteriores son necesarios para establecer el diagnóstico, con uno o más
de los siguientes:

a. Extendido de Gram de endocérvix positivo, para diplococos gram negativos


intracelulares
b. Temperatura mayor de 38°C
c. Leucocitosis (mayor de 10.000 por c.c.)
d. Material purulento (positivo para leucocitos) en la cavidad peritoneal
obtenido por culdocentesis o laparoscopia.

Establecido el diagnóstico clínico de EPI, se debe hacer la definición del estado clínico y
anatómico de la patología pélvica:

a) No complicada (limitada a trompas u ovarios)

1) Sin peritonitis pélvica

2) Con peritonitis pélvica

b) Complicada (masa inflamatoria o absceso que compromete trompa (s) u ovario (s)

1) Sin peritonitis pélvica

2) Con peritonitis pélvica

Referencia:

Beigi RH, Wiesenfeld HC. Pelvic inflammatory disease: new diagnostic criteria and
treatment. Obstet Gynecol Clin Norh Am. 2003; 30 (4): 777 – 93
Center for Disease Control. Guidelines for treatment of sexually transmited diseases.
MMWR Recomm Rep. 2002 May 10;51(RR-6):1-78
Center for Disease Control. Guidelines for prevention and management (MMWR. 40: 1 - 25
1991) Pelvic inflammatory disease: guidelines for prevention and management.
MMWR Recomm Rep. 1991 Apr 26;40(RR-5):1-25.
Hager WD, Eschenbach DA, Spence MR, Sweet RL. Criteria for diagnosis and grading of
salpingitis. Obstet Gynecol. 1983 Jan;61(1):113-4.
Prodigy Guidance. Pelvic inflammatory disease. [Internet]. UK : NHS, Department of
Health; 2003. [Acceso 18 de Junio de 2005]. Disponible en:

Ross J. Pelvic inflammatory disease. Clin Evid. 2004 Dec;(12):2259-65.


Royal College of Obstetricians and Gynaecologists. Pelvic Inflammatory Disease. Guideline
nº 32. [Internet]. RCOG; Mayo 2003. [Acceso 18 de Junio de 2005].

18.- En el cunero, se observa que un neonato presenta asimetría de pliegues glúteos. A la EF


la cadera, una de ellas puede ser fácilmente luxada posteriormente con un “click” y
regresada a su posición normal con un sonido parecido. La familia se encuentra preocupada
porque el primer hijo tuvo el mismo problema. ¿Cuál es el diagnóstico más probable?
a) Displasia de cadera
b) Enfermedad de Legg-Perthes
c) Artritis séptica
d) Parto traumático

La falta de relación normal en las estructuras que forman una articulación se conoce como
luxación. En el caso de la cadera, la cabeza del fémur no encaja debidamente en su cavidad
(acetábulo).
El desarrollo del acetábulo será normal siempre que la cabeza del fémur permanezca en
posición correcta y el esfuerzo muscular sea adecuado.
Siempre que esto no se cumpla, se producirá una alteración entre el cótilo y la cabeza
femoral, dando lugar a una alteración con el resultado de luxación de la cadera.

DIAGNÓSTICO: EXPLORACIÓN FÍSICA


a) Maniobras de Ortolani y Barlow - click audible-.
Ortolani: el click se produce cuando la fóvea de la cabeza del fémur encuentra la cresta
cotiloidea caminando sobre ella. La limitación a la abducción desaparece, esto es signo de
reducción.
Barlow: se realiza la aducción de la cadera, que tras una ligera presión longitudinal sobre el
fémur, produce una luxación.
La prueba de Ortolani traduce una luxación y la de Barlow una cadera luxable.
b) Asimetría de pliegues -no valorable en luxaciones bilaterales.
c) Limitación a la abducción.
d) Dismetría.

Referencia:

• García Portabella, M.
Luxación congénita de cadera antes de los tres meses de edad. 2001.
• Garcia-Siso Pardo, J. M.
Displasia del desarrollo de la cadera.
(Parte I). Peditría Rural y Extrahospitalaria.
Vol. 32. Nº 304 Págs. 481-491. 2002.
• Graf, R.

Classification of hip joint dysplasia by means of sonography.


Arch Orthop Trauma Surg 102:248-255, 1984.
• Edeiken, J.
Luxación congénita de cadera.
Diagnóstico Radiológico de las Enfermedades de los Huesos.
Ed. Médica Panamericana. Págs. 388-392. 1977.
19.- Femenino de 18 años originario de Tabasco, quien inicio con fiebre elevada, cefaleas,
mialgias generalizadas y artralgias, se diagnostica dengue clásico, el tratamiento de
elección en esta patología es:

a) Aciclovir
b) Cristaloides y plasma
c) Antivirales e hidratación con cristaloides.
d) Sintomático

DENGUE CLÁSICO

Las primeras manifestaciones clínicas son de inicio abrupto tras 2-7 días de incubación.
Se caracterizan por fiebre elevada (39-40ºC), cefaleas, mialgias intensas generalizadas y
artralgias con dolor cervical y lumbar, anorexia, gran astenia, náuseas, vómitos y dolor
abdominal. Los síntomas respiratorios (tos, rinitis, faringitis) son frecuentes. Se puede
presentar una erupción cutánea máculo-papular, que aparece al comienzo de la fiebre o
coincide con un segundo pico febril a los 3-5 días. Pueden observarse poliadenopatías,
granulocitopenia, linfocitosis relativa y trombopenia.

Algunos de los aspectos clínicos dependen fundamentalmente de la edad del paciente.


El dolor abdominal generalizado ha sido observado más frecuentemente en niños.
En adultos, al final del período febril se pueden presentar manifestaciones hemorrágicas
de poca entidad, como epístaxis, petequias, gingivorragias, y en casos más raros
hematemesis, melenas o hematurias. Si bien el Dengue Clásico es usualmente benigno y
autolimitado, se asocia con gran debilidad física y algunas veces con una convalescencia
prolongada, pudiendo estar presentes las manifestaciones hemorrágicas, que no son
exclusivas de la entidad clínica llamada Fiebre Hemorrágica de Dengue.

La enfermedad cursa con viremia precoz y breve (desde un día antes de los síntomas hasta
3-5 días después aproximadamente), lesiones de engrosamiento endotelial, edema e
infiltración mononuclear en torno a los pequeños vasos.

Tratamiento

El tratamiento en el dengue clásico es sintomático, no deben usarse salicílicos.

La prevención y vigilancia
Hidratación oral
Tratamiento del Shock EN CASO NECESARIO.
Rehidratación parenteral si se presenta: intolerancia a la vía oral, deshidratación moderada
o grave, hematocrito en aumento o derrames cavitarios.
Los corticoides no demostraron ser efectivos
Antibióticos no están indicados

20.- Un niño de 12 años, varón, acude al centro de urgencias 30 mins. Después de haber
ingerido comprimidos de sulfato ferroso de 325 mg. La madre indica que faltan 20
comprimidos del frasco. Cada comprimido tiene 65 mg de hierro elemental. El niño ha
vomitado una vez y su comportamiento parece totalmente normal. Los hallazgos de la
exploración física no arrojan nada importante. Pesa 13 kg. ¿Cuál de los siguientes
enunciados sobre el estado del niño es el correcto?

a) La dosis de hierro no debería causar secuelas clínicas.


b) Ese episodio único de vómito probablemente no esta relacionado con la ingestión.
c) Aunque ha vomitado una vez debería administrarse ipecacuana.
d) Los catárticos son ineficaces en intoxicaciones.

En situaciones en las que no hay testigos de la ingestión debe presuponerse inicialmente el


“peor escenario posible” para estimar la toxicidad potencial. La dosis máxima de hierro
ingerida por este niño es de 65 mg de hierro elemental x 20 píldoras / 13 kg = 100 mg /kg
que es una dosis potencialmente grave. Entre los síntomas causados por la ingestión grave
de hierro están los vómitos por tanto en este caso el vómito debe considerarse relacionado
con la ingestión. Ello implica un riesgo potencial de secuelas graves. En conjunto la ingestión
debe valorarse como clínicamente importante.
Como un solo episodio de vómitos no vacía el estómago de manera suficiente debe
administrarse ipecacuana o alternativamente proceder a un lavado gástrico. El carbón
activado no adsorbe el hierro y no es necesario. Los catárticos sirven de ayuda una vez que
e ha vaciado el estómago. Los niveles de hierro sérico y de capacidad de enlace del hierro
deben estimarse según las circunstancias. También es útil obtener una radiografía de
abdomen por cuanto las píldoras son radioopacas y puede deducirse así algún indicio de su
permanencia en el tracto GI después del tratamiento inicial.

1.- Montoya-Cabrera MA. Intoxicaciones y envenenamientos en niños. México,


Intersistemas , 2000.
2.-Montoya CMA. Toxicología clínica. 2ª. Ed, México, Méndez Editores, 1997
21.- Mujer de 37 años. Acude a consulta por presentar cefalea, cansancio e irregularidades
menstruales con ritmo de 36 a 50 x 2-3 días. No se ha podido embarazar después de 18
meses de actividad sexual regular. No tiene antecedentes importantes. EF: Campos visuales
normales, tiroides aumentada de tamaño una vez y aumentada de consistencia, no tiene
galactorrea. Resto normal. Laboratorio: química sanguínea, Bh y electrolitos normales.
Prolactina 47 ng/dL (< 25), perfil tiroideo: TSH 18 mUI/ml, T4t: 50 nmol/L (57.9 a 154.4),
T4L: 7.7 pmol/L (9 a 24), T3T: 1.06 nmol/L (1.2 a 2.9), T3L: 1.96 pmol/L (3 a 6.31)
El diagnóstico es:

a) Hiperprolactinemia
b) Hipertiroidismo
c) Síndrome de ovarios poliquísticos
d) Hipotiroidismo primario

DIAGNOSTICO

Inicialmente el hipotiroidismo se diagnosticaba mediante la cuantificación por técnicas de


Radio Inmuno Análisis (RIA) de las hormonas circulantes triyodotironina y tiroxina; el
proceso era lento y sometido a muchos factores de error que hacían su sensibilidad y
especificidad poco confiables. Posteriormente, se desarrollaron técnicas para la medición
de la TSH hipofisiaria igualmente mediante el RIA lo que mejoró en forma importante la
sensibilidad para el diagnóstico de esta enfermedad; sin embargo, los niveles de detección
de la prueba se encontraban en el orden de 1 µIU/ml lo que hacía que la prueba no fuera
sensible para valores menores de 1 µIU/ml. Debido a esto se crearon técnicas de segunda
generación mediante la cuantificación de TSH por anticuerpos monoclonales y RIA, el
IRMA (Immuno Radiometric with Monoclonal Antibodies) que permitió detectar valores de
TSH en rangos de 0.1 µIU/ml; posibilitando desde entonces diagnosticar pacientes con
hipertiroidismo primario; pero con la limitante de que para esta técnica era imposible
detectar valores de TSH menores de 0.1 µIU/ml por lo que se creo la medición de TSH
mediante quimioluminiscencia o métodos enzimáticos, es decir las técnicas de tercera
generación, las cuales pueden detectar valores de TSH de 0.01 µIU/ml; con lo que se logra
el espectro ideal para una prueba de laboratorio que tiene la capacidad de diagnosticar
tanto la hipofunción como la hiperfunción(20).

Además el avance no sólo fue en la medición de TSH sino también en las hormonas tiroideas
que han evolucionado simultáneamente con la TSH y ya se miden incluso las fracciones
libres de hormonas y las fracciones totales, lo que ha facilitado el manejo de estos
pacientes. Gracias a esta evolución en técnicas de laboratorio, el diagnóstico de
hipotiroidismo primario es bastante sencillo. Niveles de TSH superiores al valor máximo de
la técnica serían diagnósticos de la disfunción; pero no es tan fácil. Cuando tenemos un
paciente con toda la sintomatología del hipotiroidismo y la TSH se encuentra elevada el
diagnóstico es obvio; pero podemos tener pacientes con síntomas muy inespecíficos como
depresión y con examen físico normal a quienes se les encuentran valores de TSH por
encima del límite superior y con hormonas tiroideas normales. Se trata de un hipotiroidismo
o es un valor ligeramente elevado ocasional de una persona sana(21). Igualmente tenemos
otra circunstancia que ha sido descrita con mayor frecuencia: pacientes con valores de
TSH en el límite superior normal y con dislipidemia a quienes se les da tratamiento con
hormonas tiroideas y su dislipidemia se corrige manteniendo valores de TSH en rangos
normales. Todas las circunstancias anteriores han hecho que aparezca en el hipotiroidismo
primario la expresión de hipotiroidismo subclinico, que ha sido objeto de reuniones y
congresos dedicados exclusivamente a este tema. La sociedad Europea de Tiroides hace
algunas recomendaciones para el manejo de esta situación que se consideran útiles como
guía (Tabla ).

Tabla. Enfoque del paciente con disfunción tiroidea de acuerdo a los niveles de hormona
estimulante de la tiroides (TSH).

Si TSH
0.4 a 2.0 Si TSH > 5.0
Si TSH < 0.4m U/L mU/L Si TSH 2.01 a 5.0mU/L mU/L
Normal,
Medir T3 y T4 totales o Repetir Dar tratamiento
libres para diagnostico cada cinco Medir T4 libre y anticuerpos para
de hipertiroidismo. años antitiroideos hipotiroidismo
1. Si AAT (-) y T4 libre es
normal repetir screening cada
año. Si TSH es > 4.0mU/l en dos
ocasiones dar tratamiento
2. Si AAT (+) y/o T4 libre esta
baja o normal baja tratar si
TSH es mayor de 3.0 mU/l y
observar a los otros
Tomado de Koutras DA. Subclinical hypothyroidism. En G. Hennemann, E.P. Krenning,
Thyroid International Merck KGaA, Darmstadt 1999 (3), 6-9
22.- Hombre de 54 años, acude a consulta por descontrol glucémico. Tiene antecedente de
DM tipo 2 de 13 años de evolución controlada con diferentes hipoglucemiantes orales.
Desde hace un mes está bajando de peso y tiene polidipsia y poliuria a pesar de tomar su
tratamiento con metformín 850 mg 3 veces al día y glibenclamida tab 5 mg, 4 tabletas
diarias, además de la dieta. EF: peso 68 kg, estatura 1.70, TA 140/80, FC 96x’. Glucosa:
289 mg, hemoglobina glucosilada de 11%. La conducta terapéutica más apropiada es:

a) Aumentar dosis de glibenclamida


b) Iniciar insulina de acción intermedia o prolongada
c) Iniciar insulina rápida por requerimientos
d) Añadir un tercer hipoglucemiante

SELECCIÓN DEL TRATAMIENTO

DIETA + EJERCICIO

Metas no Metas sí

obeso delgado continuar

BIGUANIDA SULFONIUREA Metas sí

Metas no

SELECCIÓN DEL TRATAMIENTO


MEZCLAS SU + BG

Metas no
Metas sí Continuar

TX COMBINADO
HO día
Metas no
Insulina noche

INSULINA

Medical Management of Hyperglycemia in Type 2 Diabetes: A Consensus Algorithm for


the Initiation and Adjustment of Therapy.
Dieta, ejercicio, educación y automonitoreo

HbA1c > 9%

2 hipoglucemiantes Insulina basal


Biguanida + secretagogo o preprandial o
Biguanida + Tiazolidinediona ambas
HO + insulina

Cheng YY A. CMAJ 2005; 172(2):213-26.

23.- Una mujer de 43 años inicia recientemente con fatiga, somnolencia, piel seca,
estreñimiento y aumento de peso de 5 kg. Su tiroides está firme y tiene el doble del
tamaño normal. ¿Cuál de las siguientes pruebas de laboratorio confirma el presunto
diagnóstico de hipotiroidismo?

a) Tiroxina sérica (T4)


b) Triyodotironina sérica (T3)
c) Captación de resina T3
d) Hormona estimulante de tiroides (TSH) en suero

Hipotiroidismo
La instauración es habitualmente lenta y progresiva. Los síntomas se relacionan con una
disminución en la actividad funcional de todos los sistemas del organismo. Los más clásicos
son cansancio, intolerancia al frío (carácter muy friolero), apatía e indiferencia, depresión,
disminución de memoria y de la capacidad de concentración mental, piel seca, cabello seco y
quebradizo, fragilidad de uñas, palidez de piel, aumento de peso, estreñimiento pertinaz y
somnolencia excesiva. En situaciones extremas puede evolucionar hacia la insuficiencia
cardiaca, la hinchazón generalizada (mixedema), insuficiencia respiratoria y abocar al coma
mixedematoso con pérdida de conocimiento que conlleva un alto grado de mortalidad.

Al igual que el resto de enfermedades del tiroides, el hipotiroidismo es más frecuente en


el sexo femenino. Es a partir de los 40-50 años cuando las mujeres tienden a desarrollar
con más frecuencia hipotiroidismo de causa autoinmune (tiroiditis de Hashimoto). El
periodo postparto es igualmente propenso a la aparición de este problema. La cirugía de
tiroides y la aplicación de yodo radioactivo representan situaciones de riesgo para el
desarrollo de hipotiroidismo, lo que obliga a controlar evolutivamente la función tiroidea en
estos casos.Los recién nacidos de madres hipertiroideas, hayan recibido o no tratamiento

antitiroideo durante la gestación, deben ser evaluados en este sentido. Las personas en las
que se detectan anticuerpos antitiroideos (antimicrosomales, antitiroglobulina) tienden a
desarrollar con el tiempo alteraciones de la función tiroidea, por lo que deben ser evaluados
crónicamente de forma periódica.

La determinación de TSH es el parámetro más sensible para el diagnóstico del


hipotiroidismo. Su elevación es indicativa de que la función del tiroides es insuficiente.
Este fenómeno se produce antes de que comiencen a descender en la sangre las
concentraciones de hormonas tiroideas. Generalmente, en el hipotiroidismo establecido,
además de la elevación de TSH, se produce un descenso de T4. El nivel de T3 con
frecuencia se encuentra dentro de la normalidad. Así pues, cuando aparecen síntomas
sugestivos, el médico solicitará una determinación de TSH que es el mejor método para
descartar que exista hipotiroidismo. Puede acompañarse de una determinación de T4 y de
anticuerpos antitiroideos si se desea conocer si la causa se debe a fenómenos de
autoinmunidad. En los casos de hipotiroidismo secundario debido a disminución de la
secreción de TSH por parte de la hipófisis, el diagnóstico se basa en confirmar
concentraciones disminuidas de T4 y TSH en la sangre. Cuando la elevación de TSH se
acompaña de niveles normales de T4 la condición es conocida con el nombre de
hipotiroidismo subclínico. Si existe bocio puede ser conveniente realizar una ecografía
tiroidea. Cuando existe sospecha de alteraciones en el desarrollo de la glándula o de
deficiencia enzimática, puede ser útil obtener una gammagrafía tiroidea. Si se confirma un
diagnóstico de hipotiroidismo de causa autoinmune, es habitual evaluar la asociación de
alteraciones en otras glándulas como las suprarrenales, paratiroides o gónadas.

REFERENCIAS:

(1.) Anderson R, Harnes J. 1975. Thyroid hormones secretion rates in growing and mature
goats. J Anim Sci 40: 11301135.

(2.) Anke M, Henning A, Grun M, Partschefeld M, Groppel B. 1977. Der einluss des mangan,
zink, kupfer, jod, selen, molybdan und nickelmangels aauf die fortpflanzuggsleistung des
wiederkauers. Mathem Natur Reihe (Leipzig) 26: 283-292.

(3.) Balbuena O. 2003. Nutrición Mineral del Ganado. Sitio Argentino de Producción Animal:
1-5, www.produccionanimal. com.ar.

(4.) Beckett GJ, Beddows SE, Morrice PC, Nicol F, Arthur JR. 1987. Inhibition of hepatic
deiodination of thyroxine is caused by selenium deficiency in rats. Biochem J 248: 443-
447.
(5.) Brem JJ, Pochon DO, Roux JP, Trulls H. 1998. Exploración diagnóstica de la función
tiroidea en ovinos. Rev Vet 8/9: 23-26.

(6.) Castillo V. 2001. Cambios de la función tiroidea en cachorros alimentados con dietas
comerciales con alto contenido de yodo. On line: http://www.idealibrary.com.

24.- A través de un estudio de casos y controles se quiere conocer si existe asociación


entre tabaquismo y cáncer de lengua. La variable tabaquismo se medirá como Leve (uno a
10 cigarrillo en 24 hrs), moderado (11 a 20 cigarrillos en 24hrs) severo (más de 21
cigarrillos en 24 hrs). Como podemos clasificar esta variable:

a) Cuantitativa continua
b) Cuantitativa discreta
c) Cualitativa ordinal
d) Cuantitativa discontinua

En las variables de tipo ordinal las observaciones se clasifican y ordenan por categorías
según el grado en que los objetos o eventos poseen una determinada característica. Por
ejemplo, se puede clasificar a las personas con respecto al grado de una enfermedad en
leve, moderado o severo.

Moreno A. Principales medidas en epidemiología. Rev Salud Pública Mex, 2000;42(4): 338

25.- Femenino de 20 años que acude a consulta externa, refiere que desde hace mas de un
año presenta astenia, cansancio, pérdida de apetito y dificultades para concentrarse en los
estudios. Al interrogatorio refiere que a perdido interés en los estudios, frecuenta menos
a sus amigos, con pesimismo en la mayoría de sus actividades. ¿Cuál es el diagnóstico más
probable?

a) Anorexia nerviosa
b) Trastorno de ansiedad.
c) Trastorno Distímico
d) Depresión mayor

Criterios para el diagnóstico de


F34.1 Trastorno distímico (300.4)
A. Estado de ánimo crónicamente depresivo la mayor parte del día de la mayoría de los días,
manifestado por el sujeto u observado por los demás, durante al menos 2 años.

Nota: En los niños y adolescentes el estado de ánimo puede ser irritable y la duración debe
ser de al menos 1 año.

B. Presencia, mientras está deprimido, de dos (o más) de los siguientes síntomas:

1. Pérdida o aumento de apetito


2. Insomnio o hipersomnia
3. Falta de energía o fatiga
4. Baja autoestima
5. Dificultades para concentrarse o para tomar decisiones
6. Sentimientos de desesperanza

C. Durante el período de 2 años (1 año en niños y adolescentes) de la alteración, el sujeto no


ha estado sin síntomas de los Criterios A y B durante más de 2 meses seguidos.

D. No ha habido ningún episodio depresivo mayor durante los primeros 2 años de la


alteración (1 año para niños y adolescentes); por ejemplo, la alteración no se explica mejor
por la presencia de un trastorno depresivo mayor crónico o un trastorno depresivo mayor,
en remisión parcial.

Nota: Antes de la aparición del episodio distímico pudo haber un episodio depresivo mayor
previo que ha remitido totalmente (ningún signo o síntoma significativos durante 2 meses).
Además, tras los primeros 2 años (1 año en niños y adolescentes) de trastorno distímico,
puede haber episodios de trastorno depresivo mayor superpuestos, en cuyo caso cabe
realizar ambos diagnósticos si se cumplen los criterios para un episodio depresivo mayor.

E. Nunca ha habido un episodio maníaco, un episodio mixto o un episodio hipomaníaco y


nunca se han cumplido los criterios para el trastorno ciclotímíco.

F. La alteración no aparece exclusivamente en el transcurso de un trastorno psicótico


crónico, como son la esquizofrenia o el trastorno delirante.

G. Los síntomas no son debidos a los efectos fisiológicos directos de una sustancia (p. ej.,
una droga, un medicamento) o a enfermedad médica (p. ej., hipotiroidismo).

H. Los síntomas causan un malestar clínicamente significativo o deterioro social, laboral o


de otras áreas importantes de la actividad del individuo.
Especificar si:

Inicio temprano: Si el inicio es antes de los 21 años


Inicio tardío: si el inicio se produce a los 21 años o con posterioridad.

Especificar (para los últimos 2 años del Trastorno distímico):

Con síntomas atípicos

DSM IV

MANUAL DIAGNÓSTICO Y ESTADÍSTICO DE LOS TRASTORNOS MENTALES


(American Psychiatric Association)

26.- Mujer de 22 años que acude a consulta por pérdida de peso, diarrea, palpitaciones,
temblor y labilidad emocional. No tiene antecedentes importantes. Refiere evacuaciones
diarreicas de 5 a 7 por día, sin moco o sangre.
EF: peso 52 kg, estatura 1.59, FC 108x’, TA: 120/65, retracción palpebral, hiperemia
conjuntival y de carúncula, cuello con tiroides aumentada de tamaño 3 veces de forma
difusa, extremidades superiores con hiperhidrosis palmar, temblor fino distal, reflejos
exaltados.
Los estudios que debemos solicitar para hacer el diagnóstico es:

a) Coproparasitoscópico, coprológico y coprocultivo


b) Ultrasonido tiroideo
c) Pruebas de función tiroidea
d) Electrocardiograma y ecocardiograma

EVALUACIÓN DE UN PACIENTE CON SOSPECHA DE


ENFERMEDAD TIROIDEA
27.- Femenino de 26 años su padecimiento actual inició hace 4 años, cuando sus ciclos
menstruales empezaron a ser irregulares. Su ritmo actual es de 40-90 x 3-4. FUR: hace 3
meses. Inició vida sexual a los 23 años y no ha podido embarazarse. En la exploración física
encontramos la piel ligeramente seca, hay salida de líquido blanquecino escaso a la
expresión del pezón izquierdo y tiene giordano positivo derecho.

1. El estudio que es de mayor utilidad para aclarar el diagnóstico es:

a) Prueba de embarazo
b) LH, FSH y estrógenos
c) Tomografía de cráneo
d) Prolactina

En toda paciente con galactorrea, trastornos menstruales, hirsutismo, disminución de la


libido o infertilidad, deben determinarse los niveles de PRL plasmática, entre el tercer y
quinto día del ciclo menstrual, si este es regular o en cualquier momento, si presenta
amenorrea u oligomenorrea.2 Si en la primera determinación se obtienen cifras elevadas
debe repetirse y si se confirma nuevamente, se establece el diagnóstico de
hiperprolactinemia. Una historia clínica com-pleta, con un interrogatorio y examen físico
detallados, permitirá orientarnos hacia la causa de la hiperprolactinemia. La primera causa
que se debe descartar antes de emprender otros estudios, es el embarazo. El uso de
estrógenos, anticonceptivos orales y drogas que aumentan la secreción de PRL debe
precisarse en el interrogatorio, así como la presencia de quemaduras u otras lesiones en
tórax que se buscan en el examen físico.

Pocas mediciones hormonales tienen el significado clínico que se observa con la prolactina.
La técnica está bien estandarizada y presenta bastante confiabilidad; la muestra de sangre
se puede obtener a cualquier hora del día y debido a las variaciones fisiológicas (efecto del
ejercicio, alimentos, irritación de la pared costal) cuando se obtiene una cifra de prolactina
por arriba de lo normal, es necesario repetir el análisis.

Varios fármacos pueden producir una moderada elevación en los niveles de prolactina, sin
alcanzar los valores que se encuentran en prolactinoma; las drogas más comunes son
metoclopramida, fenotiazinas, risperidona, verapamil, metildopa, reserpina y los inhibidores
de MAO.

Exámenes: prolactina plasmática (prolactinemia), tirotropina (hormona estimulante de la


glándula tiroides o TSH), test de embarazo.
Prolactina menor de 100 ng/ml: baja sospecha de prolactinoma.
Prolactina entre 100 y 300 ng/ml: mayor posibilidad de prolactinoma.
Prolactina superior a 300 ng/ml: alta sospecha de prolactinoma. Es precisa resonancia
magnética nuclear (RNM) de hipófisis (detecta prolactinoma mayor de 3mm).
En el 50% de los casos de hiperprolactinemia se detecta tumor: microadenomas o
macroadenomas; la prolactina suele encontrarse entre 100 y 200ng/ml; los macroadenomas
tienen efecto de masa y causan hipopituitarismo; los microadenomas no presentan efecto
de masa y sólo producen hipogonadismo.

Referencias:

1. Schlechte JA. Prolactinoma. N Engl J Med 2003;349:2035-2041.


2. Zárate A, Canales ES, Jacobs LS, Soria J, Daughaday WH. Restoration of ovarian
function in patients with the amenorrhea-galactorrhea syndrome after long-term therapy
with L-Dopa. Fertil Steril 1973;24:340.
3. Tyson JE, Carter JN, Andreassen B, Huth J, Smith B. Nursing mediated
prolactin and luteinizing hormone secretion during puerperal lactation. Fertil
Steril 1978;30:154.
4. Schlechte JA, Sherman BM, Chapler FK, VanGilder J. Long-term followup of women
with surgically treated prolactin-secreting pituitary tumors. J
Clin Endocrinol Metab 1986;62:1296-301.
5. Losa M, Mortini P, Barzaghi R, Gioia L, Giovanelli M. Surgical treatment of
prolactin-secreting pituitary adenomas: early results and long-term outcome. J Clin
Endocrinol Metab 2002;87:3180-3186.

28.- Un hombre de 55 años presenta dolor precordial que le apareció cuando estaban en
reposo; se irradio al cuello y al hombro izquierdo, refiere que tuvo 20 minutos de duración
y cedió , tiene antecedentes de obesidad, Diabetes Mellitus, hipertensión y sedentarismo,
el diagnostico mas probable es:

a) Angina inestable.
b) Infarto agudo al miocardio.
c) Angina estable.
d) Pericarditis.

Entendemos por angina de pecho, o angor, un dolor torácico, generalmente retrosternal,


que puede ser descrito como opresivo, quemazón o simplemente una leve pesadez, y que
está motivado por la isquemia del miocardio. Este dolor o molestia puede irradiar o
presentarse únicamente en el cuello, mandíbula, hombros, brazos, antebrazos, manos,
espalda o epigastrio. Rara vez se presenta por encima de la mandíbula o debajo del
epigastrio. La isquemia miocárdica se presenta en ocasiones con síntomas distintos de la
angina como disnea, debilidad, fatiga o eructos. Estos síntomas son “equivalentes anginosos”
y suelen ser mas frecuentes en personas ancianas.
La situación en que se presenta la angina inestable no parece relacionada con un mayor
trabajo cardíaco. Es decir, la isquemia miocárdica no parece justificarse por un mayor
consumo miocárdico de oxígeno y, por lo tanto, la causa es una disminución aguda del flujo
sanguíneo coronario. Clásicamente se denomina angina inestable a la que se presenta en las
siguientes circunstancias:

1. Angina de reposo: ocurre en reposo o con un mínimo esfuerzo.


2. Angina de comienzo reciente: inicio de los síntomas en el último mes, en un paciente
previamente asintomático, y de aparición con esfuerzos mínimos.
3. Angina progresiva: En un paciente con angina estable previa, los síntomas se
presentan con esfuerzos menores, son cada vez mas frecuentes o su duración es
más prolongada.

La presentación clínica de la angina inestable puede ser idéntica a la de un infarto agudo de


miocardio (IAM), la diferencia es conceptual: si hay necrosis miocárdica hablamos de
infarto agudo de miocardio. Los datos que podemos obtener de la historia clínica, la
exploración física y el electrocardiograma, con frecuencia no permiten diferenciar entre
angina inestable e infarto agudo de miocardio. La elevación de marcadores de daño
miocárdico como la creatínfosfoquinasa (CPK) o la troponina T o I, identifican una necrosis
miocárdica y, por lo tanto un IAM. Es por este motivo que actualmente tanto la angina
inestable como el IAM se agrupan bajo el término de síndrome coronario agudo. Según la
presentación electrocardiográfica, el síndrome coronario agudo (SCA) se divide en SCA con
elevación del segmento ST (frecuentemente evoluciona a un infarto con onda Q) y SCA sin
elevación del segmento ST, que incluye a la angina inestable y la mayor parte de los casos
de IAM sin onda Q. Esta terminología es la que actualmente se utiliza porque tiene la
ventaja de clasificar el cuadro clínico del paciente a partir de datos clínicos y
electrocardiográficos que pueden obtenerse de modo rápido y sencillo.

LECTURA RECOMENDADA:

Guías clínicas para el manejo de la angina inestable e infartosin elevación del ST.
Estratificación del riesgo
Bibliografía Internacional
R. Marrón Tundidor*, P. Palazón Saura*, L. M. Claraco Vega*, C. Ascaso Martorell*,
J. Povar Marco*,
J. M. Franco Sorolla*, I. Calvo Cebollero**
*SERVICIO DE URGENCIAS Y **SERVICIO DE CARDIOLOGÍA-UNIDAD DE
HEMODINÁMICA Y CARDIOLOGÍA INTERVENCIONISTA.
29.- Acude a consulta un adolescente de 20 años la cual refiere que desde hace varios
meses presenta astenia, cansancio, pérdida de apetito y dificultades para concentrarse en
los estudios. Al interrogatorio refiere que a perdido interés en los estudios, frecuenta
menos a sus amigos, con pesimismo en la mayoría de sus actividades. ¿Cuál es el diagnóstico
más probable?

a) Anorexia nerviosa
b) Trastorno de ansiedad.
c) Distimia.
d) Depresión mayor

LA DISTIMIA

La distimia es un estado de ánimo crónicamente deprimido, menos grave que la depresión y


que no cumple los criterios para una depresión mayor, o lo hace sólo en períodos muy
cortos. Su evolución suele ser de más de dos años. Se caracteriza por un abatimiento
prolongado del estado de ánimo en que el sujeto distímico se describe a sí mismo como
“triste” o “desanimado”, perdiendo el interés por las cosas y viéndose a menudo como inútil
y poco interesante. Posee síntomas persistentes o intermitentes, de intensidad más leve
comparación a la depresión mayor. Aiskal (1983) la define como “mal humor” y se
caracteriza porque “el individuo está habitualmente triste, introvertido, melancólico,
excesivamente consciente, incapaz de alegría y preocupado por su insuficiencia personal”.

Los criterios de diagnóstico de Distimia son los que a continuación se detallan:

A. Estado de ánimo crónicamente depresivo la mayor parte del día de la mayoría de los días,
manifestado por el sujeto u observado por los demás, durante al menos 2 años. Nota: En los
niños y adolescentes el estado de ánimo puede ser irritable y la duración debe ser de al
menos 1 año.

B. Presencia, mientras está deprimido, de dos (o más) de los siguientes síntomas:

1. Pérdida o aumento de apetito


2. Insomnio o hipersomnia
3. Falta de energía o fatiga
4. Baja autoestima
5. Dificultades para concentrarse o para tomar decisiones
6. Sentimientos de desesperanza
C. Durante el período de 2 años (1 año en niños y adolescentes) de la alteración, el sujeto
no ha estado sin síntomas de los Criterios A y B durante más de 2 meses seguidos.

D. No ha habido ningún episodio depresivo mayor durante los primeros 2 años de la


alteración (1 año para niños y adolescentes); por ejemplo, la alteración no se explica mejor
por la presencia de un trastorno depresivo mayor crónico o un trastorno depresivo mayor,
en remisión parcial.

Nota: Antes de la aparición del episodio distímico pudo haber un episodio depresivo mayor
previo que ha remitido totalmente (ningún signo o síntoma significativos durante 2 meses).
Además, tras los primeros 2 años (1 año en niños y adolescentes) de trastorno distímico,
puede haber episodios de trastorno depresivo mayor superpuestos, en cuyo caso cabe
realizar ambos diagnósticos si se cumplen los criterios para un episodio depresivo mayor.

E. Nunca ha habido un episodio maníaco, un episodio mixto o un episodio hipomaníaco y


nunca se han cumplido los criterios para el trastorno ciclotímíco.

F. La alteración no aparece exclusivamente en el transcurso de un trastorno psicótico


crónico, como son la esquizofrenia o el trastorno delirante.

G. Los síntomas no son debidos a los efectos fisiológicos directos de una sustancia (p. ej.,
una droga, un medicamento) o a enfermedad médica (p. ej., hipotiroidismo).

H. Los síntomas causan un malestar clínicamente significativo o deterioro social, laboral o


de otras áreas importantes de la actividad del individuo.

Como vemos, la Distimia presenta síntomas más o menos similares a la Depresión mayor,
pero se diferencian entre sí respecto a:

Tipo de evolución: el estado de ánimo depresivo es crónico (no presenta intervalos libres
de síntomas o mejorías significativas) y dura por lo menos 2 años.

Severidad de los síntomas: los mismos suelen ser leves o moderados, sin una alteración
significativa de las relaciones familiares, sociales y laborales del individuo.

Referencias Bibliográficas

- Diagnostic and Statistical Manual of Mental Disorders, 4° edition (DSM-IV TR).


American Psychiatric Association. American Psychiatric Press, 2000.
- Kaplan and Sadock's Synopsis of Psychiatry, 9° edition. Lippincott Williams & Wilkins
Press, 2003.
30.- Recurren a consulta los padres de un menor, que cuenta con cinco años de edad, lleva
varias noches despertándose agitado como si hubiera soñado algo que le angustia. Cuando
acuden a su lado por la noche, el niño les mira y dice palabras que no tienen ningún
significado. Al cabo de un rato vuelve a dormirse y por la mañana no recuerda nada de lo
ocurrido. El diagnóstico sería:
a) Terrores nocturnos.
b) Pesadillas.
c) Disomnia.
d) Sonambulismo.

Criterios para el diagnóstico de F51.5 Pesadillas (307.47)

A. Despertares repetidos durante el período de sueño mayor o en las siestas diurnas,


provocados por sueños extremadamente terroríficos y prolongados que dejan recuerdos
vividos, y cuyo contenido suele centrarse en amenazas para la propia supervivencia,
seguridad o autoestima. Los despertares suelen ocurrir durante la segunda mitad del
período de sueño.

B. Al despertarse del sueño terrorífico, la persona recupera rápidamente el estado


orientado y despierto (a diferencia de la confusión y desorientación que caracterizan los
terrores nocturnos y algunas formas de epilepsia).

C. Las pesadillas, o la alteración del sueño determinada por los continuos despertares,
provocan malestar clínicamente significativo o deterioro social, laboral o de otras áreas
importantes de la actividad del individuo.

D. Las pesadillas no aparecen exclusivamente en el transcurso de otro trastorno mental (p.


ej., delirium, trastorno por estrés postraumático) y no se deben a los efectos fisiológicos
directos de una sustancia (p. ej., drogas, fármacos) o de una enfermedad médica.

Criterios para el diagnóstico de F51.4 Terrores nocturnos (307.46)

A. Episodios recurrentes de despertares bruscos, que se producen generalmente durante


el primer tercio del episodio de sueño mayor y que se inician con un grito de angustia.

B. Aparición durante el episodio de miedo y signos de activación vegetativa de carácter


intenso, por ejemplo, taquicardia, taquipnea y sudoración.

C. El individuo muestra una falta relativa de respuesta a los esfuerzos de los demás por
tranquilizarle.

D. Existe amnesia del episodio: el individuo no puede describir recuerdo alguno detallado
de lo acontecido durante la noche.

E. Estos episodios provocan malestar clínicamente significativo o deterioro social, laboral,


o de otras áreas importantes de la actividad del individuo.

F. La alteración no se debe a los efectos fisiológicos directos de una sustancia (p. ej.,
drogas, fármacos) o de una enfermedad médica.
31.- Masculino de 43 años que ingresa al servicio de urgencias por haber sufrido colisión
automovilística de frente. Sufre fractura en tibia y peroné, se practicó cirugía ortopédica
sin complicaciones y, durante los dos primeros días del postoperatorio, el paciente estuvo
inquieto pero bien orientado. Al tercer día, el paciente se mostró de repente confuso y
temeroso y refirió visión de serpientes y arañas que se desplazaban en su cama. En la ex-
ploración se observó midriasis, temblor de oscilaciones amplias de las manos y los párpados,
sudoración profusa, taquicardia con latido hipercinético y temperatura de 39ºC. Como
antecedente se consideraba un bebedor social moderado y negaba el abuso de drogas. El
diagnóstico más probable es:

a) Delirio por deprivación alcohólica.


b) Delirio postanestesia.
c) Septicemia.
d) Hematoma subdural.

CRITERIOS DIAGNÓSTICOS DE LA ASOCIACIÓN AMERICANA DE PSIQUIATRÍA


PARA LA DEPRIVACIÓN ALCOHÓLICA Y EL DELIRIO POR DEPRIVACIÓN
ALCOHÓLICA (DSM-IV)

DEPRIVACIÓN ALCOHÓLICA
A.-Cese o disminución del uso prolongado e intenso de alcohol
B.-Dos o más de los siguientes criterios, entre varias horas y
varios días después de A:
1-Hiperactividad autonómica (diaforesis, taquicardia…)
2-Aumento de temblor en manos
3-Insomnio
4-Náuseas y vómitos
5-Ilusiones o alucinaciones transitorias visuales,
táctiles o auditivas
6-Agitación psicomotriz
7-Ansiedad
Crisis epilépticas tipo gran mal
C.-Agotamiento y angustia significativas o deterioro social y
ocupacional en relación con B
D.-Exclusión de otras condiciones médicas generales y de otros
trastornos mentales
DELIRIO POR DEPRIVACIÓN ALCOHÓLICA
A.-Alteración del nivel de consciencia con reducción de
habilidad para fijar, mantener o cambiar la atención
B.-Cambio en la cognición o desarrollo de trastorno perceptual,
no explicado por demencia preexistente
C.-Se desarrolla en horas o días y tiende a fluctuar durante el
día y tras un síndrome de deprivación
32.- Masculino de 50 años que presenta súbitamente dolor intenso en primer ortejo de pie
derecho durante la noche posterior, tras ingesta de alcohol. Con los datos clínicos
anteriores usted pensaría que el paciente cuenta con:

a) Hiperuricemia crónica sintomática.


b) Síndrome de reiter.
c) Monoartritis infecciosa.
d) Artritis gotosa.

Las causas que generan hiperuricemia son múltiples, pero en general podemos dividirlo en
tres grupos:1° Hiperuricemia idiopática (10-15%) que representa los errores innatos del
metabolismo, padecimientos caracterizados por un incremento en la síntesis de purinas y
sobreproducción de ácido úrico; 2° Por incremento del recambio metabólico y que genera;
un exceso de ácido úrico sérico, observado en los síndromes mieloproliferativos, neoplasias,
etcétera, y 3° grupo conformado por padecimientos o condiciones que producen baja
excreción renal de ácido úrico. La acidosis metabólica, insuficiencia renal, enfermedades
metabólicas endocrinológicas y el uso de fármacos, son las causas más comunes de esta
situación.
Factores de Riesgo: Todas las causas de hiperuricemia son un factor de riesgo para la
gota. 1) La obesidad o el aumento o pérdida repentinos de peso; 2) mayores de 40 años; 3)
sexo masculino; 4) Miembros de la familia con gota;5) Diuréticos, tales como el
hidroclorotiazido; 6) Algunas medicinas, como la aspirina; 7) Una diera rica en cristales de
nitrógeno; 8) Consumo de alcohol; 9) Algunos tipos de cáncer o tratamientos contra el
cáncer (por ejemplo, medicinas citotóxicas); 10) Medicinas (tales como los que combaten la
apoplejía y otros); 11) Deshidratación;12) Hipercolesterolemia;13) Enfermedad renal; 12)
Desordenes endocrinos, como el hipotiroidismo y el hiperparatiroidismo

Como es una enfermedad inflamatoria tiene un comportamiento episódico, de distribución


universal, predomina en el hombre con el 90% de los casos y 10% en mujeres, de
preferencia posmenopáusicas. De inicio mas frecuente entra la 4° y 6° década. Con
diferentes estadios conocidos como hiperuricemia sintomática, artritis gotosa aguda,
periodo intercrítico y gota crónica tofácea.
Entonces, de acuerdo a los estadios de la gota podemos mencionar:
A Hiperuricemia asintomática
B Artritis Gotosa: Es la mas común y se caracteriza por dolor agudo, intenso, que afecta a
una ó más articulaciones de predominio monoarticularlas, de acuerdo a orden de frecuencia
son; el dedo gordo del pie tobillo, tarso, rodilla y muñeca.
C Periodo Intercrítico: intervalo entre un ataque y otro. El paciente queda libre de
síntomas hasta la presentación del siguiente ataque de artritis, la duración del periodo
asintomático es variable. Posteriormente los periodos asintomáticos se van acortando y el
paciente evoluciona hacia la cronicidad.

Gota Tofácea Crónica: Caracterizada por periodos recidivantes de artritis con depósitos
de uratos conocidos como tofos, tumoraciones sobre la articulación que pueden estar
excretando un material calcáreo.
Rev Paceña Med Fam 2006; 3(3): 2-5
Dra. Nilsa Selaya C.
Dr. Cesar Rabaza M.
Dr. Rafael Castillo R.

33.- Un paciente con insuficiencia renal crónica debida a hipotensión prolongada grave es
atendido porque presenta dolor retroesternal. Se le indicó hemodiálisis dos veces por
semana en los últimos dos años y en fechas recientes ha experimentado episodios de
hipotensión al inicio del tratamiento. El dolor se localiza sobre el músculo trapecio. Se
reduce un poco al adoptar la posición de pie y se exacerba con la respiración profunda.
¿Cuál de los siguientes trastornos es la causa más probable del dolor retroesternal que
sufre este paciente?

a) Arteriopatía coronaria
b) Espasmo esofágico difuso
c) Pericarditis
d) Embolias pulmonares

El dolor torácico que sufrió este paciente es característico de pericarditis e inflamación


del pericardio, complicaciones comunes en personas con insuficiencia renal crónica en
hemodiálisis. Estos enfermos también pueden tener inflamación de varios recubrimientos
serosos, como peritoneo y pleura; no se conoce el mecanismo de esta complicación. Aunque
es común la arteriopatía coronaria en pacientes en diálisis, las características de dolor en
este individuo sugieren que no es el diagnóstico. La enfermedad esofágica también es
común en sujetos en diálisis y debe descartarse específicamente como posible causa. La
relación aparente con la diálisis, así como la frecuencia de los síntomas, van contra el
diagnóstico de embolia pulmonar como causa del dolor torácico. Además, el dolor
musculoesquelético debido a diversos trastornos se observa en pacientes en diálisis y
puede deberse a anomalías en el metabolismo del calcio y el fósforo, que produce depósitos
de calcio en diversos componentes del sistema musculoesquelético.

1. Alexander JS. A pericardial effusion of gold paint appearance due to the presence of
cholesterin. Br Med J 1919; 2: 463.
2. Brawley RK, Vasko JS, Morrrow AG. Cholesterol pericarditis: consideration of its
pathogenesis and treatment. Am J Med 1966; 41: 235-248.
34.- Se reportaron 45 casos de varicela entre los alumnos de primer grado de una escuela
secundaria durante la segunda quincena del mes de agosto de 2009. La tasa de ataque es
de 26.4%. Esta situación nos indica que estamos ante un(a):

a) Epidemia
b) Endemia
c) Brote
d) Pandemia

De acuerdo a la NOM- 017 un brote se define como la ocurrencia de dos o más casos
asociados epidemiológicamente (tiempo, lugar y persona) entre sí. La medida cuantitativa
de la extensión de un brote es la Tasa de Ataque (TA) que se calcula dividiendo el número
de casos nuevos entre el total de personas expuestas por 100.

Norma Oficial Mexicana NOM-017-SSA2 -1994, Para la vigilancia epidemiológica.


Apartados 3.1.6 y 3.1.78.1.

35.- Femenino de 78 años acude a consulta externa por referir debilidad general y apatía,
añadiéndose en la última semana disnea progresiva hasta ser de pequeños esfuerzos.
Antecedentes: cardiopatía hipertensiva con función sistólica conservada, en los últimos 2
años ha tenido 3 episodios de fibrilación paroxística cardiovertidos eléctricamente.
Durante este tiempo ha recibido diversos tratamientos que incluían algunos de los
siguientes fármacos: propafenona, amiodarona, digoxina, diltiacem y captopril. El ECG
muestra fibrilación auricular con frecuencia ventricular a 130 lpm, RX de tórax
cardiomegalia signos de congestión pulmonar y el estudio de función tiroidea una T4 libre
elevada con una TSH indetectable.
El medicamento responsable de la sintomatología de esta paciente es:

a) Propafenona.
b) Amiodarona.
c) Digoxina.
d) Diltiacem.

Alteraciones de lab por amiodarona:


Hepáticos (1,2%) Elevación del ASAT y ALAT 2-3 veces valor normal
Elevaciones persistentes de LDH y Fosfatasa alcalina
· Gástricos (4,2%) Naúseas. Vómitos. Estreñimiento —
· Pulmonares (1,9%) * Síntomas respiratorios y nuevas alteraciones en Rx de tórax
> 30% de reducción en difusión pulmonar.
(PFR)
· Tiroides (3,7%) Hipertiroidismo o hipotiroidismo clínico
Cambios en la función tiroidea (TSH, T4, T3) que requiere medicación
· Neurológicos (4,6%) Temblor. Ataxia. Disestesias Insomnio
· Piel (2,3%)
Fotosensibilidad; coloración azul/grisácea
(piel)

· Ojos (1,5%) Alteraciones visuales. Halos visuales
Visión nocturna borrosa. Depósitos corneales en la lámpara de hendidura.
· Bradicardia/Conducción (3,3%)
Bradicardia sintomática. Bloqueo A-V de 3er grado
Bradicardia < 50 lpm asintomática. Bloqueo
A-V de 1er y 2º grado
· Supresión del fármaco (23%) ® Por efectos adversos no tolerados, decisión del paciente o
no cumplimentación de prescripción farmacológica
(*) Se relaciona con la dosis (más frecuente en dosis >400 mg/día) y duración del
tratamiento.

Referencias:
Volumen 13, Número 4
Octubre - Diciembre 2002
pp 149 – 152
Cardiología
Trabajo de investigación
* Cardiólogo, Ex-Residente Hospital de Especialidades, Centro
Médico Nacional “La Raza”, IMSS. Hospital de la Fe, San Miguel
Allende, Gto.
** Jefe del Servicio de Cardiología, Hospital de Especialidades
Centro Médico Nacional “La Raza”, IMSS.

36.- En el caso de encontrar una diarrea con presencia de moco, sangre y proteínas
procedentes de una mucosa intestinal inflamada constituye una diarrea:

a) Osmótica
b) Exudativa
c) Secretora
d) Motora

Diarrea exudativa. Es producto de la inflamación, ulceración de la mucosa intestinal y


alteración de la permeabilidad para agua, electrolitos y solutos pequeños como la úrea. . Se
trata de deposiciones que contienen moco, sangre, proteínas y pus. Puede tener algunos
componentes de la diarrea secretora como consecuencia de la liberación de prostaglandinas
por células inflamatorias. Es consecuencia de infecciones bacterianas (Salmonella),
clostridium difficile (frecuentemente inducidos por antibióticos) parásitos del colon
(Entamoeba histolytica), enfermedad de Crohn, enterocolitis por radiación e isquemia
intestinal, proctocolitis ulcerativa y enfermedad intestinal inflamatoria idiopática.
LECTURAS RECOMENDADAS:

1. Avery ME. Snyder JD. Oral therapy for acute diarrhea. New Engl J Med 323:891,
1990.
2. Campos J. Tercer Curso Anual de Actualizaciones en Medicina Interna.
3. Fundación Santa Fe de Bogotá. Santafé de Bogotá, 1990.
4. Diaz H, Campos J. Diarrea aguda. En: Medicina Interna. F Chalem, J Escandón, J
Campos,
5. R Esguerra, editores. Fundación Instituto de Reumatología e Inmunología. Editorial
Presencia Ltda. Santafé de Bogota, 1992.
6. Donowitz M et al. Drug therapy for diarrheal diseases. A look ahead. Rev Infect
Dis 8:202, 1986.

37. - Which of the following drugs combination would be most appropriate in the patient
treatment of acute pelvic inflammatory disease?

a) Ampicillin / cefoxitin
b) Tetracycline / gentamicin
c) Cefoxitin / Doxycycline
d) Ampicillin / Amikacin

E.P.I.
Regímenes de tratamiento CDC 2002

• Parenteral.
– Cefotetan 2gr iv/12h ó Cefoxitina 2gr iv/6h + Doxiciclina 100 mgs iv/8h.
– Clindamicina 900 mg iv/8h + Gentamicina iv/im (2mg/kg de carga, luego 1.5
mg/kg/8h.

• Alternativas.
– Ofloxacina 400 mg iv/12h ó Levofloxacino 500 mg iv diario con o sin
Metronidazol 500 mg iv/12 h.

• Oral.
– Ofloxacina 400 mg vo diario x 14 días ó Levofloxacina 500 mg vo diario x 14
días con o sin Metronidazol 500 mg vo diario por 14 días.
– Ceftriaxona 250 mg im x 1 dosis ó Cefoxitina 2 gr im x 1 dosis y Probenecid
1 gr vo x 1 dosis u otra cefalosporina de 3ª gen im + Doxiciclina 100 mgs
vo diario or 14 días con o sin Metronidazol 500 mgs vo diario x 14 días.
38.- Se trata de paciente de 32 años con antecedente de cuadros catarrales y tabaquismo
crónico, inicia con molestias faríngeas que progresan rápidamente a dolor intenso de
garganta e impide la deglución acompañado de fiebre. Al acudir al servicio y ser asistido,
se niega a acostarse en camilla, permaneciendo sentado e inclinado hacia delante. El diag-
nóstico más probable será:

a) Laringitis catarral aguda.


b) Angina de Ludwick.
c) Edema de Reinke.
d) Amigdalitis bacteriana.

Edema de Reinke

Son lesiones en las que se produce un acumulo de un liquido fluido, gelatinoso, bajo la
cubierta mucosa de las cuerdas vocales. Son generados por fenómenos de inflamación
crónica debidas al abuso vocal y al consumo importante de tabaco que encontramos en más
del 90% de los pacientes. Esta descrito que puede aparecer en casos de hipotiroidismo y
asociado al reflujo gastroesofágico.

El edema suele afectar a ambas cuerdas vocales y es de aparición más frecuente en los
varones. Se presenta como una disfonía progresiva con voz ronca y con un tono más bajo,
voces más graves y con menos potencia vocal y tendencia a la fatiga en los casos más
intensos.

El edema de Reinke puede mejorar claramente evitando el consumo de tabaco, el carraspeo


y modificando los malos hábitos vocales. Una correcta rehabilitación vocal será el
complemento adecuado en la gran mayoría de los casos.

Se considerará el tratamiento con microcirugía laringea en los casos crónicos, refractarios


al tratamiento conservador y en las que la calidad de voz es mala incapacitando al paciente
para su vida de relación o de trabajo. En algunos profesionales de la voz encontramos
edemas de Reinke con voces más graves, cálidas que conforman una ¨firma vocal¨
característica que les identifica y que no les interesa cambiar por lo que no precisaran de
un tratamiento agresivo.
Clínica Exploración Actitud y consejos Evolución

Disfonía de larga Laringoscopia Medidas de higiene Mejoría


evolución con voz indirecta: vocal Recidivas si
grave sobre todo - Irritación y edema Protectores persisten hábitos
matutina organizado en gástricos tabáquicos
Disnea ocasional forma de bolsas Microcirugía Riesgo de cicatrices
Relación con abuso en los 2/3 laríngea con incisión vocales
del tabaco anteriores de y aspirado del edema No degeneran
ambos pliegues en 1 ó 2
Predominio en aunque se asocian a
varones de 50-60 - Defecto de cierre intervenciones laringitis crónicas
años, pero aumenta posterior Supresión absoluta
la incidencia en - Aspecto del tabaco
mujeres gelatinoso o rojizo Rehabilitación
Relación con logopédica
reflujo gastro- ocasionalmente
esofágico

Bruch JM. Hoarseness in adults [Internet]. Waltham, MA: UpToDate, Rose, BD (Ed),
version 17.1 ; 2009 [acceso 6/4/2009]. Disponible en:
Chadha NK, James AL. Tratamiento antiviral adyuvante para la papilomatosis respiratoria
recurrente (Revisión Cochrane traducida). En: La Biblioteca Cochrane Plus, número 3,
2008. Oxford, Update Software Ltd. Disponible en: Colton R, Casper J, Leonard R.
Understanding Voice Problems. A Physiological Perspective for Diagnosis and
Treatment. 3rd ed. Baltimore Philadelphia: Lippincott Williams & Wilkins; 2006.
Gallagher TQ, Derkay CS. Recurrent respiratory papillomatosis: update 2008. Curr Opin
Otolaryngol Head Neck Surg. 2008;16(6):536-42.
García-Tapia R, Cobeta I. Clasificación de las disfonías. En: Diagnóstico y tratamiento de
los trastornos de la voz. Eds: García-Tapia R y Cobeta I. Editorial Garsi, S.A. 1996.
Madrid.

Ishizuka T, Hisada T, Aoki H, Yanagitani N, Kaira K, Utsugi M et al. Gender and age risks for
hoarseness and dysphonia with use of a dry powder fluticasone propionate inhaler in
asthma. Allergy Asthma Proc. 2007 Sep-Oct;28(5):550-6.

39.- Un niño de 5 años no inmunizado acude a consulta con una historia de 2 semanas de
evolución de tos paroxística, fiebre de bajo grado, emesis post-tos y descarga nasal
viscosa. El EF revela otitis media bilateral y conjuntivitis hemorrágica. Se auscultan
estertores inspiratorios bilateralmente. BH con LT 45 000, con 95% de linfocitos. ¿Cuál de
los siguientes es el diagnóstico más probable?
a) Neumonía por Chlamydia
b) Tosferina
c) Bronquiolitis
d) Neumonitis por VSR

La tos ferina es una enfermedad infecciosa altamente contagiosa causad por un bacilo gram
(-) llamado Bordetella pertussis.
El comienzo suele ser insidioso con una fase catarral, con tos irritante que poco a poco se
vuelve paroxística, por lo regular en el término de una a dos semanas, y que dura de uno a
dos meses o más. Los paroxismos se caracterizan por accesos repetidos y violentos de tos;
cada serie de ellos comprende innumerables toses sin inspiración intermedia y puede ser
seguida por un estridor respiratorio de tono alto característico. Los paroxismos con
frecuencia culminan con la expulsión de mucosidades claras y adherentes, a menudo seguida
de vómito. Los lactantes menores de 6 meses de edad, los adolescentes y los adultos
frecuentemente no tienen el cuadro típico de estridores o tos paroxística.
Resumen:
Etiología: Bordetella pertussis.
Otros: Bordetella parapertussis, Bordetella bronchiseptica, Mycoplasma pneumoniae,
Chlamydia trachomatis, Chlamydia pnuemoniae y adenovirus.
Fuentes de contagio adolescentes y adultos.
Infección intradocimiciliaria en el 80 % de los no vacunados.
Contagio: Fase catarral y hasta 2 semanas de iniciada la tos.
Periodo de incubación de 10 a 21 días.
Cultivo nasofaríngeo Bordet-Gengou (Dacrón o alginato de calcio).
Negativo en fase temprana, > 4 semanas sin estaba vacunada y > 5 días si recibió
tratamiento.

Leucocitosis con linfocitosis absoluta.

Tratamiento:
• Apoyo: Apnea, hipoxia y otras complicaciones.
• Antibiótico en fase catarral es efectivo.
• Frenar la propagación del germen.
• Eritromicina 40-50 mgkgd cada 6 hrs. x 14 días.
• Claritromicina 15-20 mgkgd cada 12hrs. x 7 días.
• Azitromicina 10-12 mgkgd cada 24 hrs. x 5 días.
• TMP SMZ 8 mgkgd cada 12 hrs. x 14 días.

Bibliografía:

• Abul K. Abbas, Andrew H. Lichtman, Jordan S. Pober. Inmunología Celular y


Molecular.
McGrall-Hill Interamericana. Cuarta edición 2001.
• Napoleón González Saldaña y Mercedes Macías Parra. Vacunas en Pediatría.
McGrall-Hill
Interamericana. Primera edición 1999.
• Report of the Committee on Infectious Diseases. American Academy of Pediatrics.
Red
Book 2000. 25 edición 2000.
• Stanley A. Plotkin, Walter A. Orenstein. Vaccines. W. S. Saunders Company.
Tercera edición 1999.

40.- Se trata de paciente femenino de 52 años de edad a la cual se le diagnostica


neumonía por aspiración, aislando en esputo bacilo Gram negativo anaerobio, en su historia
clínica niega alergia medicamentosa ¿Cuál es el tratamiento antimicrobiano de elección?

a) Metronidazol.
b) Eritromicina.
c) Ceftriaxona.
d) Ciprofloxacino

NEUMONIA POR ASPIRACION

La aspiración de contenido gástrico ocurre con relativa frecuencia en pacientes


severamente enfermos. Los factores predisponentes incluyen alteraciones mentales,
anestesia general, traqueostomía e intubación nasogástrica. Las consecuencias de la
aspiración dependen de la naturaleza del contenido gástrico:

a. Entre mayor sea el volumen aspirado, mayor será el daño causado


b. La acidez gástrica causa inflamación con exudación, disminución de surfactante y
mayor riesgo de atelectasis
c. Las partículas alimenticias inician una reacción granulomatosa
d. Las bacterias de las secreciones orofaringeas complican la respuesta inflamatoria

La aspiración es causa de fiebre y de la aparición de infiltrados pulmonares. La presencia


de broncoespasmo e hipoxemia sugieren el diagnóstico que evoluciona hacia un cuadro de
neumonitis y una infección bacteriana. La consolidación aparece rápidamente acompañada

de tos productiva y esputo purulento. Los cultivos de esputo muestran en el 63% de los
casos flora aeróbica, en el 7% anaerobios y en el 30% flora mixta.
ANAEROBIOS

Los anaerobios son la flora predominante en las secreciones orafaringeas. Por lo tanto, la
vía de infección por estos gérmenes es la aspiración. Existen cuatro patrones de infección:
1) neumonitis aguda; 2) neumonía necrotizante; 3) absceso pulmonar y 4) empiema. Todos
están relacionados entre sí puesto que una neumonía aguda si no se trata, evoluciona a un
proceso necrotizante que se puede localizar, dando origen a un absceso que al romperse
hacia el espacio pleural produce un empiema.

Hallazgos clínicos. Tos productiva con esputos de mal olor, dolor pleurítico; si existe un
absceso la respiración es anfórica o cavernosa.

Hallazgos de laboratorio. Una muestra obtenida por punción transtraqueal permite su


cultivo en un ambiente para anaerobiosis. Para obtener un resultado óptimo la muestra se
debe llevar al laboratorio en jeringas sin aire. Las infecciones pulmonares anaeróbicas que
aparecen en pacientes hospitalizados generalmente son de tipo mixto y con frecuencia se
encuentran Staphylococcus o bacilos gram-negativos. Los microorganismos más importantes
son el Bacteroides melaninogenicus y el Fusobacterium nucleatum y los estroptococos
anaeróbicos propios de la flora existente en la orofaringe. En el caso de enfermedades
abdominales como la obstrucción abdominal o cirugía, una causa frecuente de infección es
el B. fragilis.

Tratamiento. La mayoría de los microorganismos, con excepción del B. fragilis son


sensibles a la penicilina G. y la carbenicilina. Casi todos, incluyendo el B. fragilis son
sensibles al cloranfenicol y a la clindamicina. El metronidazol es eficaz contra la mayoría de
los anaeróbicos incluyendo del B. fragilis. En la neumonía aguda se recomienda la penicilina
cristalina 2 millones de unidades IV cada 4 horas; en la neumonía necrotizante y el absceso
pulmonar se prescribe penicilina cristalina más metronidazol. Es aconsejable practicar una
fibrobroncoscopia ante la presencia de un absceso para descartar un cuerpo extraño o una
lesión maligna. En el empiema anaeróbico, la maniobra terapeútica más importante es el
drenaje adecuado por medio de un tubo torácico. Se utilizan dosis altas de penicilina
cristalina (3 millones de unidades cada 4 horas), más clindamicina 600 mg IV cada 6 horas o
metronidazol, 15 mg/kg/6 horas, IV.
Referencias:

1. Des Jardins T. Neumonía. En: Enfermedades Respiratorias. Editorial El Manual


Moderno. México DF, 1993.
2. Glackman RA. Oral empirical treatment of pneumonia. The Challenge of Choosing
the Best Agent. Postgrad Med 95:165, 1994.
3. Jauregui A. Infecciones de vías respiratorias superiores y neumonías. En: Manual
de Terapéutica Médica. Instituto Nacional de la Nutrición. Interamericana. Mc
Graw Hill. Méjico DF, 1994.
4. La Force-FM. Antibacterial therapy for lower respiratory tract infections in adults:
a review. Clin Infect Dis. 14 Suppl 2:S233; discussion S244, 1992.
5. Londoño F. Neumonías. En: Neumología. Editado por Jorge Restrepo M, Darío
Maldonado G. Fundamentos de Medicina, Corporación para Investigaciones
Biológicas. Medellín, 1986.
6. Tobin M. Diagnosis of pneumonia, techniques and problems. Clin Chest Med 8:513,
1997.

41.-Masculino de 40 años que de manera inesperada y espontánea inicia cuadro


constituido por disnea, sensación de ahogo o de paro respiratorio, sensación de
inestabilidad, “siente que se desmaya”, palpitaciones o taquicardia, mareo, nauseas, dolor
abdominal. ¿Cuál de los siguientes el diagnóstico mas probable?

a) Trastornos de ansiedad generalizada.


b) Trastorno fóbico.
c) Trastorno de pánico.
d) Trastorno mixto ansioso-deperesivo.

La característica fundamental del Trastorno de Pánico es la presencia de Crisis de Pánico


recurrentes, inesperadas y que no se encuentran relacionadas con ninguna circunstancia en
particular (es decir son espontáneas), sin un factor externo que las desencadene, es decir
que no son desencadenadas por una exposición a una situación social (como es el caso de la
Fobia Social) o frente a un objeto temido (en cuyo caso se trataría de una Fobia
Específica ). Otra característica del Trastorno de Pánico es el miedo persistente a padecer
una nueva Crisis de Pánico, esto es miedo al miedo y se lo denomina Ansiedad Anticipatoria.
Esta ansiedad puede a llegar a ser tan importante que puede llevar a confundir el
diagnóstico de Trastorno de Pánico con el de Trastorno de Ansiedad Generalizada (TAG).
Estas Crisis (o Ataques) de Pánico, inesperadas y recidivantes, suelen presentar cuatro o
más de los siguientes síntomas:

• Miedo intenso a morir o a estar sufriendo un ataque cardíaco o alguna enfermedad


física grave que ponga en riesgo la vida
• Miedo intenso a volverse loco o a perder el control de si mismo
• Palpitaciones (percepción del latido cardíaco) o pulsaciones aceleradas (taquicardia)
• Sudoración
• Palidez
• Temblores o sacudidas musculares
• Sensación de ahogo o falta de aire
• Opresión en la garganta (sensación de no poder respirar) o en el pecho
• Náuseas, vómitos o molestias y dolores abdominales
• Inestabilidad, mareos o desmayos
• Sensación de irrealidad (sentir al mundo externo como algo extraño)
• Sensación de no ser uno mismo (despersonalización)
• Hormigueos (parestesias)
• Escalofríos o sensación de sufrir frío intenso

Las crisis se inician bruscamente, alcanzan su máxima intensidad en los primeros diez a 15
minutos y suelen durar menos de una hora. Dejan a quien las sufre en un estado de total
agotamiento psicofísico y con un gran temor (ansiedad anticipatoria) a volver a padecer una
nueva crisis (miedo al miedo). También pueden aparecer síntomas de tipo depresivo, miedo a
salir o alejarse del hogar o necesidad de hacerlo acompañado por una ser muy cercano
(familiar). La persona suele sentirse muy hipersensible y vulnerable. Este tipo de trastorno
es tan traumático de experimentar que quien lo padece suele cambiar en forma brusca y
desfavorable sus hábitos de vida: no querer salir solo de la casa o viajar, retraimiento
social, abandono de sus actividades laborales o académicas.

Muchas veces la persona que se encuentra padeciendo una Crisis de Pánico tiene la
necesidad de "salir corriendo" del lugar donde se encuentra o de consultar urgentemente a
un médico en el caso que crea que se esta muriendo de verdad. Si esta crisis se
experimenta por primera vez en un lugar determinado, un tren o autobús por ejemplo, suele
quedar temor de volver a ese mismo sitio o medio de locomoción, desarrollándose de este
modo una fobia al mismo.

Un paciente que sufría de Crisis de Pánico describió su primera crisis de la siguiente forma:
"Súbitamente sentí una oleada de miedo sin que hubiera razón alguna. El corazón me latía
apresuradamente, me faltaba el aire, sentía que no podía respirar, el corazón latía tan
fuerte que parecía que iba a salirse por mi boca. Me dolía el pecho, tenía mareos, ganas de
vomitar, no podía parar de temblar. Sentía que me moría. Era como una agonía mortal que
nunca terminaba"
Este trastorno se lo clasifica dentro de las neurosis, no tiene nada que ver con la locura
(psicosis), ni desencadena con el tiempo en un cuadro de locura. Es importante recalcar
esto ya que muchos pacientes que sufren este trastorno, o los familiares del mismo,
piensan que se están volviendo locos. Lo que sí es frecuente es que las personas que
padecen de Trastorno de Pánico desarrollen, si no son tratados a tiempo y adecuadamente,
cuadros de tipo depresivos, fobias múltiples (especialmente agorafobia) o abuso de
sustancias (alcohol o drogas).

Referencias Bibliográficas:

- Diagnostic and Statistical Manual of Mental Disorders, 4° edition (DSM-IV TR).


American Psychiatric Association. American Psychiatric Press, 2000.
- Kaplan and Sadock's Synopsis of Psychiatry, 9° edition. Lippincott Williams & Wilkins
Press, 2003.

42.- Acude a consulta una mujer de 35 años acompañada de su marido. Ella dice en-
contrarse muy bien, mejor que nunca, está pudiendo hacer más cosas e incluso se siente
capaz de comprender cuestiones muy complicadas. El marido dice que ella lleva una semana
durmiendo poco y que no para; se levanta temprano, sale a la calle, vuelve, se cambia, vuelve
a salir, habla sin parar con cualquier persona que se encuentra y no controla lo que gasta.
Nunca le había ocurrido nada parecido y ella no acepta tener ningún problema. El
diagnóstico sería:

a) Episodio maníaco.
b) Trastorno bipolar.
c) Trastorno psicótico breve.
d) Trastorno de ansiedad generalizada.

Criterios para el episodio maníaco

A. Un período diferenciado de un estado de ánimo anormal y persistentemente elevado,


expansivo o irritable, que dura al menos 1 semana (o cualquier duración si es necesaria la
hospitalización).

B. Durante el período de alteración del estado de ánimo han persistido tres (o más) de los
siguientes síntomas (cuatro si el estado de ánimo es sólo irritable) y ha habido en un grado
significativo:

1. autoestima exagerada o grandiosidad.


2. Disminución de la necesidad de dormir (p. ej., se siente descansado tras sólo 3 horas de
sueño)

3. Más hablador de lo habitual o verborreico.

4. Fuga de ideas o experiencia subjetiva de que el pensamiento está acelerado.

5. distraibilidad (p. ej., la atención se desvía demasiado fácilmente hacia estímulos


externos banales o irrelevantes)

6. Aumento de la actividad intencionada (ya sea socialmente, en el trabajo o los estudios, o


sexualmente) o agitación psicomotora.

7. Implicación excesiva en actividades placenteras que tienen un alto potencial para


producir consecuencias graves (p. ej., enzarzarse en compras irrefrenables, indiscreciones
sexuales o inversiones económicas alocadas)

C. Los síntomas no cumplen los criterios para el episodio mixto.

D. La alteración del estado de ánimo es suficientemente grave como para provocar


deterioro laboral o de las actividades sociales habituales o de las relaciones con los demás,
o para necesitar hospitalización con el fin de prevenir los daños a uno mismo o a los demás,
o hay síntomas psicóticos.

E. Los síntomas no son debidos a los efectos fisiológicos directos de una sustancia (p.ej.
una droga, un medicamento u otro tratamiento) ni a una enfermedadmedica (p. ej.,
hipertiroidismo).

Nota: Los episodios parecidos a la manía que están claramente causados por un tratamiento
somático antidepresivo (p. ej., un medicamento, terapéutica electroconvulsiva, terapéutica
lumínica) no deben ser diagnosticados como trastorno bipolar I.
43.- Femenino De 17 años refiere que lleva un tiempo con menos apetito y durmiendo
menos ya que le cuesta poder dormirse. Es una buena estudiante, pero en la última
evaluación le han quedado cuatro asignaturas y le cuesta concentrarse en los estudios. Los
dos últimos fines de semana no ha salido con sus amigas porque no le apetecía y se siente
irritada con su familia, aunque no entiende porqué. Tiene dolores de cabeza y a veces le
viene la idea de la muerte a la imaginación aunque piensa que no lo haría por sus
sentimientos religiosos. Nunca le había sucedido algo parecido. El diagnóstico sería:

a) Trastorno depresivo mayor.

b) Trastorno ciclotímico.

c) Anorexia.

d) Episodio depresivo mayor.

Criterios para el diagnóstico del episodio depresivo mayor (DSM-IV)

A. Presencia de cinco (o más) de los siguientes síntomas durante un período de 2 semanas,


que representan un cambio respecto a la actividad previa; uno de los síntomas debe ser (1)
estado de ánimo depresivo o (2) pérdida de interés o de la capacidad para el placer.

Nota: No incluir los síntomas que son claramente debidos a enfermedad médica o las ideas
delirantes o alucinaciones no congruentes con el estado de ánimo.

1. estado de ánimo depresivo la mayor parte del día, casi cada día según lo indica el propio
sujeto (p. ej., se siente triste o vacío) o la observación realizada por otros (p. ej., llanto).
Nota: En los niños y adolescentes el estado de ánimo puede ser irritable
2. disminución acusada del interés o de la capacidad para el placer en todas o casi todas las
actividades, la mayor parte del día, casi cada día (según refiere el propio sujeto u observan
los demás)
3. pérdida importante de peso sin hacer régimen o aumento de peso (p. ej., un cambio de
más del 5 % del peso corporal en 1 mes), o pérdida o aumento del apetito casi cada día.
Nota: En niños hay que valorar el fracaso en lograr los aumentos de peso esperables
4. insomnio o hipersomnia casi cada día
5. agitación o enlentecimiento psicomotores casi cada día (observable por los demás, no
meras sensaciones de inquietud o de estar enlentecido)
6. fatiga o pérdida de energía casi cada día
7. sentimientos de inutilidad o de culpa excesivos o inapropiados (que pueden ser
delirantes) casi cada día (no los simples autorreproches o culpabilidad por el hecho de
estar enfermo)
8. disminución de la capacidad para pensar o concentrarse, o indecisión, casi cada día (ya
sea una atribución subjetiva o una observación ajena)
9. pensamientos recurrentes de muerte (no sólo temor a la muerte), ideación suicida
recurrente sin un plan específico o una tentativa de suicidio o un plan específico para
suicidarse
B. Los síntomas no cumplen los criterios para un episodio mixto.

C. Los síntomas provocan malestar clínicamente significativo o deterioro social, laboral o de


otras áreas importantes de la actividad del individuo.

D. Los síntomas no son debidos a los efectos fisiológicos directos de una sustancia (p. ej.,
una droga, un medicamento) o una enfermedad médica (p. ej., hipotiroidismo).

E. Los síntomas no se explican mejor por la presencia de un duelo (p. ej., después de la
pérdida de un ser querido), los síntomas persisten durante más de 2 meses o se
caracterizan por una acusada incapacidad funcional, preocupaciones mórbidas de inutilidad,
ideación suicida, síntomas psicóticos o enlentecimiento psicomotor.
Bibliografía:

1. DSM-IV. American Psychiatric Association. . Diagnostic and Statistical Manual of Mental


Disorders (4th Ed.). Washington, DC.

44.- Se trata de femenino de 38 años, que acude a intercosulta al servicio de medicina


interna por presentar obesidad troncular de reciente diagnóstico, amenorrea y depresión.
En la exploración física se aprecia facies redondeadas con hirsutismo moderado, TA de
160/100 mmHg y edemas en miembros inferiores. Se realizan examenes de laboratorio los
cuales arrojan los siuentes resultados: tirotropina (TSH) 0,7 mU/ml (N: 0,4-5,0), T4 libre
16.8 pmol/l (N: 9,0-23,0), cortisol libre en orina 11 mg/24h (N: 20-100), cortisol plasmático
3 mg/dl (N: 5-25) y corticotropina (ACTH) 9 pmol/l (N: inferior a 52). ¿Cuál de las
siguientes situaciones estará dando lugar a este cuadro?

a) Síndrome de Cushing ACTH dependiente.


b) Administración exógena de glucocorticoides.
c) Hipotiroidismo subclínico.
d) Enfermedad de Cushing.

Los corticosteroides (del lat. cortex, —ĭcis, corteza, y esteroide) o corticoides son una
variedad de hormonas del grupo de los esteroides (producida por la corteza de las
glándulas suprarrenales) y sus derivados.

Los corticosteroides están implicados en una variedad de mecanismos fisiológicos,


incluyendo aquellos que regulan la inflamación, el sistema inmunitario, el metabolismo de
hidratos de carbono, el catabolismo de proteínas, los niveles electrolíticos en plasma y, por
último, los que caracterizan la respuesta frente al estrés.

Estas sustancias pueden sintetizarse artificialmente y tienen aplicaciones terapéuticas,


utilizándose principalmente debido a sus propiedades antiinflamatorias e inmunosupresoras
y a sus efectos sobre el metabolismo.
Los efectos metabólicos del exceso de glucocorticoides son los siguientes:

1. Aumento de la neoglucogénesis y resistencia a la insulina; esto puede


llevar a la diabetes mellitus
2. Aumento del catabolismo proteico; esto puede llevar a la emaciación,
osteoporosis y adelgazamiento de la piel.
3. Aumento y redistribución de la grasa corporal: se produce una
obesidad de predominio central, facie de luna, tungo o acúmulo
dorsal de grasa, manteniendo extremidades relativamente delgadas.
4. Involución del tejido linfático y disminución de la respuesta
inflamatoria: se produce una disminución de la inmunidad celular y
humoral con lo que aumenta la susceptibilidad a infecciones.
5. Aumento de la secreción de ácido por el estómago lo que lleva a una
predisposición de úlcera gastroduodenal.
6. Retención de sodio y redistribución de los fluidos corporales lo que
produce edema e hipertensión arterial.
7. Función gonadal: los glucocorticoides afectan la secreción de
gonadotrofinas. En los hombres disminuye la concentración de
testosterona. En las mujeres, suprime la respuesta de LH al GnRH,
lo que lleva a una supresión de la secreción de estrógenos y
progestinas, con anovulación y amenorrea.

Todos los efectos anteriormente enunciados pueden ocurrir independientes del origen de
los glucocorticoides. La causa más común de síndrome de Cushing se debe a la
administración exógena de dosis farmacológicas de ellos con fines generalmente
antiinflamatorios e inmunosupresores.

45.- En un estudio transversal se tienen 1000 pacientes con cáncer mamario, 32 de ellas
estaban embarazadas. A partir de estos datos, se puede concluir que:

a) El embarazo es una complicación rara del cáncer mamario.


b) Si se hacen los ajustes de la edad, se puede determinar el riesgo de cáncer de mama
durante el embarazo.
c) En este estudio el 3.2 % de las pacientes con cáncer de mama estaban
embarazadas.
d) Existe asociación causal entre estar embarazada y desarrollar cáncer de mama.
Los estudios transversales solo permiten recoger información y describir la distribución de
frecuencias de las características de salud de la población y de las posibles asociaciones de
éstas con otras variables. Solo permite calcular prevalencia (32/1000X100= 3.2)

Ruiz M. A. Epidemiología Clínica, Panamericana, 1ª. Ed. 2004; pág: 198

46.- Masculino de 34 años que acude por fiebre de comienzo súbito, tos con expectoración
purulenta y dolor en el hemitórax que aumenta al respirar. La placa de tórax muestra signos
de consolidación pulmonar y un infiltrado lobular. Ud sospecha el siguiente agente causal e
indica tratamiento:

a) Chlamydia /azitromicina
b) Coxiella /vacomicina
c) S. pneumoniae / ceftriaxona
d) M. pneumoniae /penicilina

En éste caso lo mas probable es que el paciente se encuentre cursando con una
neumonía adquirida en la comunidad originada en el 60-70% de los casos por estreptococo
pneumoniae el tratamiento ya no es con penicilina debido al alto grado de resistencia a
este tratamiento, en la actualidad se utiliza cefalosporinas es decir en el presente caso
seria la respuesta CEFTRIAXONA.

1.- Fishman AP, , Fishman JA, Grippi MA, Kaisser LR, Señor RM. Pulmonary Diseases and
disorder. 3a. Edición McGraw-Hill, EUA, 2006.
2.- Fraser, R ; Neil, C; Parè, P; Diseases of the Chest, Third Edition, Editorial Elsevier,
2005.
3.- Murray and Nadels; Textbook Respiratory Medicine, Vol 1-2, Elsevier editorial, 2005.
47.- Hombre de 73 años. Llega al Servicio de urgencias por alteración del estado de alerta,
a su ingreso se realiza glucosa capilar y tiene glucosa de 780 mg/dL. Tiene antecedente de
hipertensión arterial desde hace 12 años. EF desorientado, deshidratado, TA 90/60, FC
110, peso 68 kg, estatura 1.72 cm, se coloca sonda Foley y se obtienen 40 ml de orina
turbia. Laboratorio: glucosa 810mg/dL, urea 44 mg/dL, creatinina 2 mg/dL, Na 155 mEq/L,
K 3.2 mEq/L.
El diagnóstico del paciente es:

a) Cetoacidosis
b) Hipoglucemia
c) Estado hiperosmolar hiperglucémico
d) Estado hiperosmolar hipernatrémico

El paciente prototípico en estado hiperosmolar hiperglucémico (HHS) es un anciano con DM


de tipo 2 que tiene antecedentes de varias semanas de duración con poliuria, pérdida de
peso y decremento de la ingestión oral que culminan en confusión mental, letargo o coma.
Los datos de la exploración física reflejan deshidratación profunda e hiperosmolalidad y
revelan hipotensión, taquicardia y trastorno del estado mental. Es notable la ausencia de
síntomas como náuseas, vómitos y dolor abdominal, así como de la respiración de Kussmaul
característica de la DKA. Con frecuencia el HHS es precipitado por una enfermedad
concurrente grave, como infarto del miocardio o accidente vascular cerebral. Otros
factores precipitantes frecuentes son sepsis, neumonía y otras infecciones, y es
indispensable investigar su presencia. Asimismo, pueden contribuir también al desarrollo de
este trastorno padecimientos debilitantes (accidente vascular cerebral previo o demencia)
y situaciones sociales que obstaculizan la ingestión de agua.

Fisiopatología
El déficit relativo de insulina y el aporte insuficiente de líquidos son las causas que
subyacen al HHS. El déficit de insulina aumenta la producción hepática de glucosa por el
músculo esquelético (véase lo tratado anteriormente en la DKA). La hiperglucemia induce
una diuresis osmótica que provoca disminución del volumen intravascular, que se exacerba
todavía más por el aporte insuficiente de líquidos. No se comprende por completo la
ausencia de cetosis en el HHS. Probablemente, el déficit insulínico es sólo relativo y menos
grave que en el caso de la DKA. En algunos estudios se han encontrado concentraciones más
bajas de hormonas contrarreguladoras y de ácidos grasos libres en el HHS que en la DKA.
También es posible que el hígado sea menos capaz de sintetizar cuerpos cetónicos, o que el
cociente insulina/glucagon no favorezca la cetogénesis.

Bibliografía:
1. Lobesio C: Síndrome hiperglucémico hiperosmolar. Texto de medicina
Intensiva. 5ta Ed. 2000; 95:1050-54.
2. Jiménez Murillo J, Barca C de la, Romero M, Montero F J. Coma hiperosmolar
en Medicina de urgencia. En: Jiménez Murillo J, Montero F J. Guía diagnóstica y
protocolo de actuación.2da. Ed. Madrid: Harcourt, 1999: 403-5.
3. Remuñan C, Álvarez JL: Coma Hiperosmolar. Rev Cubana Med 2001;
40(3):189-94.
4. Shoemaker LW: Critical Care Medicine. 3era Ed. Text Book of Critical Care.
Philadelphia Saunders. 2000; 71:783-85.
5. Soler Morejón C: Coma hiperosmolar en: Temas Actualizados en: Rev Cubana
Med 1999; 38(3):183-7.
Villanueva V. Complicaciones agudas de la diabetes mellitas. Rev.

48.- Mujer de 27 años. Acude a consulta por hirsutismo e irregularidades menstruales.


Antecedente familiar de diabetes mellitus en madre y una hermana. Su padecimiento
empezó desde la adolescencia con ritmo menstrual de 40 – 60 x 5 -7 días. EF: peso 87 kg,
estatura 1.54 m, TA 130/80, FC 72x’, acantosis nigricans en cuello y axilas, no galactorrea,
perímetro abdominal de 112 cm. Vello supralabial, en patilla, en línea media infraumbilical y
en cara interna de muslos. Laboratorio: glucosa 104 mg/dL, resto de química sanguínea
normal, Bh normal. Prolactina 16 ng/dL, testosterona: 78 ng/dL (25 a 50 ng/dl). US
El diagnóstico probable es:

a) Hiperprolactinemia por microprolactinoma


b) Síndrome de ovarios poliquísticos por resistencia a la insulina
c) Virilización por tumor productor de testosterona
d) Diabetes mellitus secundaria a la obesidad

El síndrome de ovarios poliquísticos (SOPQ) afecta aproximadamente a un 4% de mujeres


en edad reproductiva y se caracteriza por anovulación crónica e hiperandrogenismo. Es la
causa más común de infertilidad en mujeres.
Se caracteriza clínicamente por acné, alopecia, hirsutismo, irregularidades menstruales e
infertilidad.
Los hallazgos de laboratorio más frecuentes son: aumento de la hormona luteinizante (LH),
aumento de la relación LH/FSH (hormona folículoestimulante), aumento de andrógenos
(tanto ováricos como adrenales) y de estrógenos circulantes. Otros hallazgos de
laboratorio habituales son una prueba tolerancia oral a la glucosa anormal y alteraciones en
el perfil lipídico.
Todo esto junto con las imágenes ecocardiográficas características definen al síndrome.
La terapéutica permite dos grandes enfoques que pueden superponerse: la corrección de
las manifestaciones de hiperandrogenismo y el tratamiento de las alteraciones del eje
reproductivo (anovulación, esterilidad).
Los antiandrógenos están fundamentalmente indicados para tratar los síntomas virilizantes.
Las alternativas para inducir la ovulación son numerosas: al citrato de clomifeno y a la
antigua resección en cuña se agregan las gonadotrofinas humanas, pulsos de GnRH (hormona
liberadora de gonadotrofinas), medidas o fármacos para modificar los niveles de insulina, y
finalmente técnicas quirúrgicas endoscópicas para reducir la masa ovárica.

Revista de Posgrado de la VIa Cátedra de Medicina - N° 125 – Marzo 2003


Pág. 37-40
SINDROME DE OVARIOS POLIQUISTICOS
Dra. Sandra Beneyto, Dra. María Andrea Ferreyra, Dr. Andrés Galfrascoli,
Dr. Andrés González, Dra. Susana Sosa

49.- Masculino de 45 años con hipercalcemia asintomática. El resto de sus exámenes de


laboratorio muestran elevación de parathormona, baja en fósforo, elevación de cloro y BUN
y creatinina normales. El calcio urinario está elevado. ¿Cuál es la etiología más probable?

a) Mileoma múltiple
b) Hiperparatiroidismo primario
c) Hipervitaminosis
d) Sarcoidosis

Al grupo de síndromes que tienen como característica común la secreción excesiva y no


controlada de hormona para-tohormona (HPT) por una o más paratiroides funcionantes se
le conoce como hiperparatiroidismo primario (HPP). La causa más frecuente es el adenoma y
le sigue en frecuencia la hiperplasia.
El cuadro clínico del HPP afecta principalmente al sistema óseo, al riñón y al sistema
gastrointestinal. El cuadro clínico se caracteriza por debilidad muscular, fatigabilidad fácil,
manifestaciones gastrointestinales y depresión. Cuando hay afección renal el paciente
presenta hipercalciuria (25%), litiasis renal recidivante (25%) y/o nefrocalcinosis (20%).
Cuando hay resorción subperióstica, tumores pardos en huesos largos o en el maxilar
inferior y a nivel del cráneo, las lesiones osteolíticas le dan el aspecto de “sal y pimienta”.
En los exámenes de laboratorio hay hipercalcemia, hipofosfatemia, fosfatasa alcalina
elevada (fracción ósea), elevación de HPT (90%), osteocalcina y desoxipiridinolinas; así
como calciuria e hiperfosfaturia. Cuando hay afección del sistema gastrointestinal hay
úlceras y hemorragias.
Por lo tanto, como podemos observar, se trata de una paciente con HPP clásico con afección
del sistema óseo y del riñón, sin manifestaciones gastrointestinales aparentemente, el cual
al someterse a tratamiento quirúrgico y médico adecuados hubo mejoría inmediata que ha
continuado hasta la fecha actual.
Hiperparatiroidismo primario
Ma. Esther Gutiérrez Díaz Ceballos,1 Héctor A. Rodríguez Martínez,1 Evelyn M. Torres
Acosta,1 Humberto Cruz Ortiz1
1 Unidad de Patología y Servicio de Endocrinología del Hospital General de México y de la
Facultad de Medicina, UNAM

50.- Un hombre de 58 años de edad con una historia de fumar 20 cajetillas/año, está
siendo evaluado para la colocación de un bypass coronario. ¿Cuál de las siguientes pruebas
preoperatorias es la mejor para valorar función respiratoria en este paciente?

a) FVC
b) PaCO2
c) PaO2
d) FEV1/FVC

La relación FEV1/ FVC le proporciona la capacidad de las funciones pulmonares del paciente
y la FEV1 le indica si hay una obstrucción de la vía aérea, la FVC Y VR aisladas le indican si
hay una obstrucción únicamente, la PaCO2 Y PaO2 solo le indican si hay retención de Co2 y
la oxemia del paciente.

1.- Fishman AP, , Fishman JA, Grippi MA, Kaisser LR, Señor RM. Pulmonary Diseases and
disorder. 3a. Edición McGraw-Hill, EUA, 2006.
2.- Fraser, R ; Neil, C; Parè, P; Diseases of the Chest, Third Edition, Editorial Elsevier,
2005.
3.- Murray and Nadels; Textbook Respiratory Medicine, Vol 1-2, Elsevier editorial, 2005.
51.- Un varón de 65 años de edad con antecedente de bronquiectasia acude al
departamento de urgencias con hemoptisis. Refiere tos acentuada y producción de esputo
en la última semana acompañados de febrículas. A menudo su esputo se tiñe de estrías de
sangre, pero el último día observó que está tosiendo sangre coagulada en cantidades
equivalentes a una cucharada, con un total aproximado de una taza en 24hrs. La exploración
física muestra signos vitales normales, con saturación de oxígeno de 98% en aire ambiente.
El paciente manifiesta disnea leve y sibilancias exhalatorias difusas. La radiografía de
tórax, además de mostrar bronquiectasias, es normal.
¿Cuál es el tratamiento inmediato más apropiado para esta hemoptisis?

a) Broncoscopia
b) Embolización de la arteria bronquial
c) TAC de tórax
d) Resección quirúrgica

En este caso se debe realizarle una broncoscopia con la finalidad de localizar el sitio de
sangrado y detener el mismo por compresión con un cateter de fogarty, la TAC no tiene
indicacion como tratamiento.

TABLA 1. Indicaciones de la broncoscopia diagnóstica


Síntomas clínicos no explicados
Tos persistente
Disnea / sibilantes no justificados
Hemoptisis
Disfonía
Asma unilateral, de comienzo súbito
Síndrome de vena cava
Parálisis diafragmática
Aclaración de una sospecha diagnóstica
Alteraciones radiológicas
Investigar el origen de una citología de esputo positiva
Estadiaje del carcinoma broncogénico
Sospecha de fístula traqueoesofágica
Evaluación de la vía aérea tras un traumatismo torácico
Evaluación de la vía aérea en quemados
Toma de muestras en una enfermedad intersticial
Estudio microbiológico de infecciones respiratorias
Evaluación de la respuesta al tratamiento del cáncer de pulmón
BRONCOSCOPIA DIAGNÓSTICA Y TERAPÉUTICA
Monografías NEUMOMADRID
Prudencio Díaz-Agero Álvarez
Javier Flandes Aldeyturriaga
VOLUMEN X / 2007
52.- Una mujer de 52 años de edad presenta neumonía adquirida en la comunidad
complicada con derrame pleural. Se lleva a cabo toracocentesis con los siguientes
resultados:
Aspecto Viscoso, opaco
Ph 7.11
Proteínas 5.8g/100ml
LDH 285UI/l
Glucosa66mg/dl
Leucocitos 3 800/mm3
Eritrocitos 24 000/mm3
PMN 93%

Muchos PMN; no se observan microorganismos.

Se envían muestras para cultivos bacterianos pero todavía no se cuenta con los resultados.
¿Cuál característica del líquido pleural es más indicativa de que la paciente requerirá
pleurostomía con sonda?

a) La presencia de más de 90% de PMN


b) Glucosa de menos de 100mg/dl
c) pH menor de 7.2
d) LDH con cifras de más de 2/3 del límite superior normal para suero

En éste caso lo mas probable es que el paciente se encuentre cursando con empiema el cual
en el inicio del mismo se valora con una gasometría del líquido pleural el cual si indica un ph
menor a 7.2 es necesario colocar sonda endotorácica para drenarlo, en algunas
publicaciones se menciona que la glucosa puede ser de utilidad pero cuando es menor a 40
mg, por lo tanto la única respuesta es la del pH menor de 7.2.

El pH del LP sirve para el diagnóstico etiológico de los exudados. Hay que medir el pH y la
PaCO2 en sangre para descartar acidosis sanguínea. En general el LP con pH bajo tiene
glucosa baja y LDH alta. El pH está bajo, incluso por debajo de 7.20 en derrames pleurales
paraneumónicos complicados y cuando es menor de 7,10 es indicación de drenaje de la
cavidad pleural. Una excepción son las infecciones por Proteus mirabilis que producen
aumento de la amoniemia. El pH bajo en las neoplasias está relacionado con el número de
células y con el pronóstico (menor supervivencia y peor respuesta a la pleurodesis).
También está disminuido en la artritis reumatoide donde puede ser menor de 7.20 mientras
que en el LES puede estar normal. En el hemotórax, también está bajo debido al consumo
de glucosa por los hematíes con la consiguiente producción de CO2 y disminución del pH. Los
urinotórax también pueden tener pH bajo. En los trasudados, el pH del líquido pleural puede
estar más alto que en la sangre debido al transporte activo del CO3H de la sangre al
espacio pleural.
Causas de derrame pleural con pH inferior a 7'30
Derrame paraneumónico complicado y empiema
Tuberculosis
Neoplasias
Rotura esofágica
Artritis reumatoide

Referencia:
1.- Fishman AP, , Fishman JA, Grippi MA, Kaisser LR, Señor RM. Pulmonary Diseases and
disorder. 3a. Edición McGraw-Hill, EUA, 2006.

53.- Masculino de 49 años de edad que se queja de dolor y rigidez en las articulaciones de
3 meses de evolución. Ambos pies y ambas manos están calientes y las articulaciones
edematizadas. ¿Cuál de los siguientes sugiere el diagnóstico de artritis reumatoide?

a) Rigidez matutina que dura 1 hr


b) Adormecimiento y palidez de los dedos al ser expuestos al frío
c) Afección simétrica de las articulaciones distales interfalángicas
d) Líquido sinovial con alta viscosidad y 30 mil linfocitos /mm3

Los criterios diagnósticos establecidos por el Colegio Americano de Reumatología, incluyen


alguno de los siguientes:
Presencia de artritis de más de 6 semanas de duración
Rigidez articular matutina prolongada (+ de una hora)
Presencia de nódulos característicos en la piel
Erosiones articulares visibles por radiología
Positividad analítica de un anticuerpo que se conoce como factor reumatoide, si bien
el 25% de los pacientes con AR nunca desarrollarán este factor y, dicho anticuerpo, puede
aparecer en sujetos que no tienen AR.

BIBLIOGRAFÍA RECOMENDADA:

1) Klippel JH, Stone JH, Crofford LJ, White PH, editors. Primer on the rheumatic
diseases. 13th ed. New York: Springer-The Arthritis Foundation; 2008.
2) Martínez-Elizondo P, editor. Introducción a la Reumatología. 4a ed. México: Colegio
Mexicano de Reumatología A.C./Intersistemas S.A. de C.V.; 2008.

3) Firestein GS, Budd RC, Harris ED Jr, McInnes IB, Ruddy S, Sergent JS, editors.
Kelley’s Textbook of Rheumatology. 8th ed. Philadelphia: Saunders Elsevier; 2009.

54.- Un estudiante universitario de 20 años de edad acude a consulta debido a tos seca,
fiebre, cefalea y dolor muscular durante las últimas 2 semanas. Refiere que sus
compañeros de casa han desarrollado síntomas similares. Niega el uso de drogas ilícitas y
no es homosexual. Su temperatura es de 38.2°C, FC 90lpm, FR 18x’. Se auscultan murmullo
vesicular bilateral. Una RX de tórax muestra opacidades intersticiales multifocales.
Presenta leucocitosis y un test de aglutinina frío +. ¿Cual de los siguientes es el patógeno
más probable?

a) Bacterias anaeróbicas
b) Mycoplasma pneumoniae
c) Pneumocystis carinii
d) Streptococcus pneumoniae

En el caso antes mencionado, se trata de una neumonía llamada atípica que se presenta por
norma en pacientes jóvenes y que han estado en lugares de convivencia con otras personas,
la imagen radiográfica de esta neumonía es de una neuropatía intersticial como se indica.

Referencias:

1.- Fishman AP, , Fishman JA, Grippi MA, Kaisser LR, Señor RM. Pulmonary Diseases and
disorder. 3a. Edición McGraw-Hill, EUA, 2006.
2.- Fraser, R ; Neil, C; Parè, P; Diseases of the Chest, Third Edition, Editorial Elsevier,
2005.
3.- Murray and Nadels; Textbook Respiratory Medicine, Vol 1-2, Elsevier editorial, 2005.
55.- Un hombre de 55 años de edad presenta una historia de 2 días de hemoptisis. Reporta
un inicio agudo de 8 episodios de tos con sangre fresca (aprox una cucharadita de sangre
por cada episodio). No reporta otros síntomas, excepto por una tos productiva de 5-10ml
de esputo cada mañana. Tiene una historia de EPOC, para lo cual toma broncodilatadores.
Ha fumado 30 cigarrillos diariamente por los últimos 30 años. El examen físico es normal, y
una RX limpia. Cual de los siguientes es la causa más probable de la hemoptisis?

a) Bronquiectasias
b) Carcinoma broncogénico
c) Bronquitis crónica
d) Tb pulmonar

La EPOC puede presentar episodios de hemoptisis con placa radiográfica normal, aunque no
es muy frecuente; las otras causas como TB pulmonar y Cáncer pulmonar así como las
bronquiectasias presentan lesiones en la radiografía que hace sospechar en estos
diagnósticos y en el presente caso no se mencionan lesiones en la Rx, la deficiencia de alfa 1
provoca la presencia de enfisema pulmonar la cual igualmente debería tener una placa con
alteraciones.

1.- Fishman AP, , Fishman JA, Grippi MA, Kaisser LR, Señor RM. Pulmonary Diseases and
disorder. 3a. Edición McGraw-Hill, EUA, 2006.
2.- Fraser, R ; Neil, C; Parè, P; Diseases of the Chest, Third Edition, Editorial Elsevier,
2005.
3.- Murray and Nadels; Textbook Respiratory Medicine, Vol 1-2, Elsevier editorial, 2005.

56. - A 38-year-old man comes to the physician because of slowly progressive visual
problems that make him “bump into objects” on both sides. He also reports that, while
driving, he has trouble switching lanes because he needs to turn his head all the way
backward to look for other cars. Ocular examination shows bitemporal field loss with
preserved visual acuity.Examination of the fundus is unremarkable. Which of the following
is the most likely diagnosis?

a) Pituitary adenoma
b) Occipital lobe meningioma
c) Optic neuritis
d) Retinal detachment
PRESENTACIÓN CLÍNICA

Los tumores hipofisarios se reconocen clínicamente por uno o más de tres patrones de
presentación muy constantes:

- Síntomas de hipersecreción hipofisiaria

- Síntomas de hiposecreción hipofisaria

- Síntomas neurológicos

El tercer patrón de presentación es el dominado por los síntomas neurológicos, aislados o


coexistentes con una o varias de las alteraciones endocrinológicas antes descritas. Como se
ha dicho, una masa hipofisaria progresivamente creciente generará una constelación de
signos y síntomas neurológicos que dependerán de la trayectoria del crecimiento y de las
estructuras nerviosas vecinas que resulten alteradas. Los síntomas y signos a encontrar
son:

a. Cefaleas: Pueden ser un signo precoz y se atribuyen al estiramiento de la duramadre


que recubre la hipófisis o del diafragma de la silla turca. Está presente en el 75% de los
casos y se localiza más frecuentemente en la región frontal y orbital.

b. Pérdida de visión: Debido a la compresión de los nervios ópticos o del quiasma óptico. El
patrón clásico de pérdida visual es una hemianopsia bitemporal a menudo asociada con
disminución de la agudeza visual. Posteriormente puede haber una ceguera completa de uno
o ambos ojos.

c. Manifestaciones hipotalámicas: Debido a la compresión del hipotálamo por grandes


adenomas hipofisarios que provocan:

- Alteraciones del sueño

- Alteraciones de la atención

- Alteraciones de la conducta

- Alteración de la alimentación

- Alteración de las emociones

d. Hidrocefalia obstructiva: Debido a la infiltración de las láminas terminales que hace


que el tumor penetre en la región del III ventrículo, ocasionando obstrucción a la salida de
LCR.

e. Compromiso de nervios craneales. Debido a la extensión lateral del tumor hacia la


región del seno cavernoso, por donde transcurren los nervios craneanos. Así las
manifestaciones más características son:

- Ptosis: Por compromiso del III nervio craneal (motor ocular común).
- Dolor o alteraciones sensitivas faciales. Por compromisos de las ramas oftálmica y maxilar
superior del Nervio Trigémino.

- Diplopia: Por compromiso del III, IV y VI nervios craneales.

Bibliografía:

1. Kovacs K, Horvath E, Vidal S: Classification of pituitary adenomas. J Neurooncol 54


(2): 121-7, 2001.

2. Ironside JW: Best Practice No 172: pituitary gland pathology. J Clin Pathol 56 (8):
561-8, 2003.

3. Scheithauer BW, Kovacs KT, Laws ER Jr, et al.: Pathology of invasive pituitary
tumors with special reference to functional classification. J Neurosurg 65 (6):
733-44, 1986.

4. Ezzat S, Asa SL, Couldwell WT, et al.: The prevalence of pituitary adenomas: a
systematic review. Cancer 101 (3): 613-9, 2004.

57. - A 27-year-old woman has been sad for the last two weeks. She is fatigued a has a
hard time concentrating at work. Just a few weeks earlier she was energetic an
enthusiastic, and was able to work 10-12 hours a day with little sleep and go dancing at
night. Her husband wants a divorce because he is tired of “these constant ups and downs”.
The most accurate diagnosis is:

a) Borderline personality disorder


b) Seasonal mood disorder
c) Dissociative identity disorder
d) Cyclothymic disorder
Criterios para el diagnóstico de
F34.0 Trastorno ciclotímico (301.13)

A. Presencia, durante al menos 2 años, de numerosos períodos de síntomas hipomaníacos y


numerosos períodos de síntomas depresivo que no cumplen los criterios para un episodio
depresivo mayor.

Nota: En los niños y adolescentes la duración debe ser de al menos 1 año.

B. Durante el período de más de 2 años (1 año en niños y adolescentes) la persona no ha


dejado de presentar los síntomas del Criterio A durante un tiempo superior a los 2 meses.

C. Durante los primeros 2 años de la alteración no se ha presentado ningún episodio


depresivo mayor, episodio maníaco o episodio mixto.

Nota: Después de los 2 años iniciales del trastorno ciclotímico (1 año en los niños y
adolescentes), puede haber episodios maníacos o mixtos superpuestos al trastorno
ciclotímico (en cuyo caso se diagnostican ambos trastornos, el ciclotímico y el trastorno
bipolar I) o episodios depresivos mayores (en cuyo caso se diagnostican ambos trastornos,
el ciclotímico y el trastorno bipolar II).

D. Los síntomas del Criterio A no se explican mejor por la presencia de un trastorno


esquizoafectivo y no están superpuestos a una esquizofrenia, un trastorno
esquizofreniforme, un trastorno delirante o un trastorno psicótico no especificado.

E. Los síntomas no son debidos a los efectos fisiológicos directos de una sustancia (p. ej.,
una droga, un medicamento) o a una enfermedad médica (p. ej., hipertiroidismo).

F. Los síntomas provocan malestar clínicamente significativo o deterioro social, laboral o de


otras áreas importantes de la actividad del individuo.
58.- Masculino de 32 años que inició con disfonía y disfagia, es referido al servicio de
endocrinología donde es confirmado el diagnostico de nódulo tiroideo único, ¿El estudio
recomendado es?

a) Biopsia por aspiración con aguja fina


b) Gammagrama tiroideo
c) Tomografía computada de cuello
d) Repetir Pruebas de función tiroidea

El nódulo tiroideo solitario se define como el crecimiento localizado de la glándula tiroides,


usualmente es benigno, la prevalencia es del 4 al 7% en la población general. Aunque el
cáncer tiroideo es el tumor endocrino más común sólo representa el 1% de todos los
cánceres y 5% de todos los nódulos tiroideos. El estudio clínico diagnóstico y terapéutico
debe iniciarse con historia clínica completa, exploración física y exámenes de laboratorio
que incluyan un perfil tiroideo para evaluar función de la glándula. La citología tiroidea
por aspiración (CTA) es el principal procedimiento diagnóstico en los pacientes con
nódulo tiroideo solitario, por ser capaz de diferenciar lesiones benignas de las
malignas, sus principales ventajas son: segura, reduce costos de atención médica,
selecciona mejor los pacientes que serán sometidos a tratamiento quirúrgico y se
realiza en pacientes ambulatorios.

Referencias:

1. Torres AP, Hernández SE, Caracas PN, Serrano GI et al Diagnóstico y tratamiento


del nódulo tiroideo. Rev Edocrinol Nutr 2000; 8 (3): 87-93.

59.- Se ingresa al hospital a masculino de 66 años con diagnóstico de Neumonía


Neumocócica. Es tratado con Ceftriaxona durante su estancia evoluciona
satisfactoriamente. A los 8 días de su estancia inicia con fiebre, diarrea líquida maloliente,
dolor abdominal tipo cólico y leucocitosis. ¿Cuál, de los siguientes, le parece el diagnóstico
más probable?:

a) Sabmonellosis.
b) Infección por Clostridium Difficile.
c) Amebiasis intestinal
d) Giardiasis.
El Clostridium difficile es un bacilo anaerobio formador de esporas subterminales grandes,
de forma oval, localizadas en el intestino. Posee dos enterotoxinas que producen amplios
daños en las células del intestino y así ocasionar diarrea. Los pacientes pueden presentar un
amplio espectro de la enfermedad, que va de la diarrea simple no complicada asociada al uso
de antibióticos, hasta la colitis seudomembranosa asociada al uso de éstos, potencialmente
mortal. El C. difficile no tiene serotipos. Existen cepas patógenas y no patógenas. Las
últimas producen cantidades variables de toxina A (enterotoxina) y B (citotoxina). Los
tratamientos con antibióticos de amplio espectro eliminan gran cantidad de la flora normal,
permitiendo sobrecrecimiento del C. difficile patógeno. No existen defensas inmunológicas
definidas del huésped. El estándar de oro para el diagnóstico de laboratorio es la prueba de
citotoxina en heces. El tratamiento puede basarse en metronidazol o vancomicina, aunque
suelen ocurrir recidivas.

Cuadro II. Opciones de tratamiento con


antimicrobianos, tasa de respuesta y recidiva para la diarrea y colitis aguda
por Clostridium difficile.
Medicamento* Tasa de respuesta
Tasa de recidiva
Metronidazol: 98%
7%
Adultos: 250 mg c/6horas
Niños: 40 mg / kg / día
Vancomicina: 96%
18%
Adultos:125 mg c/6 horas
Niños: 10 mg/kg/ c/6 horas
Bacitracina: 25,000 U c/6 horas 83%
34%
Colestiramina (4 g c/6 horas) 68%
Desconocida

Tratamiento de 10 días.

Indicaciones de cirugía
La más importante indicación de la cirugía es el agravamiento de la situación clínica del
paciente, a pesar de estar recibiendo el tratamiento indicado.
Las bases clínicas son la insuficiencia orgánica, peritonitis (posible perforación) y colitis
progresiva.
La enfermedad progresiva puede documentarse por medio de tomografía computarizada; la
pared colónica muy engrosada y la presencia de ascitis son indicativos de un pobre
pronóstico.
Gaceta Médica de México
Volumen Volume 138
Número Number 1 Enero-Febrero
January-February 2002
Artículo:
Infección por Clostridium difficile

60.- Femenino fumadora de 2 cajetillas diarias de tabaco, con antecedentes de DM tipo II


descontrolada, así como HTAS, actualmente portadora de úlcera gástrica ¿Cuál de las
siguientes es la indicación quirúrgica?

a) Sangrado de tubo digestivo alto con inestabilidad hemodinámica.


b) Menor de 60 años.
c) Biopsia negativa a tumor maligno.
d) La úlcera no cicatriza después de 12 semanas de tratamiento bien llevado por el
paciente.

Úlcera Péptica

50-75 % de casos.

La hemorragia por úlcera duodenal es 4 veces más frecuente que la úlcera gástrica, pero
ambas tienen la misma tendencia a la hemorragia.

La Hemorragia Masiva tiene una frecuencia del 10-15% y generalmente es producida por
horadación por el proceso inflamatorio de la arteria regional.

* Localización:

a) Bulbo duodenal.
b) Curvatura menor.
c) Zona prepilórica.

De ellas, la curvatura menor hace las hemorragias masivas más frecuentes, pero la
duodenal, en general, es la más frecuente.

La relación entre la localización de la úlcera y la gravedad de la hemorragia se debe a la


lesión de las arterias principales, de la curvatura menor, la coronaria estomáquica, la
pilórica y la gastroduodenal.

TRATAMIENTO DE LA HEMORRAGIA DIGESTIVA ALTA


Es una emergencia Médico-Quirúrgica, de gran importancia y frecuencia, cuya morbilidad y
mortalidad están influenciadas por un manejo oportuno, coherente, en el que
necesariamente concurren varios especialistas, en lo posible organizados en equipo.

Es evidente que el manejo en equipo y la identificación de los pacientes de alto riesgo se


convierten en las armas más importantes en el manejo actual de esta patología.

TRATAMIENTO QUIRURGICO

El momento de la indicación y la técnica a realizar son los pilares principales de una desición
correcta. Las técnicas a realizar serán de acuerdo a la experiencia del cirujano y el riesgo
quirúrgico del enfermo. Tales como: Vagotomía troncular piloroplastía, es más rápida y
menos traumática, elimina factor vagal, baja mortalidad postoperatoria, pero alto índice de
recidiva del sangrado.

Vagotomía más Gastrectomía 4/5 elimina todo el factor sangrante, usada en fracaso de
otras técnicas.

Gastrectomía total: en casos especiales. Estado general del paciente, edad, obesidad,
repetición de sangrado indican la técnica a usar.

En Úlcera Duodenal la Vagotomía más piloroplastía y sutura de la úlcera es lo más adecuado.

En Úlcera Gástrica, Gastrectomía y Sutura de la Úlcera: Gastrectomía subtotal Bilroht I,


resección en cuña de úlcera.

Várices esofágicas: sonda de Sengstaken o de Linton.

Técnica de desconección portoacigos o Tanner modificada.

Lectura recomendada:
Yamada Manual de gastroenterologia pagina 337.
61.- Masculino de 70 años de edad, que se queja de debilidad muscular. Se aprecia
heliotropo con enema y eritema en su tórax superior, cuello y cara. ¿Qué es lo más probable
que encuentre a la exploración?

a) Debilidad muscular proximal


b) Ataxia
c) Hiperreflexia de tendones
d) Inflamación de pequeñas articulaciones

Son un grupo heterogéneo de enfermedades que se caracterizan por debilidad muscular


secundaria a la inflamación del músculo estriado.

EPIDEMIOLOGÍA

• INCIDENCIA: 5 casos nuevos por millón / año


• PREVALENCIA: 10 - 60 casos por millón de habitantes. Frecuencia baja.
• 2 PICOS DE EDAD: 5-15 años 40-60 años
• FRECUENCIA MUJER : HOMBRE 2:1
• RAZA NEGRA > ORIENTALES (4:1)

La polimiositis respeta la piel, mientras que la dermatomiositis presentará alteraciones


cutáneas características acompañando a la afectación muscular
Cuadro Clínico:
- Alteraciones musculares. Vienen marcadas por la presencia de debilidad muscular aguda
o subaguda (generalmente de inicio insidioso), simétrica y difusa, con preferencia por
musculatura proximal de extremidades (cintura pélvica y escapular), tronco y cuello. En la
mayoría de los casos es indoloro. Con el tiempo, desarrollan atrofia, contracturas y
disminución de los reflejos.
- Alteraciones cutáneas. La más frecuente en la DM es una erupción cutánea
eritematoviolácea que afecta a cuello, cara y tórax. Es característico también, el eritema
heliotropo (en párpados), que puede extenderse a otras zonas fotoexpuestas), las pápulas
de Gottron (localizadas en los nudillos), telangiectasias periungueales, a veces ulceración
dérmica y calcinosis (fundamentalmente en la DM infantil).
- Articulares. Artralgias, artritis transitorias, no erosivas, con tendencia a la simetría.
- Otras. Afectación cardiaca variable (alteración ECG, arritmia, miocarditis), pulmonar
(fibrosis intersticial asociada con anti Jo-1), renal (muy rara), fenómeno de Raynaud.

Diagnóstico:
- Analítica: aumento de VSG y de enzimas musculares, (CPK, aldolasa, GOT, GPT, LDH). La
CPK es la más sensible y la que guarda una mejor correlación clínica con la actividad de la
enfermedad y la valoración de recaídas. El FR es + en 20% y
ANA es + en 10-30%. Si la destrucción muscular es intensa, puede producir mioglobinuria.
- Destacan anticuerpos: • anti-Jo1: en casos de PM asociado a neumonitis intersticial
(síndrome antisintetasa-miosistis, fibrosis pulmonar, artritis no erosiva y fenómeno de
Raynaud).
• anti-PM1 o PM-Scl: asociación con esclerodermia.
• anti-Mi, en DM.
• antimioglobina.

BIBLIOGRAFÍA RECOMENDADA:

• Klippel JH, Stone JH, Crofford LJ, White PH, editors. Primer on the rheumatic
diseases. 13th ed. New York: Springer-The Arthritis Foundation; 2008.

• Martínez-Elizondo P, editor. Introducción a la Reumatología. 4a ed. México: Colegio


Mexicano de Reumatología A.C./Intersistemas S.A. de C.V.; 2008.

• Firestein GS, Budd RC, Harris ED Jr, McInnes IB, Ruddy S, Sergent JS, editors.
Kelley’s Textbook of Rheumatology. 8th ed. Philadelphia: Saunders Elsevier; 2009.
62.- Una mujer de 26 años consulta por un episodio de hemiparesia izquierda sugerente de
ictus. Entre sus antecedentes refiere un hábito tabáquico, no se ha documentado
hipertensión ni hiperglucemia, ha tenido un episodio previo de amaurosis fugax y dos
episodios de tromboflebitis en extremidades inferiores. No refiere antecedentes
quirúrgicos ni ingesta de medicación. Ha tenido tres abortos espontáneos. El hemograma y
el estudio de coagulación son normales. La función hepática y renal y los electrólitos son
normales. El colesterol total es de 260 mg/dl (normal <240) y los triglicéridos de 160 mg/dl
(normal <150). La TC en la fase aguda no aporta datos significativos. ¿Cuál sería el
diagnóstico más probable y conducta a seguir?

a) El cuadro corresponde a síndrome antifosfolípido. Solicitaría una determinación de


anticuerpos anticardiolipina.
b) Se trata de una endocarditis infecciosa a partir de una tromboflebitis séptica.
Iniciaría tratamiento antibiótico empírico en espera de los cultivos.
c) El cuadro corresponde a un accidente aterotrombótico en una paciente con una
hiperlipemia familiar. El origen más probable es la carótida. Solicitaría un estudio de
troncos supraaórticos. Iniciaría tratamiento hipolipemiante.
d) Dada la edad, se trata de una enfermedad desmielinizante. Solicitaría una resonancia
magnética cerebral.

El Síndrome Antifosfolipídico es una entidad adquirida y multisistémica, caracterizada por


hipercoagulación, en la que las manifestaciones cutáneas, en un alto porcentaje, permiten
sospechar su diagnóstico; éstas se caracterizan por livedo reticularis y los diferentes
grados de necrosis cutánea dependiendo de los vasos afectados, además de otros hallazgos
importantes como son la trombocitopenia y los antecedentes de abortos. Los marcadores
serológicos son los anticuerpos antifosfolipídicos que corresponden al anticoagulante lúpico
y a la anticardiolipina.

SÍNDROME ANTIFOSFOLIPÍDICO
CRITERIO MAYOR
- Trombosis arterial
- Trombosis venosa
- Aborto recurrente-muerte intrauterina
- Trombocitopenia
El diagnóstico se establece con cualquiera de los criterios mayores y el hallazgo serológico
de anticoagulante lúpico o títulos altos de anticuerpos anticardiolipina (1,4,9).
Periódicamente se realizan reuniones de consenso para unificar criterios de diagnóstico y
tratamiento en el SAF.
El SAF puede clasificarse como primario o idiopático cuando no se demuestra una causa
subyacente y secundario a múltiples causas, de las cuales la más importante es el LES en el
grupo de enfermedades inmunológicas.
Referencias:

1. Gibson GE, Su D, Pittelkow MR. Antiphospholipid syndrome and the skin. J Am Acad
Dermatol. 1997; 36:970-82.
2. Griffiths MH, Papadaki L, Neild GH. The renal pathology of primary antiphospholipid
syndrome: a distinctive form of endothelial injury. QJ Med. 2000; 91:457-67.
3. Quintero del Río AI. Antiphospholipid antibodies in pediatrics. Current Reumatol Rep.
2002; 4(5):387-91.
4. Nahass GT. Antiphospholipid antibodies and the antiphospholipid antibody Syndrome. J
Am Acad Dermatol. 1997; 36:149-68.
5. Piette WW. Antiphospholipid syndrome: the problems and the promise. Br J Dermatol.
2000: 142:1079-83.
6. Gezer S. Antiphopholipid syndrome. Dis Mon. 2003; 49(12):696-741.
7. Nash MJ, Camilleri RS, Kunka S, McKie IJ, Machin SJ, Cohen H. The anticardio- lipin
assay is required for sensitive screening for antiphopholipid antibodies. J Thromb
Haemost. 2004; 2(7):1071-3.

63.- Masculino de 25 años, que 10 días después de acudir a una despedida de soltero,
comienza con inflamación de rodilla derecha y de ambos tobillos, conjuntivitis bilateral,
aftas orales y erosiones superficiales no dolorosas en el glande.
El diagnóstico más probable es:

a) Enfermedad de Still.
b) Infección gonocócica.
c) Infección por Staphylococcus Aureus.
d) Enfermedad de Reiter.

Proceso inflamatorio estéril de la membrana sinovial, precedido o precipitado por una


infección que ocurre fuera de la articulación.

DIAGNOSTICO:
• HISTORIA
• SÍNTOMAS GENERALES
• MANIFESTACIONES MÚSCULO-ESQUELÉTICAS
• Artralgias, artritis aditiva o migratoria.
• Monoartritis u oligoartritis asimétrica.
• Articulaciones grandes que sostienen peso: Rodillas, tobillos y caderas.
• Dactilitis o dedos en “salchicha”
• Afección axial: Articulaciones S-I y columna lumbar.
• Entesopatía, tenosinovitis.
• MANIFESTACIONES GENITO-URINARIAS: Uretritis, balanitis circinada
• (es importante señalar que las lesiones son indoloras) , prostatitis
• Cervicitis, cistitis, enfermedad pélvica inflamatoria.

• MANIFESTACIONES CUTÁNEAS Y DE MEMBRANAS MUCOSAS


• Queratodermia blenorrágica.
• Eritema nodoso.
• Distrofia ungueal.
• Úlceras orales.

• MANIFESTACIONES OCULARES
• Conjuntivitis y uveítis.
• MANIFESTACIONES G-I

BIBLIOGRAFÍA RECOMENDADA:

• Klippel JH, Stone JH, Crofford LJ, White PH, editors. Primer on the rheumatic
diseases. 13th ed. New York: Springer-The Arthritis Foundation; 2008.

• Martínez-Elizondo P, editor. Introducción a la Reumatología. 4a ed. México: Colegio


Mexicano de Reumatología A.C./Intersistemas S.A. de C.V.; 2008.

• Firestein GS, Budd RC, Harris ED Jr, McInnes IB, Ruddy S, Sergent JS, editors.
Kelley’s Textbook of Rheumatology. 8th ed. Philadelphia: Saunders Elsevier; 2009.

64.- Se trata de masculino de 26 años, con antecedentes de episodios recurrentes de


dolor ocular, fotofobia y lagrimeo, que desarrolla dolor insidioso y progresivo en región
lumbar con exacerbación nocturna en cama. A la exploración física: test de Schöber
positivo y soplo de regurgitación aórtica grado II-III/VI. Rx de tórax: sugerente de
retracción fibrosa apical derecha. El diagnóstico más probable es:

a) Síndrome de Reiter.
b) Espondilitis anquilosante.
c) Osteoartrosis.
d) Enfermedad de Whipple.
• La espondilitis anquilosante es un padecimiento reumático inflamatorio,
generalizado y crónico, que afecta primordialmente el esqueleto axial, con la
presencia de daño de las articulaciones S-I (sacroiliitis) como su hallazgo
fundamental.
• PREVALENCIA: Africanos y esquimales: 0.1 %. Blancos: 0.5 – 1 %. Indios Haida
(Norte de Canadá): 6 %
• PREVALENCIA DEL HLA-B27: 6 – 8 %
• PREVALENCIA EN FAMILIARES DE ENFERMOS, HLA-B27 +: 10 – 20 %
• INCIDENCIA: 6.3 – 6.9/100 habitantes por año.
• CONCORDANCIA en gemelos: Monocigotos, 63 %; dicigotos, 12.5 %
• EDAD: Se inicia en la adolescencia o la edad adulta temprana. Rara después de los
40 años.
• GÉNERO: 3 – 5:1 a favor del hombre.
• SACROILIITIS:

™ Infiltrado de linfocitos T CD4+ y CD8+, así como macrófagos.


™ RNAm de TNF abundante cerca de los infiltrados.
™ RNAm de TGF-ß cerca de las áreas de formación de hueso nuevo.

™ Tejido de granulación en médula ósea subcondral.


™ Hallazgos precoces: Sinovitis, inflamación MO subcondral.
™ Hallazgos avanzados: destrucción extensa del cartílago y el hueso
subcondral.

• COLUMNA VERTEBRAL: Osteitis, sindesmofitos, acuadramiento.


• ENTESITIS
• ARTICULACIONES PERIFÉRICAS: Hiperplasia sinovial, infiltrado linfoide, fibrosis
y lesiones vasculares.

OCULARES: uveítis anterior. En la mayoría de los enfermos se produce durante los


primeros 10 años de evolución.
En general es unilateral, con tendencia a recidivar. Cursa con dolor, fotofobia y
lagrimeo. No suele dejar secuelas.
CARDIO-VASCULARES: aortitis ascendente, insuficiencia aórtica, anormalidades de
la conducción.
La más característica es la insuficiencia aórtica por inflamación de la aorta y de la
válvula aórtica. Es más frecuente en la espondilitis anquilosante de larga duración,
especialmente en las que cursan con artritis periférica importante y con
manifestaciones generales (fiebre, adelgazamiento y anemia). Otras manifestaciones
son la insuficiencia cardiaca, la cardiomegalia y los defectos de conducción.
• PULMONARES: fibrosis apical
• GASTRO-INTESTINALES: EII
• NEUROLÓGICAS: Complicaciones por fracturas y luxaciones, síndrome de la “cola
de caballo”

• RENALES: Amiloidosis, nefritis por IgA, uso de AINE, prostatitis inespecífica


• OTRAS.

Referencias:

Braun J, et al. Arthritis Rheum 1995;38:499-505.

65.- Femenino de 43 años que desde hace 1 mes inicia con debilidad a nivel de cintura
escapular y pelviana. En la piel se aprecia edema palpebral y coloración eritematoviolácea
periorbitaria así como lesiones eritematodescamativas sobre prominencias ósea de dorso
de manos. El diagnóstico más probable en esta paciente es:

a) Dermatomiositis.
b) Eritema polimorfo.
c) Lupus eritematoso sistémico.
d) Artritis reumatoide.

DERMATOMOSISTIS
Cuadro Clínico:
- Alteraciones musculares. Vienen marcadas por la presencia de debilidad muscular aguda
o subaguda (generalmente de inicio insidioso), simétrica y difusa, con preferencia por
musculatura proximal de extremidades (cintura pélvica y escapular), tronco y cuello. En la
mayoría de los casos es indoloro. Con el tiempo, desarrollan atrofia, contracturas y
disminución de los reflejos.
- Alteraciones cutáneas. La más frecuente en la DM es una erupción cutánea
eritematoviolácea que afecta a cuello, cara y tórax. Es característico también, el eritema
heliotropo (en párpados), que puede extenderse a otras zonas fotoexpuestas), las pápulas
de Gottron (localizadas en los nudillos),
telangiectasias periungueales, a veces ulceración dérmica y calcinosis (fundamentalmente
en la DM infantil).
- Articulares. Artralgias, artritis transitorias, no erosivas, con tendencia a la simetría.
- Otras. Afectación cardiaca variable (alteración ECG, arritmia, miocarditis), pulmonar
(fibrosis intersticial asociada con anti Jo-1), renal (muy rara), fenómeno de Raynaud.

Diagnóstico:
- Analítica: aumento de VSG y de enzimas musculares, (CPK, aldolasa, GOT, GPT, LDH). La
CPK es la más sensible y la que guarda una mejor correlación clínica con la actividad de la
enfermedad y la valoración de recaídas. El FR es + en 20% y
ANA es + en 10-30%. Si la destrucción muscular es intensa, puede producir mioglobinuria.
- Destacan anticuerpos: • anti-Jo1: en casos de PM asociado a neumonitis intersticial
(síndrome antisintetasa-miosistis, fibrosis pulmonar, artritis no erosiva y fenómeno de
Raynaud).

• anti-PM1 o PM-Scl: asociación con esclerodermia.


• anti-Mi, en DM.
• antimioglobina.

BIBLIOGRAFÍA RECOMENDADA:

• Klippel JH, Stone JH, Crofford LJ, White PH, editors. Primer on the rheumatic
diseases. 13th ed. New York: Springer-The Arthritis Foundation; 2008.

• Martínez-Elizondo P, editor. Introducción a la Reumatología. 4a ed. México: Colegio


Mexicano de Reumatología A.C./Intersistemas S.A. de C.V.; 2008.

• Firestein GS, Budd RC, Harris ED Jr, McInnes IB, Ruddy S, Sergent JS, editors.
Kelley’s Textbook of Rheumatology. 8th ed. Philadelphia: Saunders Elsevier; 2009.
66. - Masculino de 56 años que inicia con dolor precordial súbito irradiado a brazo
izquierdo, cuello y madíbula, sudoración profusa y ansiedad, con antecedentes de
tabaquismo postivo de 20 años de evolución consumiendo 1 cajetilla al día, obesidad grado I,
niega otras patologías. Se sospecha de IAM por lo que se procede a toma de ECG, el
siguiente trazo electrocardiográfco es compatible con:

a) IAM Postero-lateral
b) Angina inestable
c) IAM lateral Alto
d) IAM cara anterior

Electrocardiograma (EKG) de 12 derivaciones de un paciente con un infarto agudo del


miocardio de cara anterior. Nótese el supradesnivel convexo del segmento ST desde V1-
V5, con elevación del punto J que alcanza los 6 mm en V3.
Diagnóstico topográfico del infarto agudo de miocardio (IMA): 5, 6

Infarto anterior extenso: DI, aVL, V1, V2, V3, V4, V5, V6
Infarto anteroseptal: VI, V2, V3, V4
Infarto anterolateral: DI, aVL, V4, V5, V6
Lateral alto: DI, aVL
Lateral bajo: V5, V6
Infarto inferior: Se observa en DII, DIII, aVF
Infarto posterolateral: DII, DIII, aVF, V5 y V6

67.-El dolor abdominal intermitente, la anemia, la proteinuria, la necrosis tubular, la


esterilidad, los trastornos de la conducta, la polineuropatía periférica se encuentran en la
intoxicación por:

a) Organoclorados.
b) Sulfuro de hidrogeno.
c) Plomo.
d) Etilenglicol.

La intoxicación por plomo es la más común de las exposiciones a metales, el cual tiene
muchos usos, las fuentes más frecuentes vienen de las minas y del reciclado de materiales
conteniendo plomo. Este metal es absorbido por pulmones y del tracto gastrointestinal. El
mecanismo de acción es por unión a los grupos sulfhidrilo y tóxico para las enzimas
dependientes de zinc. Diagnóstico: La toxicidad aguda se presenta luego de una exposición
respiratoria a altas concentraciones, con encefalopatía, insuficiencia renal y síntomas
gastrointestinales. La toxicidad crónica es la más frecuente y se manifiesta con
compromiso multisistémico: hematopoyético, del sistema nervioso, gastrointestinal, riñón y
sistema reproductor. (Astenia, dolor abdominal, irritabilidad, náusea, vómitos, pérdida de
peso, cefalea, anemia, neuropatía periférica, ribete de Burton, IRC, proteinuria, Nefritis
intersticial,etc). En los exámenes auxiliares podemos encontrar anemia, punteado basófilo,
aumento del ácido úrico, etc.
REFERENCIA BIBLIOGRÁFICAS:

1. Keogh JP y Boyer LV. «Lead» en Sullivan y Krieger editores: Clinical Environmental


Health and Toxic Exposures. Lippincott Williams & Wilkins, 2da edición, 2001.
2. Decreto Supremo N° 019-98-MTC. Dispone eliminar del mercado la oferta de gasolina 95
RON con plomo y reducir el límite máximo de contenido de plomo en la gasolina 84 RON.
(14/07/98)
3. Shannon Michael. «Lead» en Haddad, Shanon y Winchester editores: Clinical
Management of Poisoning and Drug Overdose. WB Saunders, 3ra edición, 1998.
4. Krantz A, Dorevitch S. Metal exposure and common chronic diseases: A guide for the
clinician. Dis Mon 2004; 50:215- 262.
5. Nogué S. Burton’ s Line. N Engl J Med 2006; 354:e21.
6. Rempel D. The lead-exposed worker. JAMA 1989; 262:532-4.

68.- Usted es un cirujano general y descubre que haciendo algunas modificaciones a una
técnica quirúrgica disminuye el tiempo de uso de quirófano, pero quiere comparar si tiene
los mismos resultados clínicos que cuando aplica la técnica clásica , ¿Qué tipo de estudio
debe realizar?

a) Casos y controles.
b) Cohorte.
c) Ensayo clínico controlado.
d) Transversal.

Los ensayos clínicos controlados son estrategias diseñadas para evaluar la eficacia de un
tratamiento en el ser humano mediante la comparación de la frecuencia de un determinado
evento de interés clínico o desenlace en un grupo de enfermos tratados con la terapia en
prueba con la de otro grupo de enfermos que reciben un tratamiento control.

Referencias:

Calva M.J.J. Estudios Clínicos Experimentales. Salud Pública de México vol.42, núm. 4,
2000 (349).

69.- Se siguieron por un periodo de 10 años a 500 hombre hipertensos y 500 hombres sin
hipertensión para detectar evento vascular cerebral (EVC). Durante el estudio 80 de los
hipertensos tuvieron EVC (Incidencia de 160 por 1000) y 30 sin hipertensión presentaron el
evento (Incidencia 60 por 1000), el RR fue de 2.66. Este es un ejemplo de un estudio:

a) Cohorte.
b) Transversal.
c) Ensayo clínico controlado.
d) De casos y controles.

En los estudios de cohorte se eligen dos grupos uno de expuesto y otro de no expuestos los
cuales son seguidos a través del tiempo para detectar las posibles consecuencias.
Posteriormente se analiza la información calculado la incidencia en el grupo de expuestos y
en el grupo de no expuestos; y una vez obtenidos se calcula el Riesgo Relativo.

Referencia:

Ruiz M. A. Epidemiología Clínica, Panamericana, 1ª. Ed. 2004; págs: 287-289.

70.- Masculino de 65 años es trasladado por ambulancia al servicio de urgencias,


encontrado en la vía pública una madrugada del reciente invierno, con fuerte aliento
alcohólico, sólo responde a estímulos dolorosos, hemiplejía braquiocrural derecha.
Frecuencia cardiaca irregular. Se ingresa y se realiza un ECG. Según los siguientes trazos,
el diagnóstico cardiológico es:

a) Fibrilación auricular.
b) IAM cara posterior.
c) Fibrilación ventricular.
d) Angina estable.
Electrocardiograma:
- Frecuencia cardiaca 65 lpm.
- AQRS -20°, aunque cuesta definirlo categóricamente, son ejes llamados indeterminados.
- Ausencia de onda P.
- Línea de base con oscilaciones irregulares:
- en DII y aVR son a muy alta frecuencia (corresponden a temblor muscular).
- en las otras derivaciones, la frecuencia y configuración son compatibles con fibrilación
auricular.
- Los complejos QRS en V4-6 tienen voltaje aumentado, aunque no llegan a cubrir los
criterios de crecimiento ventricular izquierdo.
- En la porción final del QRS están encerradas en círculo rojo las ondas J de
Osborn.
- Segmento ST y onda T de difícil definición en DI-II, aVL, aVF (alteraciones difusas en la
repolarización ventricular).
La hipotermia produce enlentecimiento en la despolarización ventricular – prolongación en la
fase 2 del potencial de acción-, por lo que se prolonga el periodo refractario; induciendo la
aparición de las ondas J de Osborn. Las fibras auriculares pueden responder con fibrilación
cuando son expuestas al frío.
El solo tomar helado y sentir el dolor retroesternal está asociado a inversión en la onda T
en DII-III.
Las ondas J de Osborn –o simil- también se observan en la hipercalcemia y la angina
vasoespástica.
71.- Femenino de 39 años de edad la cual inicia con un cuadro de exoftalmos axial de varias
semanas de evolución, con predominio en su ojo derecho. En la exploración se aprecia una
conjuntiva con síntomas discretos de hiperemia y edema, una queratitis de carácter
punteado en tercio inferior corneal y se sospecha una retracciónpalpebral al observar cómo
el borde del párpado superior se encuentra por encima del limbo, permitiéndonos visualizar
la esclerótica. La paciente no refiere diminución de visión ni alteraciones tipo visión doble
y toma presión intraocular con parámetros dentro de los normales. ¿Cuál de los siguientes
diagnósticos le parece más compatible con el cuadro?

a) Tumor intraorbitario.
b) Enfermedad de Graves-Basedow.
c) Queratoconjuntivitis epidémica.
d) Tumor intraocular.

OFTALMOPATÍA TIROIDEA.
Es la causa más frecuente de exoftalmos tanto bilateral como unilateral en adultos. La
forma típica aparece en pacientes con enfermedad de Graves-Basedow, en los que podemos
encontrar exoftalmos y síndrome palpebro-retráctil. Este síndrome ocular puede aparecer
en pacientes eutiroideos o hipotiroideos, pudiendo constituir el signo más precoz de una
tirotoxicosis incipiente.
CLÍNICA.
Se distinguen dos formas clínicas:
a) Tirotóxica (hipersensibilidad a las catecolaminas y habitualmente hay hipertiroidismo):
exoftalmos moderado depresible.
Hay edema del contenido orbitario, pero no fibrosis ni oftalmoparesia.
b) Maligna (inflamación orbitaria autoinmune y puede haber normo o hipotirodismo):
exoftalmos irreductible severo con oftalmopejia progresiva y queratitis por exposición.
Puede afectar al nervio óptico por compresión y producir pérdida visual. Hay fibrosis de la
grasa y vientres musculares.
Manifestaciones oculares asociadas: retracción palpebral bilateral que permite ver la
esclera por encima del limbo, disminución de la motilidad palpebral, alteración de la
pigmentación palpebral, hiperemia conjuntival.

DIAGNÓSTICO.
Por los signos clínicos descritos y exploraciones complementarias, como la exoftalmometría
(medida de la protrusión ocular), la radiología (aumento de densidad de los tejidos blandos),
el engrosamiento del vientre de algunos músculos extraoculares (apreciados en la TC, la RM
y la ecografía orbitaria) y la analítica sistémica
Referencias:

AACE Thyroid Task Force. American Association of Clinical Endocrinologists medical


guidelines for clinical practice fo rthe evaluation and treatment of hyperthyroidism and
hypothyroidism. Endocr Pract. 2002;8(6).

Davies TF, Larsen PR. Thyrotoxicosis. In: Kronenberg HM, Melmed S, Polonsky KS, Larsen
PR, eds. Williams Textbook of Endocrinology. 11th ed. Philadelphia, Pa: Saunders Elsevier;
2008:chap 11.

72.- Mujer de 25 años de edad que presenta en la cara diseminado comedones, pápulas y
pústulas, crecimiento excesivo de vello en mejillas mentón y cuello, acompañada de
seborrea, refiere caída de pelo exagerada. Inició desde la adolescencia. ¿El diagnóstico
clínico es?

a) Acné e Hirsutismo.
b) Síndrome de masculinización.
c) Lupus eritematoso discoide.
d) Alopecia androgenética.

El hirsutismo es el crecimiento excesivo de vello terminal en mujeres siguiendo un patrón


masculino de distribución, en zonas andrógeno-dependientes: patillas, barbilla, cuello,
areolas mamarias, tórax, en área inmediatamente superior o inferior al ombligo, así como en
muslos, espalda. Frecuentemente se asocia a acné, calvicie con patrón masculino (alopecia
androgénica) e irregularidades menstruales.

Es un trastorno que afecta aproximadamente al 10% de las mujeres en edad fértil, y puede
ser leve, lo que representa una variación del patrón de crecimiento normal, y en raras
ocasiones es signo de un trastorno subyacente grave.

Por lo general, es idiopático, pero puede estar relacionado al exceso de andrógenos, como el
síndrome de ovario poliquístico o la hiperplasia suprarrenal congénita.

Los pacientes con acné presentan de un modo más o menos frecuente seborrea de cuero
cabelludo, con o sin pitiriasis.

Otras alteraciones andrógeno-dependientes pueden asociarse, pero no de un modo habitual,


como sucede con el hirsutismo y la alopecía, que pueden presentarse en menos del 10% de
las pacientes.
En el Síndrome de Cushing Hipofisario, la ACTH estimula la producción suprarrenal de
hormonas andro-génicas, resultando esto en acné e hirsutismo.

En el síndrome de poliquistosis ovárica (Stein-Leventhal), hay un incremento de la secreción


gonadal de andrógenos a nivel del hilio y de la teca interna. La hipersecreción de
andrógenos dotados de bioactividad puede inducir acné e hirsutismo. La presencia de acné
en una mujer -adolescente o adulta - con niveles de testosterona sérica superiores a 300
ng/dl sugiere la presencia de un tumor ovárico.

En la Anorexia Nerviosa, el acné se desencadena generalmente en la fase de recuperación,


donde puede coexistir con un ovario poliquístico. En fase de amenorrea LHRH, LH, FSH,
estrógenos y progesterona están disminuídos. El cortisol plasmático está elevado con vida
media prolongada y pérdida de la variación diurna, otro factor que puede producir acné.

No hay que olvidar aquellos síndromes donde el acné coexiste con importantes trastornos
osteoarti-culares, como ser el Síndrome de Apert y el Síndrome SAPHO.

El acné se halla generalmente presente en los pacientes epilépticos - tal vez provocado en
gran medida por la medicación - y en pacientes psiquiátricos (maníaco-depresivos y
obsesivo-compulsivos). Es importante tener en cuenta la asociación de una neurosis de
angustia inespecífica y el acné.

Referencia:

1. Azziz R, Carmina E, Sawaya ME. Idiopathic hirsutism. Endocr Rev 2000;21:347-62.


Hirsutismo. Gonzalez Guerra. Servicio de dermatología.Fundación Jiménez Díaz (Madrid)

73.- Femenino de 56 años con antecedentes de prótesis de cadera; no utiliza sus vendas
elásticas. Inicia súbitamente con disnea, taquipnea, dolor torácico, ansiedad y taquicardia.
Ud ordena una ECG que revela que el eje de QRS es mayor de 90° más taquicardia sinusal.
Ud sospecha:

a) IAM
b) Cor pulmonare
c) TEP
d) Infarto pulmonar
TEP
Los factores de riesgo más importantes son:
Infarto de miocardio, insuficiencia cardiaca congestiva e insuficiencia venosa
crónica.
Lesión por traumatismo, sobre todo fractura de miembros inferiores y huesos
largos.
Anticonceptivos y terapia estrogénica.
Edad > 40 años.
Inmovilización prolongada > 4 días aproximadamente.
Neoplasias con compresión del sistema venoso profundo y liberación de sustancias
procoagulantes.
Estados de hipercoagulabilidad primaria.
Accidentes cerebrovasculares.
Parto y puerperio.
Antecedentes de TEP y TVP.
Cirugía ortopédica, abdominal y neurológica.

Síntomas del TEP (%)


Disnea de aparición súbita 84
inexplicable
Dolor torácico de tipo pleurítico 76
Tos 50
Dolor en pantorrilla 39
Sudoración - ansiedad 36
Hemoptisis 28
Infartos pulmonares: dolor 10
pleurítico, roce pleural,
hemoptisis y fiebre.
Dolor no pleurítico 17
Síncope 13
Palpitaciones 10
Dolor anginoso 1
Asintomático --
Sígnos del TEP (%)
Taquipnea (> 20 r.p.m) 85
Taquicardia (> 100 l.p.m) 58
Aumento del 2º tono pulmonar 57
Estertores pulmonares 55
Fiebre > 37.5ºc 50
Signos de TVP en extremidades 41
inferiores
Roce pleural 18
Cianosis 18
Hepatomegalia 10
Reflujo hepatoyugular 5

ECG. En el que podemos encontrar:

Inversión de la onda T en derivaciones precordiales derechas.


Taquicardia sinusal.
Bloqueo de rama derecha.
Arritmias supraventriculares.
Trastornos de la conducción.
Clásico patrón S1 Q3 T3 que es raro e indicativo de hipertensión pulmonar.
Eje de QRS es mayor de 90°

Bibliografía:

• Barranco Ruiz F. Martos López J. Simon Martos B. Tromboembolismo pulmonar


masivo. En: Principios de Urgencias, Emergencias y Cuidados Críticos. [Internet].
Universidad de Burgos. [Fecha de consulta 28-7-2003]. Disponible en:
http://www.uninet.edu/tratado/c0208i.html.
• Belle L, Martin M, Brunier S, Brunet N, Bosson JL, Gros C et al. Evaluation d'un
algorithme diagnostique de l'embolie pulmonaire non grave avec successivement: D-
Dimeres, echodoppler veineux des membres inferieurs et tomodensitometrie
helicoidale au centre hospitalier d'annecy.Ann Cardiol Angeiol (Paris). 2002
Nov;51(5):243-7. [Medline]
• Cereza G, Danés I. Eficacia y seguridad de Heparinas de bajo peso molecular en el
tratamiento de la tromboembolia pulmonar. Med Clin (Barc) 1999; 113: 115-116.
[Medline]
• Conthe Gutierrez P, Lobos Bejaranoa JM, Alonso Garcia A. Tromboembolismo
pulmonar. FMC 1998; 5 (2): 89.
• Dalen JE. Trombolytic therapy in patients with submassive pulmonary embolism. N
Engl Med 2003 Jan 23; 348(4): 357-359.
• Farrell S, Hayes T, Shaw M A negative SimpliRED D-dimer assay result does not
exclude the diagnosis of deep vein thrombosis or pulmonary embolus in emergency
department patients. Ann Emerg Med 2000; 35:121-5.
74.- Masculino de 66 años, con Insuficiencia Cardiaca por cardiopatía hipertensiva, en
situación estable (en clase funcional I según grado de disnea), presenta en el Ecocar-
diograma, Disfunción Sistólica (Fracción de Eyección < 35%). ¿Qué grupo de fármacos
estaría más indicado como tratamiento inicial?

a) Digitalicos.
b) Inhibidores de la ECA.
c) Antagonistas de Calcio.
d) Betabloqueantes.

ƒ IC Izquierda:
à Disnea al ejercicio, tos, fatiga, ortopnea, DPN, cardiomegalia, estertores,
ritmo de galope, congestión venosa pulmonar.
ƒ IC Derecha:
à Presión venosa elevada, hepatomegalia, edema, usualmente asociado a IC
Izq.
TIPOS:
• Insuficiencia Cardíaca Aguda

– Edema pulmonar agudo


– Shock Cardiogénico

• Insuficiencia Cardíaca Crónica

Estado Fisiopatológico Caracterizado por la incapacidad del corazón para bombear la


cantidad de sangre necesaria para abastecer el metabolismo celular.
La ICC representa un síndrome clínico complejo caracterizado por alteraciones de función
ventricular y de regulación neurohormonal que se acompaña de:
-intolerancia al esfuerzo.
-Reducción en la calidad de vida.
-Reducción de la esperanza de vida.

SINTOMAS:
• Disnea
• Disminución capacidad funcional
• Síntomas urinarios
• Síntomas cerebrales
• Síntomas insuficiencia cardíaca derecha
PROPUESTA DE UN ESQUEMA DE TRATAMIENTO 

Fibrilación A. Diurético

contraindicación o efecto 
I ECA adverso

ARA
Siguen síntomas BB

Espironolactona 12,5 a 25 mg/d Preferible ingreso hospitalario 
o inicial con seguimiento 
mensual
Digoxina 0,125 mg a 0,25 mg/d

ƒ Se ha demostrado claramente la importancia del control neurohumoral en el


paciente con disfunción ventricular.

ƒ Manejo inicial del paciente con ICA


= Diurético + Vasodilatador.

ƒ Solo el paciente con ICA + BGC = Inotrópico.

ƒ Nuevos estudios clínicos e investigación básica se requiere para buscar nuevas


estrategias de manejo.

REFERENCIAS:

Los inhibidores de la enzima conversora de angiotensina Rev Cubana Cardiol Cir Cardiovasc
1997:11;29-47.

75.- Se trata de femenino de 39 años la cual presenta amenorrea secundaria de 2 años y


medio de evolución. Los niveles reportados de prolactina son de 150ng/ml (normal hasta
20 ng/ml). La resonacia magnética detecta macrotumor de 2,8 cm. de diámetro con
expansión lateral izquierda. No alteraciones visuales. ¿Cuál sería el tratamiento de elección?

a) Cirugía por tratarse de un macrotumor.


b) Tratamiento médico con agonistas dopaminérgicos.
c) Somatostatina previa a cirugía.
d) Radioterapia hipofisaria previa a cirugía.
Tratamiento
Los dopaminérgicos han revolucionado el tratamiento del prolactinoma y virtualmente han
dejado fuera a la cirugía; así independientemente del tamaño del adenoma la primera opción
terapéutica es la farmacológica.1,2 Con los dopaminérgicos se consigue en poco tiempo
restaurar el funcionamiento ovárico y corregir la esterilidad, incluso antes de que se
normalice la concentración de prolactina; asimismo se consigue reducir el tamaño del
adenoma.
Los dopaminérgicos actúan sobre los receptores localizados en las células mamotrópicas de
la hipófisis anterior y suprimen la síntesis y secreción de prolactina con la consecuente
normalización del eje hipotálamogonadotropico hipotálamogonadotropico- ovárico. La acción
dopaminérgica puede ocasionar efectos colaterales indeseables como náusea, hipotensión
arterial, constipación nasal, mareo y estreñimiento, los cuales no necesariamente
corresponden con la dosis utilizada, pero sí se relacionan con el tipo de dopaminérgico. Está
ampliamente documentada la superioridad de la farmacoterapia para el tratamiento de los
prolactinomas; además la cirugía es raramente curativa, incluso en el caso de
microadenoma.4,5.

Cuadro I. Agentes dopaminérgicos que se usan como tratamiento de la


hiperprolactinemia y el prolactinoma.
Genérico Comercial Dosis (mg)
Bromocriptina Parlodel 2.5-5 diaria
Lisurida Dopergin 0.2 diaria
Quinagolida Norprolac 25-50 diaria
Cabergolina Dostinex 0.5 c/4 días

Los adenomas hipofisarios representan el 10% de todos los tumores intracraneales


diagnosticados y 25% de los tumores cerebrales que son intervenidos quirúrgicamente. Los
objetivos del tratamiento de un paciente con un adenoma de la pituitaria son: eliminar el
efecto de la masa tumoral (compresión sobre estructuras vecinas) disminuir la producción
excesiva de hormonas, restaurar la función normal de la pituitaria y evitar la recurrencia.

El tratamiento de elección para todos los prolactinomas es con un agonista de la dopamina.


La bromocriptina y la cabergolina son efectivas para reducir el tamaño del tumor y para
restaurar la función gonadal. El tratamiento quirúrgico debe recomendarse sólo cuando
falla el tratamiento médico.

Los tumores de la pituitaria productores de hormona de crecimiento son tratados


preferentemente mediante adenomectomía transesfenoidal, pero la normalización de los
niveles de HC y de IGF-1 ocurre en menos de la mitad de los pacientes con macroadenomas;
por lo tanto, un importante número de pacientes acromegálicos requiere un tratamiento
adicional. Los análogos de la somatostatina son en la actualidad los medicamentos que más
usados para el control de la acromegalia. En grupos especiales de pacientes, el tratamiento
con agonistas de la dopamina y somatostatina parece que suprimen mejor los niveles de HC
que cuando se administran esos fármacos en forma separada.
Figura 1. Paciente de 20 años quien consultó por amenorrea primaria. El estudio de
RMN muestra un macroprolactinoma que invade el seno cavernoso izquierdo y envuelve
la carótida del mismo lado.

RMN de control 10 meses después de tratamiento con un agonista dopaminérgico.


Corte coronal en T1. No se observa tumor. Tallo hipofisario central y quiasma óptico
libre.

REFERENCIAS BIBLIOGRÁFICAS

1.-Gac Méd Méx Vol. 140 No. 5, 2004

2.- Schlechte JA. Prolactinoma. N Engl J Med 2003;349:2035-2041.


3.- Zárate A, Canales ES, Jacobs LS, Soria J, Daughaday WH. Restoration of ovarian
function in patients with the amenorrhea-galactorrhea syndrome after long-term therapy
with L-Dopa. Fertil Steril 1973;24:340.
4.- Tyson JE, Carter JN, Andreassen B, Huth J, Smith B. Nursing mediated.
76. - A 4-year-old boy presents with a history of constipation since the age of 6 months.
His stools, produced every 3-4 days, are described as large and hard. Physical examination
is normal; rectal examination reveals a large ampulla, poor sphincter tone, and stool in
rectal vault. The next step in the management of this infant would be.

a) Lower GI barium study.


b) Parental reassurance an counseling in Hirschprung’s disease.
c) Serum electrolyte measurement.
d) Upper GI barium study.

Expresión Clínica:

Dentro de los síntomas que permiten una sospecha precoz, se encuentra el estreñimiento o
constipación, definida en el recién nacido como el retraso en la eliminación de meconio
mayor a 48 horas asociada a distensión abdominal, y en los niños mayores como
deposiciones infrecuentes de consistencia aumentada (. El 98% de los lactantes elimina el
meconio en las primeras 48 horas de vida. Los prematuros eliminan más tardíamente el
meconio, pero la EH es rara en prematuros. De los pacientes con EH, sólo el 60% elimina el
meconio después de las 48 horas, por lo que este signo no es patognomónico de la
enfermedad.
La mayoría de los niños que presentan aganglionosis congénita, son sintomáticos los
primeros días o las primeras semanas luego del nacimiento (2). Alrededor de dos tercios de
los pacientes presenta síntomas dentro de los tres primeros meses de vida y 80%
desarrolla síntomas dentro del primer año de vida. Sólo un 10% de los pacientes inicia
síntomas entre los 3 y 14 años de edad y en general se trata de pacientes con enfermedad
de segmento ultracorto. (13)
Los recién nacidos y lactantes pequeños presentan con frecuencia signos de obstrucción
intestinal, distensión abdominal, vómitos biliosos e intolerancia a la alimentación. La
inspección anal y la radiografía pueden orientarnos hacia una causa mecánica de obstrucción,
pero no descarta EH. Si la obstrucción no tiene una causa mecánica, además de pensar en
una EH, debe plantearse el diagnóstico diferencial con hipotiroidismo, insuficiencia
suprarrenal, hipokalemia, hipercalcemia, hipomagnesemia, y en casos excepcionales
alteraciones neuromusculares.
Cuando la sintomatología es poco evidente,
Puede presentarse como un cuadro de constipación crónica, con historia de dificultad en la
eliminación de deposiciones, masas fecales palpables en fosa ilíaca izquierda y un tacto
rectal en que no se encuentran deposiciones en la ampolla rectal y esfínter anal hipertónico.
En muchas ocasiones la estimulación rectal provoca salida explosiva de heces líquidas de
olor fétido (17). Por lo tanto, frente a pacientes con constipación crónica, en los cuales se
ha descartado causa mecánica de obstrucción intestinal, que no cede a las medidas
dietéticas ni farmacológicas, debe plantearse el diagnóstico de:
EH. También puede encontrarse dilatación de asas intestinales, adelgazamiento de la pared
abdominal, alteraciones de la nutrición y el crecimiento.
En niños mayores, los síntomas más comunes incluyen constipación crónica progresiva,
impactación fecal recurrente, mal incremento ponderal y malnutrición.

Rev. Ped. Elec. 2008, Vol 5, N° 1. ISSN 0718-0918.


Servicio Salud Metropolitano Norte.
Facultad de Medicina Hospital Clínico de Niños.
Departamento de Pediatría y Cirugía Infantil Roberto Del Río.

Referencias:

1. De Manueles J. Enfermedad de Hirschsprung.


Protocolos diagnósticos y terapéuticos en pediatría. Sociedad Española de Pediatría. Pag.
56-60.
2. Feldmon T., Wershil B. Hirschsprung Disease. Pediatrics in review. Vol 23. N 11, August
2003.
3. Luis L.A., Encinas J.L., Avila L.F., et cols.
Enfermedad de Hirschsprung: enseñanzas de los últimos 100 casos. Cir Pediatr 2006;
19:177- 181.
4. J.M. Gil-Vener y cols. Diagnóstico dieferncial de Hirschsprung-neurodisplasia intestinal.
Fiabilidad de las pruebas diagnósticas. Cir Pediatr 2006; 19: 91-94.
5. M. López, y cols. Índices de fiabilidad de la manometría anorrectal para el diagnóstico de
la enfermedad de Hirschsprung en cualquier edad. Cir Pediatr 2005; 18:13-16.
6. Hernández F., Rivas S., Ávila L.F., Díaz M., ET cols. Aganglionismos extensos.
Tratamiento y resultados a largo plazo. Cir Pediatr 2003; 16: 54-57.
7. Goulet O. y cols. Intestinal transplantation in children: preliminary experience en Paris.
JPEN.
J Parenter Enteral Nutr 1999; 23 (5 Suppl)
8. Peña A. Enfermedad de Hirschsprung. Los avances y las preguntas no contestadas. Cir
Pediatr 2002: 15:46-47.
9. Polliotto S, Heinen F, Anduna G, Korman R. Evaluación de resultado a tres años de
nuestra primera experiencia en el tratamiento laparoscópico de la enfermedad de
Hirschsprung. Cir Pediatr 2001; 14: 85-87.
77.- Un hombre de 50 años acude al servicio de urgencias por presentar fiebre de 39.0c,
exantema máculo-papuloso generalizado, incluyendo palmas y plantas. El paciente labora en
el campo ordeñando vacas frecuentemente parasitadas por garrapatas. Señale la
enfermedad a la que se refiere, el germen causante y el tratamiento adecuado:

a) Kala-azar, Leishmaniae Donovani: Antimoniales.


b) Fiebre Q, Coxiella Burnetti, Doxiciclina.
c) Fiebre botonosa, Ricckettsia Conori: Doxiciclina.
d) Fiebre de Malta, Brucella Mellitensis: Cotrimoxazol.

Las rickettsias son organismos coco-bacilares, Gram negativos, de 2-3 micras de diámetro,
intracelulares, incapaces de crecer en ausencia de células vivas del huésped.
El género Rickettsia es ubicado taxonómicamente en la familia Rickettsiaceae, junto a
otros dos géneros:
Coxiella, con la especie C.burnetii, responsable de la fiebre.
Q, y Ehrlichia con las especies E.chaffeensis, agente de la ehrlichiasis monocítica, y
E.phagocytophila, productora de la ehrlichiasis granulocítica humana.
Junto a la familia Rickettsiaceae está la familia Bartonellaceae con tres especies
principales: Bartonella henselae, agente de la enfermedad por arañazo de gato (“cat
scratch disease”); B.quintana, responsable de la angiomatosis bacilar(1), y B.bacilliformis,
productora de la bartonelosis o enfermedad de Carrión (verruga peruana).
Las rickettsiosis son zoonosis transmitidas desde los huéspedes o reservorios animales al
hombre a través de picaduras de artrópodos diversos, que varían con cada enfermedad
(piojos, pulgas, garrapatas, otros ácaros, esencialmente).
La rickettsiosis por R.conorii es conocida con el nombre de fiebre botonosa o manchada del
Mediterráneo o fiebre de Marsella, siendo transmitida al hombre desde el perro que
constituye su reservorio por garrapatas de los géneros Amblyomma y Riphicephalus
principalmente. En ellas el germen cumple un ciclo que incluye el pasaje transovárico a los
descendientes por lo cual representan también otro verdadero reservorio del parásito.
Clínicamente la enfermedad se caracteriza por la aparición en el sitio de la picadura de la
garrapata de una lesión inicial indurada con centro necrótico muchas veces (“mancha negra”
o “tache noir”) rodeada de aureola inflamatoria, seguida de adenopatías regionales de
carácter inflamatorio en los días subsiguientes. Concomitantemente, fiebre
frecuentemente alta de 39º-40ºC, malestar general, cefaleas a veces intensas, dolores
musculares y articulares.
Es relativamente frecuente la observación de un exantema máculo-papuloso que explica el
nombre de fiebre botonosa y que puede afectar varios territorios. Es una afección
endémica en Sudáfrica, Europa del Sur y Medio Este (2). El diagnóstico se confirma
esencialmente por la técnica de inmunofluorescencia indirecta (IFI) empleando láminas que
contienen antígenos de R.conorii y utilizando, siempre que sea posible, sueros pareados
obtenidos al inicio del cuadro y 20-30 días después para investigar la seroconversión. La
histopatología de las lesiones iniciales o “taches noires” fue estudiada en detalle por

Montenegro y colaboradores en 1983(3). En otro trabajo posterior, Montenegro y


colaboradores (4) demuestran en ratones inoculados con R.conorii la importancia crucial de
la inmunidad celular con respecto a la humoral en el control de la infección experimental y
reducción del índice de mortalidad. El ratamiento de elección es dicloxacilina, alternativas
macrólidos y quinolonas (ciprofloxacino).

Bibliografía:

1 . Sampaio SAP, Rivitti EA. Dermatologia. São Paulo: Artes Médicas, 1998: 1155.
2 . Harris RL, Kaplan SL, Bradshaw MW, Williams Jr, Temple W. Boutonneuse fever in
american travelers. J Infect
Dis 1986; 153:126-8.
3 . Montenegro MR, Mansueto S, Hegarty BC, Walker DH. The histology of “taches
noires” of boutonneuse fever and demonstration of Rickettsia conorii in them by
immunofluorescence. Virchows Arch (Pathol Anat) 1983; 400:309-17.
4 . Montenegro MR, Walker DH, Hegarty BC. Infection of genetically immunodeficient
mice with Rickettsia conorii . Acta
Virol 1984; 28:508-14.
5. Conti Díaz IA, Rubio I, Somma Moreira RE, Pérez Bormida G. Rickettsiiosis
cutáneo-ganglionar por Rickettsiaconorii en el Uruguay. Rev Inst Med Trop (São Paulo),
1990.

78.- Femenino de 62 años con diagnóstico de miastenia gravis. ¿Cuál de los siguientes
medicamentos está dirigido al manejo de esta patología?

a) Neostigmina.
b) Quinidina.
c) Sumatriptán.
d) Succinilcolina.

Manejo farmacológico: Existen diferentes pautas terapéuticas dirigidas a contrarrestar


los síntomas de la enfermedad o el mecanismo inmunológico. Los fármacos utilizados son:

- Inhibidores de la Acetilcolinesterasa (Neostigmina, Piridostigmina). Dirigidos al manejo


sintomático de la MG, mejorando la fuerza motora pero no la progresión de la enfermedad.
Su mecanismo de acción es la inhibición reversible de la acetilcolinesterasa, lo cual genera
un aumento de ACh en la placa motora. La dosis a utilizar es variable y debe modificarse en
distintas etapas de la enfermedad, incluso siendo frecuente no lograr un efecto uniforme
en los diferentes grupos musculares en un mismo paciente. El objetivo por lo tanto será
utilizar la dosis mínima con la que se genere la mejor respuesta clínica. El efecto se obtiene
de 30 minutos a 2 horas de la administración y tiene una duración de hasta 6 horas. Las
dosis recomendadas de Piridostigmina son de 15-60 mg cada 4-6 horas vía oral y de
Neostigmina 0,5-2 mg/kg cada 4-6 horas intramuscular. Las reacciones adversas asociadas
son: dolor abdominal, hipersalivación, aumento de las secreciones respiratorias y
bradicardia y se relacionan con el efecto colinérgico generado, por lo que es necesario
administrar concomitantemente atropina2,3.

Referencias:

1.- Ponsetia JM: Miastenia Gravis. Manual Terapéutico. Barcelona; Springer Verlag Ibérica,
1995.

2.- Ponsetia JM, Espina E, Armengola M: Diagnóstico y Tratamiento de la Miastenia grave.


Med Clin (Barc) 2000; 115: 264-70.

3.- Drachman DB: Myasthenia gravis. N Engl J Med 1994; 330: 1797-810.

4.- Andrews PI: Autoimmune myasthenia gravis in childhood. Semin Neurol 2004; 24: 101-
10

5. - Anlar B: Juvenile myasthenia: diagnosis and treatment. Paediatr Drugs 2000; 2: 161-
9.

6. - Gajdos P: Myasthenic syndrome. Diagnosis trends. Rev Prat 2000; 50: 419-23 7.

79.- El examen microscópico de una biopsia pulmonar de un paciente con mucormicosis


pulmonar mostrará:

a) Micelio Dicotomizado Septado Con Conidias


b) Micelio Dicotomizado Hialino Cenocitico
c) Micelio Con Clamidoconidios Y Blastoporas
d) Micelio Septado Y Esclerotes De Mediar

Examen directo. Se realiza a partir de exudados o secreciones nasales, expectoración,


lavados bronquiales y heces, inclusive se puede hacer a partir de biopsias. La muestra se
debe aclarar con KOH al 10 %. Al microscopio se observan numerosas hifas no tabicadas,
hialinas, dicotómicas, de aproximadamente 5F de ancho por 20-25 de largo, esta imagen es
patognomónica.

Referencia:

Harada M, Manabe T, Yamashita K, Okamoto N. Pulmonary mucormycosis with fatal massive


hemoptysis. Acta Pathol 1992;42(1):49-54.
80.- Al realizar la exploración clínica y colocar un diapasón que está vibrando frente al
conducto auditivo del oído que queremos explorar (conducción aérea) y apoyando después
sobre la mastoides (conducción ósea), podemos de modo sencillo y en la consulta, distinguir
entre sordera nerviosa (alteración en la cóclea o nervio auditivo) y sordera de conducción
(trastorno en el sistema de transmisión tímpano-osicular). ¿Cuál de estas afirmaciones es
correcta para un paciente que presenta una sordera de conducción?:

a) La percepción del sonido es igual por vía aérea que por vía ósea.
b) La percepción del sonido es mejor por vía aérea que por vía ósea.
c) La percepción del sonido es mejor por vía ósea que por vía aérea.
d) La percepción del sonido es indistinguible tanto por vía aérea como ósea.

MEDICIÓN CLÍNICA DE LA AUDICIÓN

La valoración audiológica mínima debe incluir la determinación de los umbrales de


conducción aérea y ósea, el umbral de recepción y la discriminación del lenguaje, una
timpanometría y pruebas de reflejos acústicos, que incluyan la prueba de deterioro de los
reflejos. La información obtenida por medio de estas técnicas permite determinar si hace
falta una mayor diferenciación entre la sordera neural y la sensorial.

La audición por conducción aérea se valora presentando un estímulo acústico mediante


auricular o altavoces. Una sordera o elevación del umbral de audición detectada por esta
prueba se puede deber a defectos en cualquier parte del aparato auditivo: pabellón
auricular, conducto auditivo, oído medio o interno, VIII par craneal o vías auditivas
centrales.

La audición por conducción ósea se valora colocando una fuente sonora (el vibrador de un
audiómetro o un diapasón) en contacto con la cabeza. El sonido produce una vibración a
través del cráneo, que alcanza las paredes óseas de la cóclea y estimula directamente el
oído interno. La audición por conducción ósea no atraviesa los oídos externos y medio y
permite valorar la integridad del oído interno, del octavo nervio craneal y de las vías
auditivas centrales.

Si aumenta el umbral de conducción del aire y el umbral de conducción ósea es normal, la


sordera es de conducción, mientras que si ambos umbrales aumentan por igual es de tipo
neurosensorial. Existen algunas formas de sordera mixtas con componente neurosensorial y
de conducción, en las que aumentan ambos umbrales, aunque el de la conducción aérea es
más significativo.
Referencia:

• Manual Merck 10. Edición en Español


Editors of The Merck Manual.

• Robert S. Porter, MD, Editor-in-chief.

• Justin L. Kaplan, MD, Senior Assistant Editor.

• Editorial Board of The Merck Manual.

You might also like